Вы находитесь на странице: 1из 126

In general, if you have been doing Kaplan q bank and BSS you will be very prepared.

I would say that the level of difficulty is around that of Q-bank..Don't let anyone scare you, we know how cutthroats can be.The exam is very do-able and is fair..I would say on average 12.5% is very easy q's, 12.5% are very very hard q's, and the rest falls in the middle... Make sure u do the sample cd that they send you, I had 2 q's exactly the same (word for word).. The stuff stressed on my exam was mostly path and pharm..I had a lot of mri's, and CT scans of stuff that u won't normally notice (ex. Quadratous lumborum, SVC, post limb of internal capsule, etc..) They had some weird shit with the cell bio and histo, stuff we never learned at Ross and stuff not in the review books. Time was not a problem on most of my sections, I did however finish 2 sections at the last minute.. Here are the q's I recall: 1. know your mediators (IL-2, IL-456, IL 10, TNf,TGF, IFN) 2. know cd 16 and 56 are killer cells 3. where both clotting pathways meet 4. acute MI what the gross and histo look like at all stages, 5. brachial plexus- I had ulnar nerve b/c interossei weren't working 6. know 2nd messengers-oxtyocin, alpha 1 7. herpes pneumonitis, with giant cells 8. splenic infarct, from where? 9. cardiac tamponade 10. go over jamie's sheets- 10 icm type q's 11. know the arachadonic acid pathway and enzymes and inhibitors 12. t(9;22) cml 13. lat rectus on mri 14. wilsons- they showed the kleiser fleicher ring 15. cystic hygroma- turners syn 16. mitochondrial inheritance -2 q's 17. know everything about digoxin 4 q's 18. pku 19. b-cell def- no germinal centers in LN 20. know the cyto p450 inducers and inhibitors - 6 q's 21. scleroderma- ab against? 22. I had no sle 23. bph and prostate cancer- drugrs, recepters, - 3q's 24. ebv in Aids cancer- lymphoma

25. know your endocrine physio- hormones up and down 26. know the resp. curves in first aid, where would fibrosis be? 27. know histo of the hepatocyte- fxn of organelle- I had mitochiondria 28. know histo of kidney- pct, dct, loh- where glucose is reabsordeb, where vit d is activated 29. restrictive cardiomyopathy 30. p. carinii -know what it looks like 31. marasmus 32. carcinogens in bladder cancer 33. schistosomes in bladder cancer 34. ASCVD-2 q's 35. prinzmetal angina- drugs, recepters, -2 q's 36. endocarditis-viral, bacterial 37. India ink-crypto 38. ida, anemia of chronic dz 39. vwb dz- whats up? 40. henoch-scholheim purpura on the leg-vasculitis 41. pathogenesis of TB 42. didn't have a lot of behavioral but what I did have was alzheimers, and a few "what do u tell the pt" q's 43. hand foot mouth dz- coxsackie 44. cherry red macula and European-hexoaminidase a2 45. gauchers 46. tuberous sclerosis- 2 q's (when did we learn this)- know it is assoc. w/ embryonal rhabdomyosarcoma and calcifications in brain 47. cancer met. to colon 48. crohns dz 49. hirshprungs dz- failure of neural crest cells 50. know your gi physio- 5 q's 51. moa of omeprazole 52. moa of vancomycin 53. moa of neostigmine 54. causes of malig. Hyperthermia 55. pathogenesis of spider angioma 56. malig. Htn- pic 57. min. change dz- vignette was classic 58. pid-chlamydia 59. carnitine def-s/s 60. transformation- dnase 61. fracture to fibula- loss of deep peronial nerve- loss of dorsiflexion- 2q's 62. damage to base of penis-blodd flow is lost in which part of urethra (weird q) 63. hypertinic bladder-whereis lesion? 64. where does seminoma drain? 65. know your fractures- what muscles pull them where- 3 q's 66. fracture to lt. 9-11 ribs- spleen 67. cavernous sinus thrombosis- fxn's lost

68. av nicking , banking- learnit well ,2q's 69. myasthenia gravis- 2 q's 70. hashimotos= 2 q's 71. dermatomes 72. 6-8-10, 8-10-12, lung -pleura relationship 73. rt. Atrial hypertrophy on cxr 74. paradoxical emboli 75. brca met. via axillary nodes 76. soft tissue sarcoma 77. hemoglobin c 78. hla b27- reiters syn 79. proteus- uti stones 80. pathogenesis of gall stone-supersat. Of cholesterol 81. acute panreatitis vignette-inc. amylase 82. nodular glomerulosclerosis in D.M. 83. know your vaccines, and active and passive immunizations 84. cohort study, case series 85. I had 5 q's on epidem. (ppv, spec, sens) 86. p=id 87. endometriosis 88. 2 bitemp. Hemianopsia's 89. know your goiters- toxic, hot, cold, non toxic, multitoxic 90. brached hyphi- dermatophyte 91. hyper -PTHism 92. cushings syn 93. addisons dz 94. duchenne's musc. Dystrophy- pedigree 95. osteo sarcoma 96. comparing osteomyleitis and osteo chondritis- weird q 97. pseudo gout 98. neuro syph- arguile robinson pupil 99. vitamins- b12, pyridoxine, and vit c 100. desmopressin in diabetes insipidus 101. know your murmers. How thy look normally and on a wiggers 102. I had no ekg-s 103. salmonella 104. mhc 1- tap ptotein 105. cavernous hemangioma, is it bad? 106. lot of angio tension -renin stuff- 4 q's 107. ipratropium in copd 108. colchicimein acute gout 109. filtration fraction up and down..why? 110. know your acid-base stuff- weird q's with combined.alcoholic that vomits and had diarrhea (what the !@#$#$?) know the values 111. know your lesions in the CNS- whats ipsi whats contra 112. little biochem on the exam-know the rls's

First of all, there were 20-30 slides, about half were radiographs. Anatomy/Histo/Embryo differential dx for eosinophilia slide of plasma cell lecithin:sphingomyelin ration in fetuses (several questions on this) Rathke's pouch neurohypophysis derived from ectoderm what causes increased left atrial pressure and decreased right atrial pressure after birth several branchial apparatus questions question about remnant of thyroglossal duct Meckel's Diverticulum mesonephric vs. paramesonephric ducts Bartholin's glands function of sertoli cells Potter's sequence common peroneal nerve lesion PCL injury Congenital megacolon several collagen questions Kartagener's syndrome sliding filament theory for muscle contraction Neuro LGN Huntington's disease slide of mammilary bodies (pertaining to chronic alcoholic) extraocular muscles and innervations many, many questions on blood supply as they pertain to stroke presentations (including several brain arteriograms) many, many vision questions (know it cold) several B12 neuropathy questions several brachial plexus questions muscle spindle control Behavioral Very few questions on disorders--mostly stats, epi, and doctor/patient relationship questions prevalence vs. incidence sensitivity and specificity odds ratio statistical distribution and 95% confidence intervals

reliability and validity cohort study clinical trial informed consent child abuse cocaine, heroin, barbiturate overdose normal vs. abnormal grief sleep apnea Alzheimer's schizophrenia narcolepsy major depression bipolar malingering conversion borderline personality Biochem frameshift mutation topoisomerase (what is their function) promoter sequesnces intron vs, exon 3' polyadenylation tRNA charging Northern Blot several pedigrees several Hardy Weinberg questions Pompe's Disease Hunter's syndrome Ehlers-Danlos syndrome Osteogenesis Imperfecta (picture of kid with blue sclerae) carnitine shuttle for fatty acids many, many questions on G-protein linked 2nd messengers (make sure you know all the ANS receptor types and what type of 2nd messenger it is linked to---it was SO MONEY to know that) many insulin/glucagon questions several questions on arachidonic acid pathway kwashiokor vs. marasmus enzyme kinetics CO2 transport in blood (which way does the Hb curve shift) which vitamins are fat soluble vitamin A deficiency (can cause alopecia) osteomalacia

Micro Lipid A induces TNF and IL-1 H. flu requires V and X on choc agar which bugs make H2S which ferment lactose H. pylori which go systemic TB Rocky Mountain spotted fever syphilis (rash includes palms and soles) Mycoplasma (cold agglutinins are IgM!!!) Lyme disease PCP Toxoplasmosis Trichomonas vaginalis Trypanosoma cruzi--at least 2 questions Schistosoma, Ascaris lumbricoides, Enterobius vermicularis JC virus in AIDS patient viral complementation heterophile positive mono many, many hepatitis questions (know it cold) several questions about congenital infections with various bugs (CMV, syphilis, herpes, histo) osteomyelitis pyelonephritis meningitis thrush Immuno Fc portion of Ab--what does it do? allotype several hypersensitivity questions CGD autoantibodies--know them all DiGeorge Bruton's antigenic variation Path Down's assoc. with ALL PDA coarctation of aorta

Klinefelter's (picture of karyotype) CF Von Hippel-Lindau disease Hereditary Spherocytosis Burkitt's HTLV-1 prostate adenocarcinoma basal cell carcinoma many, many anemia question beta thalassemia DIC ITP, TTP achalasia several smoking questions hemochromatosis acute pancreatitis pancreatic adenocarcinoma COPD Diabetes abnormal pulmonary flow volume loops (at least 2 questions) neonatal RDS epidural hematoma (MRI) PML absence seizure Horner's syndrome osteoarthritis rhematoid arthritis celiac sprue many SLE questions Graves disease Sarcoidosis scleroderma Goodpasture's Conn's syndrome pheochromocytoma BPH hydatiform mole many MEN I, II, III questions breast cancer preeclampsia iscehmic heart disease aortic stenosis syphilitic heart disease Takayasu's temporal arteritis polyarteritis nodosa (assoc with Hep B)

many acidosis/alkalosis questions Pharm at least 5 kinetics questions clavulanic acid tetracycline as used for acne gentamicin assoc. with nephrotoxicity trimethoprim for PCP flouroquinolones contraindicated in kids anti TB drugs fluconazole--inhibits steroid hormone synthesis HIV therapy mebendazole ANS was big! bethanechol, cocaine, antimuscarinics, clonidine, B blockers, succinylcholine, phenylyephrine (rebound nasal congestion), epi, norepi, prazosin, glaucoma drugs...just know ANS real well several antipsychotic drugs opioid analgesics carbamazepine--causes aplastic anemia nitrous oxide as anesthetic ACE inhibitors acid/base balance with diuretic use hydrochorothiazide causes hyperglycemia K+ sparing diuretics--spirinolactone can cause gynecomastia quinidine several questions on lipid-lowering agents anti-cancer drugs were big too: 5 fluorouracil, doxorubicin, prednisone, raloxifene (make sure you know all their side effects) Magnesium antacid abuse heparin/warfarin many NSAID questions glucocorticoids propylthiouracil Physiology many heart questions with graphs preload vs afterload resistance, pressure and flow capillary fluid exchange surfactant function many Hb dissociation curve questions somatostatin vagotomy effect on digestion

bilirubin, bile glucose clearance in kidney several PTH/calcitonin/Vitamin D questions ANP many renin-angiotensin questions know hypothalamic/pituitary regulation well many estrogen/progesterone/menstruation/pregnancy questions spermatogenesis

Step 1 Questions I remember: 1. MOA of rifampin (darn question showed up 2 times) 2. MOA of erythromycin 3. MOA of cephlosporins 4. What do u give first to unconcious alcholic who comes in to ER? IV thiame first 5. You have bad news on patients condition and have to tell her, she see statement on your face and says son makes all the decisions, what do u do? Tell her, Tell her and son, Tell son, Tell nurse to tell son, ect... 6. 16 yo girl comes in and has bf and they have protected and unprotected sex, what do u tell her first in regard to contraception? 7. Cystic Fibrosis kid, what chromosome is the defect on? 8. Philadelphia chromosome found in what condition? 9. What do u see on EM with a kid with minimal change disease? 10. Athelete suddenly collapses while playing baseball, what kind of cardiomyopathy? 11. Rate limiting step in glycolysis? 12. Weber - Rinne test of person with cerumen in his right ear what happens? 13. Weber - Rinne test with person who is deaf in R ear what happens? 14. Corneal reflex question with R eye have consenual reflex only and when with L eye, direct only, where is the problem? 15. Cross section of spinal cord asking location that is affected with loss of pain and temp. 16. Cross section of spinal cord asking location that is affected with UMN lesion. 17. CT of brain of person involved in car accident, what kind of hematoma? Subdural, epidural or subrachnoid? 18. Enlargement of lateral ventricles with blood in them, where is the problem? 19. Brachial plexus drawn and sking for where lesion is with Erbs? 20. X-Ray of femur asking location where avasular necrosis occurs? 21. Drawing of chest asking where u hear the mirtal valve sound the best. 22. Pressure volume loop curves asking where mitral valve opens? 23. Diagram with normal heart sound and abnormal one asking what kind of murmur is seen on the abnormal? It was a late diastolic murmur. 24. Side effects of combo drugs of person being treated with Hodgkins? 25. If person has megaloblastic anemia with folate and neuro problems and after Rx u only have neuro problems what did u give patient?

26. Question on Fe deciciency anemia, what values u see with TIBC, seurm Ferritin, etc. 26. Person with diabetes insipidus what lab values do u see? 27. Person with SIADH what lab values do u see? 28. How do u distinguish between an insulimona and person who is taking insulin for diabetes? 29. What cardiovascular problem is seen in a patient with Marfan's syndrome? 30. What cardiovasular problem is seen in patient with teritiary syphilis? 31. What problems with eyes is seen in patient with teritiary syphilis? 32. MC organism seen in hikers who drink water from mountain stream? 33. Know the steps in Gram staining and for acid fast staining. 34. What is vaccine for H. Influ b made from? 35. How do u calculate sensitivity? 36. How do u calculate positive predictive value? 37. How do u calculate attributable risk? 38. What is a cohort study? 39. What is a double blind study? 40. How do u calculate prevalance? 41. In what situation is prevalance greater than incidence? Chronic diseases 42. What happens to GFR and RBF if there is constriction of efferent arterioles? 43. Lineweaver Burk plot asking what happens with competitive inhibitor? 44. Michaelis Menton curve asking what happens with a competitive inbibitor? 45. What kind of mutation happens when there is change from long protien being made to short one? 46. Know about Wobble hyopthesis 47. Where is mRNA made in the cell? 48. MOA of erythromycin? 49. Family members all vomit 5 hours after eating BOILED rice that sat overnight with NO refrigration? B Cerus or ETEC 50. SE of person taking aminoglycosides? 51. How old is child when they don't have stranger anxiety? 52. Characteristics of person suffering from bipolar disorder? 53. Characteristics of person suffering form schizophrenia? 54. Parent bring kid in who was unconcious, he has nystagmus and has been withdrawn from parents, what drug has he been abusing? 55. Person who is taking heroin can have what kind of infection? 56. Organism causing diarrhea in person with AIDS? 57. Organism causing diarrhea in infants who go to daycare?

58. What kind of pattern seen on EM of patient with Goodpasteurs? 59. Picture of vegetations on aortic valves, what kind of problem will they cause? 60. Know difference between R sided heart failure and L sided heart failure 61. Person is bulemic, what is PCO2, HCO3 and pH? 62. If go to high altitude what is compensatory mechanism of respiratiory alkalosis and what are they values u would see PCo2, HCO3, pH. 63. Person with H.Pylori given medication that covers ulcer, what is MOA of that drug? 64. SE of person taking H2 blockers? 65. Best treatment of person who is Black suffering from CHF? 66. Give heparin to person but no change in PT or PTT what is deficient? 67. What changes in persons PT, PTT and Bleeding time with von Willebrands disease? 68. Same as above but now person with hemophilia A? 69. Know steps involved in doing a PCR. 70. Person comes in with a magazine ad or a new drug which you as a doctor know won't help them in their condition and patient asks if you can give them the drug, what is first thing you tell the patient? 71. 13 yr old kid comes in who admits to being sexually abused by stepfather, what is the first step you take? Notify child services or call police or talk to mom or talk to step father

Cases/questions: Female patient is on a whole bunch of drugs including Warfarin, diuretics, antihyperlipedimics, etc. She has recently become pregnant and has consulted the doctor, which of the drugs should be discontinued. Answer: Warfarin (teratogenic for fetus). Which of the abdominal aorta branches supply the foregut Answer: Celiac trunk Patient presents with inflammation and necrosis of small intestine, ascending colon and proximal 2/3 of transverse colon, which blood vessel is involved: Answer: superior mesenteric A. Which branchial arch makes up Mandible and Mylohyoid, etc. Answer: first arch Picture of endochondral bone formation (similar to picture in first aid) and asks which of the following bones are formed by the shown process. Choices were clavicle, skull, femor, etc. Answer: Femur (long bones) Picture of a child being able to bring her shoulders together, which bone is she missing: Answer: clavicle Which cells in testes provide the Blood Testes barrier: Answer: Sertoli cells Knee injury and tear in MCL and ACL Which organ in the body releases most ANG II into the body? Answer: lungs (has ACE) EM of muscle asking to point to Z line Scraping the stratum corneum from the skin, what is the most important complication? Answer: loss of fluid from body and dehydration (since stratum corneum contains Keratin and protects against blood loss) Gap junctions are able to transmit Electrical current between cells

Patient presents with fever and shivering. What is the purpose of shivering? Answer: thermoregulation (it generates heat). Sudden wild flailing of one arm, where is the lesion of the drawing: Answer: subthalamus Patient presents with deviation of uvula and shoulder droop, where is the lesion? Answer: jugular foramen Point to the muscle that depresses the eye from the adducted position Answer: Superior Oblique muscle (same diagram as in page 108 first aid, 2001 edition). Marcus Gunn Pupil Lesion in which blood vessel causes left homonymous hemianopsia (angiography similar to circle of willis pictures in High Yield Neuro) Answer: right post. Cerebral Brown Sequard lesion Dorsal Column pathway and lesions of Gracilis fibers and Cuneatus fibers and how they affect sensation on upper and lower limbs. Radial nerve injury Cervical rib and what it affects Answer: inf. Trunk of brachial plexus Cohort study Heroin addiction and long term maintenance Answer: methadone Delirium (patient not oriented as compared to dementia in which patient is oriented) Conversion disorder Post traumatic stress disorder Personality disorders: Avoidant and Schizotypal

Narcolepsy Borderline patient using splitting as a defence mechanism (either everything is good or everything is bad) Lots of questions on shizophrenia (at least 4) Thought disorder, Loose associations, etc. Hardy Weinberg population genetics calculations Glucose-6-phosphate dehydrogenase deficiency and reaction to sulfa drugs Which a.a. become essential in a patient with PKU with phenylalanine restricted diet: Answer: tyrosine Picture of patient with blue sclera Answer: abnormal collogen synthesis Patient presents with muscle weakness and hypertrophy of calf muscles, which chromosome is defective? Answer: X chromosome (DMD) Uncoupling agents and on which part of mitochondria they work? Anwer: dissipate proton gradient across the inner mitochondrial membrane Urea cycle Arachidonic Acid products (very heavily tested, at least 5 questions) LT B4 (neutrophil chemotactic agent) LT C4, D4, E4 (slow reacting substances of anaphylaxis) TX A2 (vasoconstrictor and stimulates platelet aggregation) PG I2 (inhibits platelet aggregation and vasodilation) Collogen Synthesis and structure Vitamins (heavily tested): B1 (alcoholics)

B2 (cofactor in oxidation and reduction-FADH2) B6 (given along with isonizid to prevent peripheral neuropathy) Vitamin C (hydroxylation of proline and lysine in collogen) Vitamin D (which organ hydroxylates 25 hydroxy D3 at alpha1 position: kidney) Vitamin K (coagulation factors) Which test diffentiates staph aureus from other staph (believe it or not!) Answer: Coagulase test Alpha Hemolytic bacteria H. pylori infection (at least 2 questions) BMT regimen Which of the pairs result in long term neurological complications? Answer: Borrelia and Treponema Mucor infection in diabetics (same picture shown as on page 191 of first aid, 2001 edition) After an earth quake in california, epidemic of lower respiratory infection which lasts for 2-3 weeks, which of the bugs responsible for it? Answer: Coccidiodomycosis Candida vaginalis in diabetics Toxoplasma and brain abcess in HIV Giardia and foul smelling stools Most important sequale in infectious Mono: Anwer: splenic rupture (I think). Patient has C3 deficieny, which disease is he most likely to be susceptible to Answer: only capsular organism listed was pnomococcus 2 questions on Huntington Disease:

trinuclear repeat pattern point to Caudate Nuclei on MRI alpha fetoprotein as a marker for malignancy 11,22 translocation in Ewing sarcoma of the bone

Hereditory Spherocytosis and treatment Answer: splenectomy IgG spike on plasma proteins (shown in a diagram) and asked how the patient presented Answer: bone pain Hirschprungs disease and what causes it Patient presents with cirrhosis after being diagnosed of liver disease many years ago. When he presented at first ALT was elevated, what is the most likely cause of cirrhosis? Answer: Hep C (if AST was higher, it would have been alcoholic cirrhosis). Chronic pancreatitis and its association with alcoholism Pancreatic adenocarcinoma and signs associated with it (eg. Weight loss, obstructive jaundice and palpable gallbladder Diagnosis of emphysema in a non-smoker 25 year old with dyspnea, hyperexpanded lung fields and lowered breath sounds and its association with a1antitrypsin deficiency (panacinar emphysema) Restrictive lung disease and FEV/FVC ratio (80%) Paraneoplastic syndromes associated with small cell carcinoma of the lung and the signs associated with it (eg. Hyponatremia associated with SIADH in small cell carcinoma) Patient presents with tremor and rigidity, upon taking history you find out that patient has been experiencing with a designer drug. What is the most likely region of the CNS thats involved Answer: Dopaminergic neurons of substantia nigra Treatment of Parkinsons Patient presenting with upper and lower motor neuron problems

Answer: ALS Epidural and Subdural hematomas and their diagnosis based on history. Patient presenting with signs of Brocas Aphasia, point to region of the brain that is most likely involved. Diagnosis of Horners Syndrome Patient presents with signs and symptoms of arthritis, morning stiffness and Dup. Deformity of the hands and fingers, what abnormal test result is expected. Answer: Anti-IgG antibodies in the serum (diagnostic of RA) At least 3 questions on gout and drugs used for it (eg. Patient presents with joint pain after chemotherapy for Hodgkins lymphoma, etc.) SLE and kidneys wireloop lesions, False positive on syphilis test (VRDL) and drugs which induce SLE and diagnostic features Patient presents with Sacroiliac joint inflammation, which HLA tests are likely to be positive Answer: choose whichever one had HLA B27 Hyperaldosteronism and its diagnosis based on renin, Na and K values in plasma Distinguishing between primary and secondary adrenal insufficiency Pheochromocytoma and drugs used for it (esp. nonselective irreversible alpha blocker) All the features associated with MEN I and MEN II Hypho and Hyperthyroidism and diagnosis based on symptoms and signs and plasma values of T3, TSH for each Diabetes types I and II (at least 5 questions)

Ketoacidosis associated with type I Symptoms associated with diabetic coma Renal complications (arteriosclerosis and glomerular lesions on light microscope Lab values associated with diabetic ketoacidosis

Carcinoid tumor diagnosis based on symptoms and what abnormal test is positive in urine Answer High HIAA in urine

Osteoprosis and its association with corticosteroids BPH and drugs used for it Hyadid mole and increased HCG associated with it early in pregnancy Endometrial hyperplasia and incrased risk of endometrial carcinoma with abnormal uterine bleeding An obese female presents with high LH and hirsutism. Dx? Answer: Polycystic Ovarian Syndrome Cardiomyopathies esp. Hypertrophic cardiomyopathy: hypertrophy involves the interventricular septum and 50% is caused by mutation in B globin gene of heavy chain (familial). Usually seen in athletes. Heart murmurs and their diagnosis based on features of the murmur DVT and pulmonary embolism. Bone marrow embolism seen as a complication of bone fractures. May lead to sudden death. Wegeners granulomatosis and its Dx based on symptoms (C-ANCA was not given). WG involves the upper airways and youll see granulomas (this is the distinguishing feature from Goodpastures syndrome Minimal change disease effacement of BM Acid Base physiology Pharmacodynamics graphs distinguishing Agonist with competitive antagonist and irreversible antagonist Mechanism of Tetracycline Nephrotoxicity associated with Aminoglycosides and also ototoxicity. Isoniazid and B6 which can prevent neurotoxicity HIV triple therapy (two nucleotide reverse transcriptase inhibitors with a protease inhibitor. Mechanism of Dantrolene Isoprotrenol effect and reflex tachycardia associated with it Mechanisms of action of sympathetic receptors, Dopamine receptors (eg. cAMP, IP3, etc.)

Antipsychotics and Tardive dyskinesia Lithium and its effect on thyroid (may cause hypothyroidism) Sumatriptan and its mechanism of action ACE inhibitors and its effect on blood pressure and its mechanism of action Cardiac glycosides and its effect on intracellular and extracellular Na and K Busulfan associated with pulmonary fibrosis Cisplastin associated with Nephrotoxicity Lead poisoning and its features (eg. lead lines) and treatment H2 Blockers and their mode of action and their effects on gastrin and stomach acid Arachidonic acid products and their effects on vascular tone and platelet action Leuprolide and its indications (also mechanism of action). Pacemaker action potential and its differences with ventricular action potential (eg. upstroke is opening of Ca channels rather than Na channels). Calculation of cardiac output with Fick principle Cardiac cycle graphs Mechanism of smooth muscle contraction and its differences with skeletal muscle (eg. involvement of calmodulin) Capillary fluid exchange and calculations involving hydrostastic and oncotic pressures, etc. Glucose clearance and at plasma glucose of 200 glucosuria begins PTH and Vit D effects on plasma Ca and phosphate. Steroid hormone mechanisms (binds to receptor located in nucleus or in cytoplasm) Distinguishing 11B hydroxylase and 21B hydroxylase Prolactin regulation and Dopamine Oxygen dissociation curve and which factors effect it (eg. pH) Secretin and stimulation of release of antacid

Bile and how its recycled (absorption via ileum and how it causes malabsorption of fat in patients with ileal resection). Here are the q's I recall: 1. know your mediators (IL-2, IL-456, IL 10, TNf,TGF, IFN) 2. know cd 16 and 56 are killer cells 3. where both clotting pathways meet 4. acute MI what the gross and histo look like at all stages, 5. brachial plexus- I had ulnar nerve b/c interossei weren't working 6. know 2nd messengers-oxtyocin, alpha 1 7. herpes pneumonitis, with giant cells 8. splenic infarct, from where? 9. cardiac tamponade 10. go over jamie's sheets- 10 icm type q's 11. know the arachadonic acid pathway and enzymes and inhibitors 12. t(9;22) cml 13. lat rectus on mri 14. wilsons- they showed the kleiser fleicher ring 15. cystic hygroma- turners syn 16. mitochondrial inheritance -2 q's 17. know everything about digoxin 4 q's 18. pku 19. b-cell def- no germinal centers in LN 20. know the cyto p450 inducers and inhibitors - 6 q's 21. scleroderma- ab against? 22. I had no sle 23. bph and prostate cancer- drugrs, recepters, 3q's 24. ebv in Aids cancer- lymphoma 25. know your endocrine physio- hormones up and down 26. know the resp. curves in first aid, where would fibrosis be? 27. know histo of the hepatocyte- fxn of organelleI had mitochiondria 28. know histo of kidney- pct, dct, loh- where glucose is reabsordeb, where vit d is activated 29. restrictive cardiomyopathy 30. p. carinii -know what it looks like 31. marasmus 32. carcinogens in bladder cancer 33. schistosomes in bladder cancer

34. ASCVD-2 q's 35. prinzmetal angina- drugs, recepters, -2 q's 36. endocarditis-viral, bacterial 37. India ink-crypto 38. ida, anemia of chronic dz 39. vwb dz- whats up? 40. henoch-scholheim purpura on the leg-vasculitis 41. pathogenesis of TB 42. didn't have a lot of behavioral but what I did have was alzheimers, and a few "what do u tell the pt" q's 43. hand foot mouth dz- coxsackie 44. cherry red macula and European-hexoaminidase a2 45. gauchers 46. tuberous sclerosis- 2 q's (when did we learn this)- know it is assoc. w/ embryonal rhabdomyosarcoma and calcifications in brain 47. cancer met. to colon 48. crohns dz 49. hirshprungs dz- failure of neural crest cells 50. know your gi physio- 5 q's 51. moa of omeprazole 52. moa of vancomycin 53. moa of neostigmine 54. causes of malig. Hyperthermia 55. pathogenesis of spider angioma 56. malig. Htn- pic 57. min. change dz- vignette was classic 58. pid-chlamydia 59. carnitine def-s/s 60. transformation- dnase 61. fracture to fibula- loss of deep peronial nerveloss of dorsiflexion2q's 62. damage to base of penis-blodd flow is lost in which part of urethra (weird q) 63. hypertinic bladder-whereis lesion? 64. where does seminoma drain? 65. know your fractures- what muscles pull them where- 3 q's 66. fracture to lt. 9-11 ribs- spleen 67. cavernous sinus thrombosis- fxn's lost 68. av nicking , banking- learnit well ,2q's 69. myasthenia gravis- 2 q's 70. hashimotos= 2 q's 71. dermatomes

72. 6-8-10, 8-10-12, lung -pleura relationship 73. rt. Atrial hypertrophy on cxr 74. paradoxical emboli 75. brca met. via axillary nodes 76. soft tissue sarcoma 77. hemoglobin c 78. hla b27- reiters syn 79. proteus- uti stones 80. pathogenesis of gall stone-supersat. Of cholesterol 81. acute panreatitis vignette-inc. amylase 82. nodular glomerulosclerosis in D.M. 83. know your vaccines, and active and passive immunizations 84. cohort study, case series 85. I had 5 q's on epidem. (ppv, spec, sens) 86. p=id 87. endometriosis 88. 2 bitemp. Hemianopsia's 89. know your goiters- toxic, hot, cold, non toxic, multitoxic 90. brached hyphi- dermatophyte 91. hyper -PTHism 92. cushings syn 93. addisons dz 94. duchenne's musc. Dystrophy- pedigree 95. osteo sarcoma 96. comparing osteomyleitis and osteo chondritisweird q 97. pseudo gout 98. neuro syph- arguile robinson pupil 99. vitamins- b12, pyridoxine, and vit c 100. desmopressin in diabetes insipidus 101. know your murmers. How thy look normally and on a wiggers 102. I had no ekg-s 103. salmonella 104. mhc 1- tap ptotein 105. cavernous hemangioma, is it bad? 106. lot of angio tension -renin stuff- 4 q's 107. ipratropium in copd 108. colchicimein acute gout 109. filtration fraction up and down..why? 110. know your acid-base stuff- weird q's with combined.alcoholic that vomits and had diarrhea (what the !@#$#$?) know the values 111. know your lesions in the CNS- whats ipsi whats

contra 112. little biochem on the exam-know the rls's 1. I had two questions r/t Baye's Theorum (Genetics) 2. Remember how I said ANS &CNS Pharmacology are Key!! 3. Gliaden and wheat causing Celiac Sprue 4. They showed a picture of a Penis with a lot of Painful nodules. They did not have H. ducreyi, T. pallidum, Herpes, as the choices!!! By the way, this was the first question I saw after I got back from Lunch. 5. Conditions favoring Metabolic Acidosis/Alkalosis 6. Slide of Hairy Cell Leukemia. 7. I had five Questions on Lyme Disease (Go Watsons!) A word of Caution, Lyme Disease does happen in Oregon. 8. What Causes Trendelenberg Sign and Waddling Gait 9. Know all about Heparin and Warfarin 10. I had a question on Croup and Slapped Cheek Disease (Fifth Disease) 11. Lots of Vitamin Questions, so know the roles of Vitamin A,C,K,B12,B6, Thiamine. 12. For the behavioral questions, they would ask you what the next step is. Pick the least invasive and the answer that gives the most degree of freedom. What I mean is that if the competent patient refuses to comply with the doctor, that is his right. 13. For those that have the plagarized MSA Notes (Red ICM Book), it would be a good idea to go over them before the exam. I had a few respiratory questions that could not have been answered had I not red the chart in the book. 14. Role of Gap Junctions 15. Nephritic and Nephrotic Syndromes 16. I only had one SLE Question 17. McArdles Disease, Tay Sachs, Gaucher's Disease were questions. ANATOMY -mesonephric ducts=male spermatochord -inguinal fascia and saddle injury (kids starts to pee, where will urine go inside body cavity, NOT down thigh) -diaphragmatic hernia -GI blood supply (celiac trunk=forgeut) -ureters and obstructions -macula adherens -ciliar dynein (Kartagener's syn) -picture of lesion in Broca's area -spianl cord lesions -braichial plexus (of course, mine was axillary n.) BEHAVIORAL -lots of "how would you respond as the physician" questions, WITH lengthy clinical descriptions. read the actual question first, it will allow you to skip a lot of the data they

give -graph asking how you would maximize sensitivity -odds ratio -types of studies (cohort, case-control) -speech development (what the hell?) -developmental milestones -narcolepsy -sleep apena (central vs. obstructive) X2 BIOCHEM -DNA mutations -where in cell introns are removed -tRNA attachemnt to ribosomes -modes of inheritence (I actually had one ask me the chance of a MAN passing an Xlinked recessive gene to a son, read CAREFULLY) -urea cycle -carnitine -glucokinase vs. hexokinase kinetics -enzyme kinetics -collagen structure and the role of lysyl oxidase MICRO -staph aures food poisoning -fungal questions X3 -pork tape worm -normal flora for nose -infant pnuemonia -hypersensitivity (for contact dermatitis) -Rhogam is PASSIVE immunity PATH -congenital heart defects -fragile X dz -risk of translocation with maternal age -Duchenne's MD is xlinked -CF: psuedomaonas infections and autsomal recessive -marfans and dissecting aortic anuerysms -neural tube defects and alpha FP -melanoma (depth of lesion is worst sign) -epidural hematoma (they called it extradural) -anemias -bleeding disorders -multiple myeloma -obstructive lung dz -ARDS -arthritis -SLE -ankylosing spondylitis

-osteoporosis and compression fractures -hydatidiform mole is 46 XX -breast dz (what a blocked lymph duct would look like??) -Coxsackie B (dilated cardiomyopathy) -heart murmurs -temporal ateritis (tx: prednisone) -acid/base graphs -Xray of pnuemothorax PHARM -maintanence dose -first order elim. -effect of antagonist in general -tetracycline mechanism (MORE than knowing 30S) -aminoglycosides=ototoxicity -INH mechanism -cocaine additction mech. -PAM -Dantrolene mech. (Ca2+ from SR) (yes, got it right Kelly) -Beta blockers -Alcohol and Benzodiazipenes -Sumatriptan mech -Diuretics X5-6 -Quinidein efx X3 -Bleomycin side efx (cancer bear will be your friend) -warfarin mech -misoprostol (w/NSAIDs) -H2 blockers -Arachidonic acid pathway X3 -diabetic drugs (increasing insulin receptor sens) -prednisone PHYS (not that many calculations, few were there weren't too bad) -cardic output -cariac loop cycle (then inferring which one had largest CO) -control of BP -filtration fraction -Glc. clearance -kidney hormones -PTH -uses of estrogen -menopause profile -O2 diss. curve -secretion and bicarb -calculate calories (from CHO, prot. and fat)

Step 1 Questions 2004 1- 57 y/o male is on hemodialysis for CRF. Which of the ffg metabolic derangements can be anticipated? ABCDEhypercalcemia hypophosphatemia osteomalacia Vit. D excess Hypoparathyroidism

Answer is c- osteomalacia. Kidneys failure to excrete phosphate hyperphosphatemia & fail to synthesize 1,25(OH)2D3. then Vit. D def. impaired intestinal ca absorption. Phosphate retention, defective intestinal absorption & skeletal resistance to PTH hypocalcemia secondary hyperparathyroidism worsens hyperphosphatemia by increasing the Ph. release from bone renal osteodystrophy

2- 50 y/o woman with a Hx of essential HTN, presents to ER with severe headache, nausea, vomiting and photophobia. On P/E, BP= 160/100 mmHg, shes confused with nuchal rigidity without focal neurologic signs. ** Whats the Dx? A- hemorrhagic stroke B- ruptured cerebral aneurysm C- meningitis D- ischemic cerebrovascular accident E- TIA

** Dx is best confirmed with? A- LP B- Doppler US of carotid artries C- Head MRI D- Head CT followed by cerebral angiography E- EEG

** Whats the next most important step in Mx? A- urgent surgery with aneurysm clipping B- admission to ICU, close monitoring and Tx of HTN C- admission to ICU, close monitoring and IV antibiotics D- anticoagulation & antiplatlete therapy E- serial LP to drain CSF

Answers are B, D, A most likley the pt. ruptured cerebral anurysm, as pt. is confused, no focal signs present and pt. has nuchal rigidity, could be and most likely due to the blood in the CSF--> meningeal irritation, etc.. although you also see nuchal rigidity w/ meningitis, here you have confusion as well although they give us a nice high B.P of 160/100, and we might want to choose stroke over cerebral anurysm, given the rest of the prez. here anurysm is more likly, as well as you would expect to see neuro focal findings w/ stroke, not present here ischemic cerebrovascular accident --> expect to see some focal signs as well as may not have nuchal rigidity TIA, usually transient and other symptoms present , not seen here

3- During an elective laparascopic cholecystectomy, the patient developes a sudden onset drop in BP, atrial desaturation and an increase in ventilatory pressure. Whats the most appropriate step of Mx? A- an IV fluid bolous B- decompression of the pneumoperitoneum C- inserting a chest tube D- re-evaluating the position of the endotracheal tube & portable CXR E- aborting the procedure & converting to an open cholecystectomy Answer is C, coz patient has developed tension pneumothorax.

4- 2 wk old boy is brought for check up. Hes been doing well at home. Results of his newborn screen shows he has sickle cell dis. Whats the most important step in his Mx? A- avoiding heat exposure B- immunization with pneumococcal vaccine C- folic acid supplements D- iron supplements E- prophylactic penicillin

** Six months later, the same patient returns with a 3 day Hx of lethargy & fever with rhinorrhea and cough. On P/E, he is pale, tacchycardic with a LUQ mass. Hgb= 4, Plt= 100000, WBC= 15000 with 50% segmented neut. , reticulocyte count=15%. Whats the Dx?

ABCDE-

Acute chest syn. Acute splenic sequestration Aplastic crisis Intussusception Vaso-occlusive crisis

Answer is E & B. because hes at high risk of infection, especially with encapsulated organisms and penicillin will dramatically decrease this risk. Pneumococcal vaccine is not effective in neonates. Its usually given at 2 yrs age.

5- A 52 y/o man receives a preoperative evaluation before an alective surgery. He is asymptomatic with a normal P/E, but he is noted to have a Hgb= 10.8, Hct= 33, MCV= 70, RBC= 6.1x1000/ microlit. Whats the most likely Dx? A- Sickle cell dis. B- Iron def. Anemia C- Alpha thalassemia major D- Beta thalassemia minor E- Anemia of chronic dis. Answer is D. low MCV is either IDA or beta-thal. BUT, high no. of RBCs is characteristic of beta-thal. In IDA, RBC count is below normal. pt. has hypocromic, microcytic anemia,..as well as increased retic. count--> increased RBC's, ..this last feature makes the better choice here to be thalassemia, as oposed to iron def. for example iron def. seems to be avery good choice, as one should think anemia in older MAN , is due to cancer-COLON ca , until proven otherwise..however you would expect to see low reticulocytes count w/ iron def. anemia. note- elevated retic. counts--> seen in thalassemia/hemoglobinopathy,..like SS anemia

6- match the clinical description with the most likely organism: abcdefghStrep. Pneumoniae Staph. Aureus Strep. Viridans Providentia stuartii Actinomyces israelii Hemophilus ducreyi Neisseria meningitides Listeria monocytogene

1- 30 y/o female with MVP, MR develops fever, anorexia & weight loss after a dental procedure 2- 80 y/o male hospitalized for hip Fx, has foley cath. in place, develops shaking chills, fever & hypotension. 3- young man develops painless, fluctuant, purplish lesion over mandible, after several weeks cutaneous fistula is noted. 4- sickle cell patient presents with high fever, toxicity signs of pneumonia & stiff neck.

Answers: 1- c, 2- d, 3- e, 4-a

7- 65 y/o male with Hx of DM & cardiomyopathy, presents with severe knee pain. On P/E, knee is swollen, red & tender. Knee X-Ray shows linear clcification. *** Dx is best made by: A- Serum uric acid B- Serum calcium C- Arthrocentesis & identification of birefringent rhomboid crystals D- Rheumatoid factor *** Further workup inthis patient should include evaluation for: A- Renal dis. B- Hemochromatosis C- PUD D- Lyme dis. Answers are C & B. Acute monoarticular arthritis in association with linear calcification in the cartilage of knee maqkes the Dx of pseudogout which is positive for birefringent crystals in joint fluid. Pseudogout maybe associated with hemochromatosis.

8- 65 y/o woman with a 12 Hx of symmetric polyarthritis, presents with splenomegaly, ulcers on lat. Malleoli, synovitis of wrists, shoulders and knees, and no hepatomegaly. Lab results : WBC=2500, RF= 1:4096, this patients WBC diff is most likely to show what ? A- pancytopenia B- lymphopenia

C- granulocytopenia D- lymphocytosis E- basophilia Answer is C. case of feltys syn. ( RA+ splenomegaly+ leukopenia ), the mech. Of granulocytopenia is poorly understood.

9- A patient with low grade fever & weight loss has poor excursion on the Rt. side of chest with decreased fremitus, flatness to percussion and decreased breath sounds. Trachea is deviated to the Lt. Whats the most likely Dx? A- Pneumothorax B- Pleural effusion secondary to histoplasmosis C- Consolidated pneumonia D- Atelectasis Answer is B. Physical findings all consistent with pleural effusion, which in large amount can shift trachea to the Lt. in pneumothorax hyperresonance of the affected side is present. Atelectasis on the Rt would shift trachea to the Rt.

10- A patient have an unexpected high value for diffusing capacity, this finding is most consistent with which of the ffg? A- Anemia B- Cystic fibrosis C- Emphysema D- Intrapulmonary hemorrhage Answer is D. Decreased diffusing capacity is seen in: primary parenchymal disorder, anemia & removal of lung tissue. Increased value is seen in : polycythemia, CHF & intrapulmonary hemorrhage.

11- A 65 y/o man is admitted to the hospital with anginal chest pain. His general health has been excellent, although he has had a multinodular goiter for many years. He had a series of thyroid function tests 4 weeks before admission, and the results are :
Serum T4 8.0 g/dL Free T4 index 8.0 Serum T3 152 ng/dL Serum thyrotropin (TSH) 0.7 U/mL

A MI is ruled out, but chest pain continues. A coronary arteriogram shows a 90% stenosis of the left main coronary artery. A coronary artery bypass graft is done. The patient has an uneventful postoperative course and is discharged on the seventh postoperative day. One month later, he is readmitted in atrial fibrillation with a rapid ventricular response. Repeat thyroid function testing shows the following:
Serum T4 15.0 g/dL Free T4 index 15.8 Serum T3 220 ng/dL Serum TSH <0.01 mU/L

What is the most likely Dx? A. Graves' disease B. Stress-induced hyperthyroidism C. Iodine-induced hyperthyroidism D. Silent thyroiditis Answer is C. The natural history of multinodular goiters is slow growth and gradual decrease in thyrotropin (TSH), reflecting increasing thyroid hormone production. This progression occurs over years to decades, however. Many patients with multinodular goiters have autonomous areas within their thyroid. This patient had normal thyroid function 1 month before admission. However, his serum TSH level was near the lower limits of normal, suggesting the possibility of autonomous thyroid function. When patients with multinodular goiters are exposed to excess iodine, severe hyperthyroidism may occur. This is known as iodine-induced hyperthyroidism or the JodBasedow phenomenon. When iodine supplementation is introduced into areas of iodine deficiency, iodine-induced hyperthyroidism may occur in patients with multinodular goiters. Iodine-induced hyperthyroidism may occur in nonendemic goiter areas as well, often with devastating consequences. The high iodine content of the dye used for the cardiac catheterization undoubtedly precipitated the hyperthyroidism in this patient. The onset of hyperthyroidism may be delayed for several weeks to months after the iodide exposure. Although other causes of hyperthyroidism are possible, none is as likely as this scenario. When patients with multinodular goiter must be exposed to excess iodine (for example, during cardiac catheterization, computed tomographic [CT] scan with contrast medium, or amiodarone therapy), premedication with antithyroid drugs (methimazole or propylthiouracil) should be considered.

12- A 43 y/o woman complains of itching that keeps her awake at night. Physical examination is normal, except for the liver, which is felt 7 cm below the right costal margin.The blood count is normal; the results of serum chemistry tests are as follows:
Creatinine 0.8 mg/dL Bilirubin 0.6 mg/dL Alanine aminotransferase 78 U/L Albumin 4.2 g/dL Alkaline phosphatase 450 U/L

Which test would you order next in order to diagnose the underlying disorder? (A) Serum protein electrophoresis (B) Antismooth-muscle antibody (C) Antimitochondrial antibody (D) Technetium-99m liver-spleen scan (E) ERCP

Answer is C. This is the classic description of primary biliary cirrhosis. Itching is the most common specific symptom of early primary biliary cirrhosis. Approximately 70% of affected patients have enlarged livers. The best screening test for suspected primary biliary cirrhosis is the antimitochondrial antibody test. It is positive in 95% of affected patients and has a 98% specificity if newer enzyme-linked immunosorbent assay (ELISA) tests are used. Serum protein electrophoresis might show a diffuse increase in immunoglobulins. However, this finding is nonspecific and may be found in many chronic liver diseases. Antismooth-muscle antibody tests are positive in some patients with autoimmune chronic hepatitis. However, the test is nonspecific and not terribly useful. The sulfur colloid technetium liver-spleen scan is useful for detecting portal hypertension and hypersplenism. However, it is not specific and would not be helpful in diagnosing primary biliary cirrhosis. Endoscopic retrograde cholangiopancreatography (ERCP) is typically normal in patients with primary biliary cirrhosis. Its only role in the diagnosis of primary biliary cirrhosis is in the patient who presents with a similar syndrome but who has a negative antimitochondrial antibody test. ERCP would then be performed to look for other causes of disease such as primary sclerosing cholangitis.

13- A patient presents to your office with complaints of severe earache and decreased hearing on the left side. Examination confirms left otitis media. You prescribe an oral cephalosporin and an analgesic. The next day the patient is brought to the emergency room with a rash and moderate bronchospasm. Reviewing his office record you are horrified to discover that he is known to have a severe allergic reaction to penicillin.

What should you tell the patient at this point? (A) Apologize and explain what happened. (B) Apologize and offer to compensate him financially. (C) Attribute the rash and the bronchospasm to his infection. (D) Explain to the patient that he is allergic to cephalosporin as well as penicillin. (E) No explanations are necessary. Answer is A. Medical errors are common but are infrequently reported to patients. The fear is that acknowledging a mistake may lead to litigation. Nevertheless, there is no excuse for a physician not to admit a mistake, which is an inevitable part of medical practice. Apologizing and explaining is not only ethical, but also has been shown to decrease the risk of litigation. There is no need or obligation to offer financial compensation. Trying to attribute your mistake to other causes, or attempting to blame the patient for your negligence is not only unethical, but will increase your risk of litigation. Finally, nothing will anger a patient more than no explanation at all.

14- A 29 y/o woman presents with an exacerbation of her asthma. She is 11 weeks pregnant. She has mild intermittent asthma and usually takes a -agonist as needed. She has one 4-year-old child who is in day care and has had a recent upper respiratory tract infection. She has a dry cough, clear nasal discharge, myalgias, and fatigue. On physical examination, she is talking in full sentences and has normal tympanic membranes, mildly erythematous oropharynx without exudates, no adenopathy; she has positive wheezing bilaterally. The peak flow is 300 mL; her usual result is 390 mL. Pulse oximetry is 93% on room air.Which of the following is indicated in the management of this patient? (A) Amoxicillin (B) Theophylline (C) Prednisone (D) Montelukast (E) Flunisolide Answer is E. Treatment principles for asthma in the nonpregnant patient apply also to the pregnant patient. She has mild intermittent asthma with an exacerbation, for which therapy with an inhaled glucocorticoid is an appropriate choice. Use of inhaled glucocorticoids is safe in pregnancy. Asthma in pregnancy has a variable course, with one third of patients getting better, one third getting worse, and one third staying the same. This patient has had mild symptoms and now has an exacerbation related either to an upper respiratory tract infection, or to

worsening of her asthma because of her pregnancy. She has no signs of streptococcal pharyngitis, sinusitis, bacterial tracheobronchitis, or pneumonia; antibiotics should therefore not be given. Prednisone therapy should be reserved for further symptoms. Theophylline can be used in pregnancy with close monitoring, but it is not the next drug of choice. Montelukast may be used in pregnancy, but should be reserved for progressive symptoms. The patient should continue therapy with -agonists. Use of a peak flow meter at home is integral to her management; she should be instructed and encouraged in its use.

15- A 30-year-old woman presents because of 6 months of amenorrhea. She had regular menses starting at age 13 years and is otherwise healthy. Menses gradually became irregular over the past 5 years, in spite of relatively stable weight and activity. She has no acne or hirsutism. The results of recent laboratory tests included normal serum thyroidstimulating hormone and prolactin levels and a negative serum pregnancy test. What is the next most useful diagnostic test for the evaluation of secondary amenorrhea in this patient? (A) Measurement of LH (B) Measurement of FSH (C) Pelvic US (D) Measurement of serum estradiol (E) MRI of the pituitary gland Answer is B. In this patient, the negative pregnancy test, and normal serum TSH and prolactin levels exclude the most common causes of secondary amenorrhea. The remaining causes include hypothalamic amenorrhea and ovarian failure. Of the tests listed, serum FSH is the most likely to be diagnostic of a reproductive endocrine abnormality, as it has a longer half life than luteinizing hormone and becomes elevated relatively early in the process of ovarian aging. Luteinizing hormone is less useful because of its greater variability across the normal menstrual cycle (including the dramatic, often 10-fold, increase at the mid-cycle surge), its more striking 1 to 2 hourly pulsatility, and its elevation in polycystic ovary syndrome as well as ovarian failure. Serum estradiol is frequently in the normal range in various reproductive disorders and therefore rarely useful. MRI of the pituitary gland is not indicated unless other tests are normal and a hypothalamic cause of amenorrhea needs to be ruled out. Finally, pelvic ultrasound can show the presence of ovaries, their size, and the uterine morphology and endometrial thickness, but cannot diagnose the cause of secondary amenorrhea. It may be more useful in the evaluation of primary amenorrhea, if the physical examination is inadequate to confirm the presence of intact ovaries and a uterus, or in the evaluation of excessive menstrual bleeding, when it can identify endometrial thickening, polyps, and/or

uterine leiomyomas. A medroxyprogesterone challenge test can show that the uterine anatomy and vaginal outflow track are normal, which is suggested by the history of previously normal menstrual cycles. In addition, the challenge test provides some information about recent estrogen exposure. However, it cannot be diagnostic in estrogen deficiency.

16- A 2 y/o has a chronic cough. CXR reveals hyperinflation of the left hemithorax. Bilateral decubitus views showed that the right lung becomes appropriately atelectatic however, the left side shows no change in appearance with decubitus positioning. Whats the most likely diagnosis? Asymmetric hyperinflation suggests the possibility of an aspirated foreign body in this two year old infant. These findings suggest the diagnosis of an aspirated foreign body or a possible obstruction of the left main stem bronchus from a central etiology. Bronchoscopy is both diagnostic and therapeutic.

17- match this drugs with their associated syndrome: ABCDEFbarbiturates Ecstasy Inhalants Marijuana Methamphetamine PCP

123456-

Severe encephalopathy Lung cancer Rhabdomyolysis during intoxication Wanting to touch/be touched during intoxication Seizures during withdrawal Swiss cheese appearance on functional brain imaging

A-5, B-4, C-1, D- 2, E-6, F-3

18- 72 y/o man with Hx of UTI & CHF is admitted for sepsis & pulmonary edema. Hes treated with clindamycin, tobramycin & IV furosemide. After 4 days, sepsis signs are improves but BUN= 60 & Cr.= 5 mg/dl. BP=125/75 , PR=72( no postural changes ). 1-Whats the most likely cause of his renal dysfunction? ABCDPrerenal azotemia ATN Interstitial nephritis Hypercalcemic nephropathy

2- Whats the best way to confirm the Dx? ABCDUrine Na of 25 mEq/L Renal tubular epithelial cells & muddy brown casts in sediment Negative US Abnormalities of medulla in IVP

Aswers are B & B. Theres no clinical evidence of prerenal azotemia, so the most likely Dx is ATN due to toxicity with aminoglycoside. Urine sediment in ATN is abnormal and shows renal tubular epithelial cells, debris & muddy brown casts. Since patient has been getting diuretics, high urine Na is less specific.

19- 70 y/o man with Hx of COPD, presents with worsening SOB of the last couple days. Hes coughing yellow-colored sputum and gets no result from his beta 2 agonist & ipratropium aerosolized pumps. On P/E, RR=40, HR=110, BP= 155/85, hes afebrile and using his accessory muscles for respiration. Also inspiratory & expiratory diffuse wheezing on both sides of his lungs are heard. Whats the most likely Dx? ABCDEAcute exacerbation of COPD Alfa-1 antitrypsin def. Chronic bronchitis Exacerbation of asthma Pneumonia

Answer is A- Acute exacerbation of COPD, it occurs when patient develops acute onset of marked dyspnea & tachypnea with use of accessory mucles with no response to medication.

20- 73 y/o male with Hx of HTN, presents with short episode of Lt. sided weakness & slurred speech. Also he has a Hx of 3 brief episodes of sudden Rt. eye vision impairment in the last month. 1- Whats the best next diagnostic test? ABCDECerebral MRI Holter monitoring Visual evoked responses Carotid artry doppler US Conventional cerebral angiography

2- Episodes of visual loss are related to: ABCDERetinal vein thrombosis Central retinal A. ischemia Post. cerebral A. ischemia Middle cerebral A. ischemia Post. ciliary A. ischemia

Answers are D & B. This is a classicd case of extracranial internal carotid A. dis. which include episodes of ipsilat. transient monocular blindness ( amaurosis fugax ) & contralat. TIA consisting of motor weekness. The most appropriate test to confirm the Dx of carotid stenosis is Doppler us. The mech. of transient monocular blindness is embolism to the central retinal A. or one of its branches.

21- A 5 y/o presents to the ER 3 h after a possible button battery ingestion. The patient is in no acute distress, vital signs are stable, and examination is benign. CXRshows what appears to be a small button battery in the stomach. Which of the following is the MOST appropriate next action? (A) Upper GI series to further delineate the exact location of the foreign body (B) Attempt battery removal by the Foley balloon catheter technique (C) Immediate GI consultation for endoscopic removal (D) Immediate surgical consultation (E) Discharge to home with parental observation and weekly radiographs The answer is E. Button batteries lodged in the esophagus require emergencyremoval to avoid esophageal burns and perforation. If the button battery has passed the esophagus and the patient is asymptomatic, home observation with serial x-rays toensure passage through the pylorus is the appropriate course of action. Most button batteriesthat have passed the esophagus will transit through the entire body within 24 to 48 h without difficulty. If the battery is of large diameter and the patient is younger than 6

years, the battery is less likely to pass, and endoscopic retrieval is the preferred treatment.

22- A 45 yo male with a long Hx of alcohol use and presumptive gastritis presents to the with sudden onset of severe abdominal pain and vomiting. V/S: BP=110/60, HR=110/min , T=101F, RR= 30/minhes diaphoretic with epigastric tenderness and mild guarding. Lab data: WBC= 30,000, amylase =2,000. CXR shows a small amount of free air under the diaphragm. What is the MOST likely diagnosis? (A) Acute pancreatitis with associated Mallory- Weiss syndrome (B) Acute pancreatitis with associated Boerhaaves syndrome (C) Acute pancreatitis with associated enzymatic destruction of bowel wall (D) Acute pancreatitis secondary to anterior duodenal ulcer perforation (E) Acute pancreatitis secondary to posterior duodenal ulcer perforation answer is E. Because the pancreas adheres to the posterior duodenum, ruptured posterior duodenal ulcers generally penetrate into the pancreas rather than perforate into the free peritoneum. Anterior ulcers are more likely to perforate into the peritoneal cavity. A Mallory-Weiss tear of the esophageal wall usually presents with symptoms similar to reflux esophagitis and causes moderate, self-limited bleeding. Patients with Boerhaaves syndrome rapidly deteriorate to a state of shock and septicemia due to a malignant mediastinitis.

23- A 55 y/o male without significant medical history presents with LLQ pain and constipation. V/S: T= 100.5F, PR= 85, BP=150/80, RR=12. P/E is unremarkable except for mild LLQ tenderness without guarding and rebound. Rectal examination shows hemenegative stool and no tenderness.Lab results are within normal limits except for WBC=13,000 with a left shift. Which of the following would be the MOST appropriate management for this patient? (A) Prompt surgical evaluation in the ED (B) Emergent upper GI series (C) Emergent barium enema (D) Discharge to home with bowel rest and oral antibiotics (E) Discharge home with repeat abdominal examination in 12 h or sooner if worse The answer is D. The patient described in the scenario most likely has diverticulitis. Patients with localized pain and no signs and symptoms of peritonitis or systemic infection do not require hospitalization. Outpatient management consists of bowel rest, broad-spectrum oral antibiotic therapy, and close follow-up.

24- Which of the following etiologic agents is the MOST common cause of infection in liver transplant patients? (A) Candida (B) Cytomegalovirus (C) Herpes simplex virus

(D) P. carinii (E) L. monocytogenes The answer is B. Complications of infections in liver transplant patients account for nearly 90 percent of deaths. The most common infectious agent after transplantation is Cytomegalovirus (CMV. Reported occurrence ranges from 23 to 85 percent of all liver transplant patients. Fortunately, CMV is rarely fatal.

25- A 56 y/o heavy alcohol user male presents with vomiting blood for several hours. BP=90/60, PR=110, RR=16, and T=98F. Placement of an NGT shows active bright red bleeding. Which of the following is the treatment of choice? (A) Tamponade with a Sengstaken-Blakemore tube (B) Therapeutic upper GI endoscopy (C) Octreotide infusion (D) Vasopressin infusion (E) Immediate referral for surgical intervention The answer is B. GI bleeding is a common problem seen in the ED and is potentially life threatening. For patients with significant active bleeding, emergency endoscopy is the treatment of choice. Esophageal varices can be treated endoscopically with either band ligation or sclerotherapy, resulting in control of acute bleeding in up to 90 percent of patients. Hemostasis can be achieved with nonvariceal sources of bleeding as well. Drug therapy with both octreotide and somatostatin reduces bleeding from both varices and PUD and is a useful adjunct to endoscopy. Vasopressin therapy has largely been discontinued due to a high rate of adverse effects including hypertension, arrhythmias, myocardial ischemia, and decreased cardiac output. Balloon tamponade can be used as a temporizing measure to control bleeding by placing direct pressure on the gastric and esophageal mucosa. However, it is frequently associated with complications, including mucosal ulceration, esophageal or gastric rupture, asphyxiation from dislodged balloons, and aspiration pneumonia. Although it is appropriate to make a surgeon aware of a critical patient, emergency surgical intervention is indicated only in those patients who fail endoscopic hemostasis and medical therapy.

*** 36 y/o woman with meno/metrorrhagia, inlarged uterus, which of the ffg would tell the severity of this condition? a- CBC b- Hysteroscopy c- US d- Pelvic exam

Its US, when severity of the condn is asked, it need not necessarily mean bleeding (submucosal and intramural) it could be pressure sym also(subserosal), hence US. to add, since 36 yr most common is fibroids since adenomyosis in the premenopausal, tho' both can have meno metrorrhagia.

*** whats the most effective way to prevent kidney stones? a- exercise b- diet c- hydration d- periodic U/A *** 25 y/o female G1P0, 16 wks of gestation, presents with vaginal bleeding & cramps. No products of conception is expelled yet. Whats the next step of Mx? a- Adimt to hospital, observation & monitoring b- Bed rest at home c- Admit to hospital, D&C *** Which of the ffg is a contraindication for vaginal delivery after previous c/s ? a- Low segment transverse uterine incision b- breech c- clinically adequate pelvis d- placenta previa a. repeat cs possible, unless longitudinal incision, not done nowadays. b. vaginal can be done. c. prerequisit for vaginal. d. regardless of previous delivery, since the incision would be on the low implanted placenta- torrential bleeding unless the (lower placental edge is more than 2cm from the internal os and it the first delivery without previous CS answer is d. *** 23 y/o female with primary amenorrhea, on P/E breats are abesnt, but uterus is present, what test will u orderfirst ? a- FSH b- Karyotype c- Testosterone d- estrogen FSH. Good explanation in first aid..

*** Post MI pulmonary edema, what do you do next? reduction of pulmonary venous return (preload reduction) and reduction of systemic vascular resistance (afterload reduction) with diuretics ( furesmide ) & morphine sulfate, in a case of hypotension give inotropic agents, supplemental O2.

*** 50 y/o male with known Hx of TB, presents with shoertness of breath, CXR shows massive Rt. side pleural effusion, whats the next step? a- CT b- Thoracocentesis c- Pericardiectomy d- etc. b- thoracocentesis

*** in coarctation of aorta: 1- peripheral resistance a- increase b-decrease 2- pulmonary resistance a- increase b- decrease 1- decrease, 2- increase

*** to prevent hepatitis B in a IV drug abuser, what would you do? a- HBV vaccine b- HBV Ig c- Both d- etc

*** patient presents with trauma to base of neck, damage to which structure causes most severe compromise of cardiorespiratory system? a- thoracic aorta b- jugular vein c- esophagus d- SVC

19 y/o with a small lump in her Lt./ breast is very concerned that its malignant cancer, work up & Bx shows its benign, but she is still extremely worried inspite of reassurance by her physician. Which of the ffg is the best Tx in this case? A- a careful explanation of benign nature of the complaint B- Use of benzodiazepine C- Skillful physician reassurance D- Use of placebo medication

E- Psychotherapy to explore her current life circumstances Answer is E. case of hypochondriasis which is usually becomes evident during psychological stress.

Which of the ffg risk factors are related to breast carcinoma? A- Obesity, late manopause, first pregnancy > 35 yr, smoking, high fat diet B- Obesity, late menopause, low fiber diet, smoking C- First pregnancy>35 yr, obesity, smoking, family Hx, age D- Low fiber diet, age, smoking, first pregnancy> 35 yr, late menopause E- Age, obesity, late menopause, first pregnancy>35 yr, low fiber, high fat diet Answer is E. Smoking is not a risk factor. The most important risk factors are: - Age - Family Hx - Nulliparous state - Early menarche - Late menopause - Hx of contralat. breast cancer - First pregnancy > 35 yr - High fat diet - Low fiber diet

2 y/o boy who palys in a sandbox in a nursery that has couple cats for children to play with, presents with wheezing, hepatosplenomegaly & peripheral blood eosinophilia. Whats the Dx? A- Pinworm infestations B- Lofflers syn. C- Ascariasis D- Visceral larva migrans E- Strongyloidiasis Answer is D. Visceral larva migrans is caused by Toxocara larva. Its most common in children 1-4 y/o especially who have close contacts with digs & cats. Sandboxea are common places for both pets & children. Sx are: fever, hepatosplenomegaly, wheezing, pulmonaer dis. & eosinophilia

17 y/o woman G1P0 with 10 yr Hx of asthma undergoes pulmonary evaluation & spirometery. She doesnt have any respiratory complaints or Sx. Which of the ffg regarding her status during pregnancy is true? A- RR increases

BCDE-

Vital capacity decreases Minute ventilation increases Functional residual capacity remains unchanged Tidal volume decreased

Answer is C. since Px doest have any respiratory problem, shell undergo the normal changes in respiratory physiology that occur during pregnancy: - RR is unchanged - TV is increased - Minute ventilation is increased - VC remains unchanged - FRC is decreased ( uterus elevates the resting position of diaphragm )

35 y/o woman with major depressive disorder, single episode has responded well to imipramine after 1 month of Tx. Which of the ffg is the most appropriate next step? A- Continue imipramine for 6 months B- Continue imipramine indefenitly C- Gradual imipramine decrease till shes medication free, unless depression occurs D- Stop imipramine immediately E- Switch to fluoxetine Answer is A. Maintanace therapy after response to antidepressants should be continued for 6 months after initial response.

14 y/o girl presents with trouble concentrating at school just three months after witnessing her best friend being shot to death by another classmate. Ever since that "awful day" she has been very "sad" and withdrawn, often sitting on her bed, staring at the wall "for hours." She has frequent crying spells and refuses to play with friends or participate in her normal after-school activities. Her appetite has "dwindled down to nothing" and she feels very guilty that she survived the shooting. On further questioning, she reluctantly admits that she has constant thoughts of "joining her friend." She did not receive any counseling after the incident. *** Whats the most likely Dx? A. adjustment disorder B. brief psychotic disorder C. dysthymic disorder D. major depressive disorder E. normal grief F. PTSD

*** Whats the most important q to ask at this time? A. "Are you hearing voices?" B. "Do you have any friends that you feel comfortable talking to at school?" C. "Do you wear a helmet when you ride your bicycle?" D. "Can you describe your typical weekly alcohol intake?" E. "Have you thought of the means by which you can 'join your friend'?"

1- The correct answer is D. This patient most likely has major depressive disorder. To meet the criteria for this disorder a patient must exhibit a 2-week history of a distinct change in mood or a loss of interest or pleasure, along with at least 4 of the following: a decreased appetite and weight loss, difficulty sleeping, psychomotor retardation or agitation, fatigue, feelings of worthlessness or guilt, an inability to concentrate, and suicidal ideation. The symptoms must cause functional impairment. ( Kaplan, step 3 samples ) 2- The correct answer is E. Since she has already told you that she has suicidal ideation ("joining her friend"), it is very important to ask if she has "thought of the means by which" she can "join her friend" or has made any preparatory actions.

A 64 y/o married man has been diagnosed to be HIV positive. He comes in to the office to discuss the results and begs you not to tell his wife, who is also your patient. He says that hearing what he has been in involved in "will kill her." You remind him that HIV may also "kill her." The most appropriate first step is to: A. contact his wife anonymously and tell her that she has been exposed to HIV B. contact the appropriate government health agency and report your findings C. promise him that as his doctor you will respect his privacy and maintain confidentiality D. promise him that you will keep the results confidential if he agrees to use condoms with his wife E. try to persuade him to voluntarily discuss the issue with his wife The correct answer is E. Physicians must violate confidentiality and warn third persons about the danger of HIV infection if the patient is unwilling to inform the person himself. Before informing third parties, the physician should try to do everything possible to persuade the patient to voluntarily discuss the issue with their partner. If the physician believes that an individual may really be saved from a deadly infection, confidentiality should be violated.

A 4 y/o boy presents with fever, irritability, and erythema of the hands and feet for the past week. His mother has been giving him aspirin to reduce his temperature. P/E on admission showed a T=39.7 C (103.4 F), bilateral conjunctival injection, an enlarged right-sided cervical lymph node (1.8-cm), fissured lips, a red tongue with red papillae, pharyngeal hyperemia, erythematous and edematous palms and soles, and a confluent, blanching erythematous rash on the trunk. IV fluids were started, the aspirin therapy was continued. Laboratory studies show ESR= 28mm/h Plt= 490,000/mm3. The patient is extremely uncomfortable and now shows desquamation of the fingers and toes. The most appropriate therapy at this time is: A. corticosteroids B. ibuprofen C. intravenous gammaglobulin D. oxacillin E. penicillin V The correct answer is C. This patient most likely has Kawasaki disease, which is treated with aspirin and intravenous gammaglobulin. The disease is characterized by a high fever for longer than 5 days, bilateral conjunctival injection, fissured lips, a "strawberry tongue", mucosal change in the oral pharynx, erythematous and edematous palms and soles with desquamation, a polymorphous rash, cervical lymphadenopathy, an elevated erythrocyte sedimentation rate, and thrombocytosis. The most important complication is coronary artery aneurysms, which may be prevented by early treatment with aspirin and intravenous gammaglobulin. An echocardiogram is necessary to evaluate cardiac involvement.

47 y/o woman with a Hx of similar attacks of epigastric abdominal pain in the past was admitted to the hospital with a Dx of gallstone pancreatitis. She was NPO and IV fluid started. On the evening of admission day, the patient is noted to have T=103.4 F. Her BP & HR are within normal range. Her abdomen is diffusely tender to palpation with guarding. Whats the most appropriate management at this time? A. draw blood cultures and await results B. draw blood cultures and initiate ampicillin, gentamicin, and metronidazole therapy C. draw blood, urine, and sputum cultures and await results D. obtain an urgent abdominal CT scan E. start ampicillin, gentamicin, and metronidazole therapy The correct answer is B. The most appropriate management at this time is to draw blood cultures and initiate ampicillin, gentamicin, and metronidazole therapy. Intravenous antibiotics are only indicated if there is evidence of pancreatic necrosis or if the patient develops a fever after the diagnosis of pancreatitis is made. There is a substantial amount of clinical literature validating this approach to treating pancreatitis. The appropriate sequence of events is to draw blood cultures prior to initiating therapy in order to

maximize chances of detecting an organism.

A 24 hour old male infant is noted to have some peculiar jerking movements of the right foot and arm. His axillary temperature an hour before was 36.2"C. The physical examination reveals no unusual findings except that he appears small and premature. His birth weight was 2,550 g. The mother's and infant' s history show that he was the second of twins born after 37 weeks' gestation, presented in transverse position and his heart rate had dropped to 80 per minute 10 minutes prior to birth, with documented fetal hypoxia. He had an Apgar score of 3 at 1 minute and 7 at 5 minutes. The mother had nausea and vomiting during pregnancy for which she was given vitamin B6. She had mild preeclampsia at delivery. *** Which is the most likely diagnosis? a) b) c) d) e) Brain tumor Hypoxemia in utero and possibly during delivery Cerebral trauma during delivery Vitamin B6 dependency None of the above

*** In the diagnostic work-up, you may obtain the following tests or procedures, EXCEPT: a) b) c) d) e) CT scan of the head Lumbar puncture for examination and culture of spinal fluid Electroencephalography Serology for toxoplasmosis Blood levels of sugar and calcium

B&D Hypoxemia is the leading cause of seizures considering the history of drop in fetal heart rate and low Apgar score at 1 minute. Hypoglycemia, hypocalcemia, vitamin B6 dependency, and meningitis are possible causes of seizures but unlikely and should be ruled out. Some cerebral anomaly is possible. Incidence of congenital malformations is higher in twins than , in singletons, and central nervous system malformations lead all others in frequency. Brain tumors at this age are rare and usually present with recurrent vomiting and not with seizures.

An infant who was healthy at birth is brought to your office for her first office visit at the age of 6 weeks. You notice that the infant is jaundiced and that there is bilirubin staining of the wet diaper. Which one of the following diagnoses is most consistent with these findings? a. Physiologic jaundice of the newborn b. Hemolysis secondary to Rh incompatibility c. Crigler-Najjar syndrome d. Gilbert's syndrome e. Biliary atresia Answer is e- biliary atresia. bilirubbin in urine a clue of direct hyperbilirubinemia

A 22 y/o man used illicit IV drugs briefly 1 year ago. He is asymptomatic and has no history of viral hepatitis. Serum ALT and bilirubin concentrations are normal. Serologic studies show: Hepatitis B surface antigen (HBsAg) Hepatitis B core antibody (HBcAb) Hepatitis B e antigen (HBeAg) Antibody to surface antigen (Anti-HBs) Antibody to e antigen (Anti-HBe) positive positive negative negative positive

Which of the following statements best describes the current condition? a-He is in the incubation period, is highly infectious, is likely to develop acute hepatitis B. b-He is in the incubation period, is minimally infetious, and is likely to develop acute hepatitis B. c-He is a chronic carrier of hepatitis B and is highly infectious. d-He was infected previously with hepatitis B. has recovered and is immune to hepatitis B. Answer is a.

A 35 y/o woman requests a routine health assessment. She has no specific medical complaints or past medical hstory. Her father developed colon cancer at age 54 but is still living. His brother died of colon cancer at aae 41. Their father (the patient's grandfather)

died in his sixties of colon cancer. His sister died of gastric cancer. The patent's sister was recently diagnosed with endometrial cancer at age 43. The patient has a good appetite and has no nausea, vomiting, or weight loss. She has had no change in her bowel habits or hematochezia, and her menstrual periods are normal. Physical examination is normal. Digital rectal examination is negative for fecal occult blood. A screening complete blood count and biochemical profile are normal. Which of the following actions is most appropriate? a. Perform annual occult blood tests of three stool specimens. Initiate screening with flexible sigmoidoscopy at age 50. b. Perform colonoscopy at least every 2 years. c. Barium enema now; if normal no further evaluation. d. Perform colonoscopy now; if no polyps are seen, no further investigation is necessary. e. Perform fecal occult blood testing and flexible sigmoidoscopy now. If no polyps are seen now, the patient should be followed routinely with fecal occult blood testing and flexible sigmoidoscopy beginning at age 50. Answer is b.

A 20 y/o woman is evaluated for fever of nine weeks duration. During this time she has had daily temperature elevations to 40.0 C (104.0 F). The only other symptoms have been malaise and occasional aches in the hands and knees; on one occasion she noted a transient pink rash on the abdomen. A one-week course of ampicillin had little effect. T=38.9 C (102.0 F); PR=108 per minute, and rhythm is regular. A grade 1/6 systolic ejection murmur is heard best at the LSB. The spleen is palpable 3 cm below the left costal margin on deep inspiration. Electrocardiogram and chest radiograph are normal. Laboratory studies: Hct Hgb WBC 35% 11.5 g/dL 12,800/cu mm; 81%neutrophils, 4% monocytes, 14 lymphocytes, 1% eosinophils 250 Todd units {<200} Negative Negative Pending

ASO RF ANA Blood cultures

Which of the following is the most likely diagnosis? (A) (B) (C) (D) (E) Answer is e. Enteric fever Bacterial endocarditis Lyme disease Hodgkin's disease Adult-onset Still's disease

A healthy 71 y/o man describes visual loss in his right eye. Flashes of light and a curtainlike loss of lateral vision began when he awoke eight hours ago. These symptoms have persisted. Which of the following is the most likely explanation? (A) Retinal vein occlusion (B) Retinal detachment Atheroembolic occlusion of a lateral branch of the right retinal (C) artery (D) Ocular migraine (E) Occipital lobe seizure Answer is B.

What is the marker for CREST syn. ? A. anti-scl-70 B. anti centromere C. anti ds-DNA D. SS-A (Ro) The marker for CREST is anti centromere. They both have C in it. anti-scl-70 is for scleroderma diffuse type. They both have scl in it. D. anti ds-DNA is for SLE. E. SS-A is for Sjogren's disease

Whats the characteristics of rapidly progressive glomerulonephritis (RPGN)?

A. crescent formation B. "lumpy-bumpy" subepithelial deposits C. IgM mesangial deposition D. Associated with Hep C the answer is A. Rapidly progressive glomerulonephritis is characteristic of rapidly losing at least 50% of its glomeruli in as short period of time--usually days to 3 month max. Extensive fibrinoid necrosis is found on biopsy.

Pt. In mVA was brought to ER, you suspected cardiac tamponade what is next step? a. pericardiocentesis b. echo first.. C. CXR d. CT Answer: depending on senario if pt is unstable proceed to Pericardiocentesis, If stable echo first.

A patient has end stage pancreatic cancer and as her primary care pysician, she asks you how long she could survive. What you should tell her. a. 5 year rate is 30% b. 5 year rate is 20% c. " 15% d. " < 5% Answer: D- The overall 5-year survival rate for this disease is less than 5%.

A 24-year-old white primigravida has developed several 1- to 2-mm erythematous papules on her abdomen in the third trimester. They are pruritic and tend to appear in her striae. Liver function tests and a CBC are normal. Which one of the following is the most likely diagnosis? a. Pruritus gravidarum b. Spangler's papular dermatitis c. Impetigo herpetiformis d. Herpes gestationis e. Pruritic urticarial papules and plaques (PUPP) The ans is e The findings in this patient are most consistent with PUPP. This condition is usually benign, is not associated with increased fetal morbidity, and resolves after delivery, and

there is usually no recurrence in subsequent pregnancies. Herpes gestationis, impetigo herpetiformis, and Spangler's papular dermatitis have different presentations and may be associated with increased fetal morbidity. Pruritus gravidarum is characterized by pruritus without skin lesions.

Ultrasonography reveals placenta previa in a 41-year-old asymptomatic G4P3 at 21 weeks gestation. Appropriate management would be a. weekly speculum examinations under aseptic conditions beginning in her third trimester to assess the risk of bleeding b. an MRI scan, with a repeat scan later in the pregnancy if indicated c. repeat ultrasonography in her third trimester d. cesarean delivery at 28 weeks gestation if her L/S ratio is favorable e. reassurance that ultrasound diagnosis of placenta previa without evidence of bleeding is no cause for concern and can be disregarded The ans is c The incidence of placenta previa ranges from 6% to 45% in the second trimester, but more than 95% of these resolve by the third trimester. However, it remains a cause for concern and should be watched, not ignored, even if there is no bleeding. This patient should have repeat ultrasonography in her third trimester. An MRI is very helpful but need not be used except in a difficult diagnostic situation. Weekly speculum examinations would create a risk of hemorrhage. Delivery at 28 weeks would not be appropriate in a patient with no symptoms and without confirmation of the persistence of placenta previa.

Which one of the following is an absolute contraindication to tocolytic treatment for preterm labor? a. Urinary tract infection b. Documented gestation less than 28 weeks c. Chorioamnionitis d. Uncontrolled diabetes mellitus e. Any vaginal bleeding due to mild abruptio placentae The ans is c Before tocolytic treatment is instituted, absolute contraindications to tocolysis must be ruled out. These include chorioamnionitis, severe abruptio placentae, severe bleeding from any cause, severe pregnancy-induced hypertension, fetal death, fetal anomaly incompatible with life, and severe fetal growth retardation. Chorioamnionitis may precipitate preterm labor and is an absolute contraindication to tocolysis. It may be present in a febrile pregnant patient even with intact membranes. In this case

amniocentesis may be required to rule out infection. There are also a number of relative contraindications. These include uncontrolled diabetes, hyperthyroidism, maternal cardiac disease, mild chronic hypertension, mild abruptio placentae, stable placenta previa, fetal distress, fetal anomaly, mild fetal growth retardation, and cervical dilatation greater than 5 cm. In patients with relative contraindications to tocolysis the risk of complications from prematurity must be weighed against the risk of tocolysis. Not all vaginal bleeding is due to a serious obstetric condition. Cervical effacement or dilatation may be the cause. Even if the source of bleeding is determined to be a placental abruption, if the bleeding is minor, the abruption is mild, and the fetus is not in distress, tocolysis is not absolutely contraindicated. While diabetes mellitus may be adversely affected by beta-adrenergic tocolytic agents, it is not an absolute contraindication to tocolysis. Close glycemic monitoring is, of course, mandatory. Even a few weeks of effective tocolysis may significantly alter the perinatal outcome of gestations between 25 and 27 weeks. Gestational age less than 28 weeks is therefore not a contraindication to tocolysis. While a urinary tract infection may precipitate preterm labor, tocolysis is not contraindicated. The infection, of course, should be treated.

Which one of the following statements is most accurate concerning juvenile rheumatoid arthritis? a. Fever is a rare systemic manifestation b. Ten years after the onset of disease, most patients have excellent functional status c. Most patients have a permanent deformity of at least 1 extremity d. The disease is characterized by lifelong recurrences e. Most patients require corticosteroid treatment answer is b. At least 50% of patients followed for up to 15 years have complete remission of juvenile rheumatoid arthritis, and 70% regain normal function. A few patients are left with crippling joint deformities, but 75% have no significant residual deformity. Systemiconset disease is accompanied by high fever, rheumatoid rash, polyarthritis, and other systemic manifestations

A 5-year-old African-American male fell off his bicycle and hit the back of his head on a hard surface. There was no loss of consciousness. No other injury was noted. He was obviously agitated and restless, and his only complaint was a loss of vision. When you see him, his examination is unremarkable except for moderate swelling over the occipitoparietal area of the scalp. His skin is intact, and no gross neurologic deficit is noted except for the visual loss. A CT scan is negative. An EEG shows only slight slowing of activity. Which one of the following statements is true regarding this patient?

a. The child's vision will probably return within 24 hours b. The child should be hospitalized for 72 hours for further observation c. It will be months before the child's vision returns d. The loss of vision is probably caused by damage to the optic nerve Ans is a Transient cortical blindness following mild head trauma is usually associated with a benign outcome. The special features are mild head trauma, no loss of consciousness, onset of blindness occurring within hours of the trauma, duration of blindness less than 24 hours, absence of skull fracture or visible injury on CT scan, and no other neurologic deficits. The EEG shows initial slowing with normalization on follow-up.

which of the following medication is known to exacerbate psoriaisis? a)prednisone B NSAID c hydroxychloroquine d methotrexate answer is c.

1- A 20 y/o man who moved to the US from Cambodia 2 years ago presents to the ER with the sudden onset of left hemiparesis. His wife, who emigrated with him, reports that he has not felt well for at least a month and has had a weight loss of about 9 kg (20 lb). On P/E, T=38 C (100.4 F) and BP=116/52 mm Hg. He is somewhat cachectic and hemiparetic. The cardiac examination shows a murmur. The physical examination is otherwise normal.Which of the following tests is most likely to give the diagnosis? (A) Blood cultures (B) Lumbar puncture (C) Radiography of the chest (D) Complete blood count, differential, platelet count, and erythrocyte sedimentation rate (E) Biopsy of inguinal lymph node (F) head CT scan Answer is A.The appearance of focal neurologic signs in a young person from Cambodia raises a wide differential diagnosis. Atherosclerotic cerebrovascular disease is uncommon in the 20-year-old age group. Among other diagnostic considerations are an embolic event from a cardiac source, such as a valvular vegetation associated with endocarditis or

an atrial myxoma; vasculitis; tuberculomatous or bacterial brain abscess; brain tumor; aneurysm or arteriovenous malformation; and coagulation disorder such as thrombotic thrombocytopenic purpura or hyperviscosity syndrome caused by multiple myeloma. Neurologic complications occur in 25% to 40% of patients with infective endocarditis. About 15% develop cerebral emboli with associated neurologic symptoms. The easily associated triad of new focal neurologic deficits, fever, and changing heart murmur occurs in only 33% of patients, so infective endocarditis must be considered in any patient with sudden focal neurologic deficits who has no conventional risk factors, such as atherosclerosis.

2- A 35 y/o woman presents with a 1.5-cm Lt breast mass which is nodular with indistinct borders. Her mother and maternal aunt both had breast cancer in their forties. How would you evaluate this woman's condition? (A) Observe through a menstrual cycle (B) Diagnostic mammogram (C) Breast ultrasound (D) Office needle aspiration This woman has a significant risk for breast cancer. The suspicion of cancer is high because of the characteristics of the mass and the family history of early-age breast cancer. The first approach would be a diagnostic mammogram to further define the lesion and, equally important, to look for suspicious lesions in the opposite breast. Even if the mammogram is negative, referral to a surgeon for biopsy would be the next step.

3- A 22-year-old female primagravida is seen in prenatal clinic 5 months after her last menstrual period. Her pregnancy has been uneventful, and she has gained weight progressively over the last 2 months. Three months ago, her blood pressure was 120/80 mm Hg, and there was 2+ protein on dipstick urinalysis. Today she has a blood pressure of 150/95 mm Hg and marked bilateral lower extremity edema.
Laboratory studies: Hematocrit Leukocyte count Platelet count 29.7% 4200/L 209,000/L

Blood urea nitrogen Serum creatinine Serum uric acid Urinalysis: Protein Microscopic 24-hour urine protein excretion Creatinine clearance

15 mg/dL 1.1 mg/dL 6.0 mg/dL 4+ Hematuria, rare erythrocyte casts, rare broad casts, few leukocytes 12.5 g 80 mL/min

This clinical presentation is most consistent with: (A) Preeclampsia (B) Underlying renal disease present before conception (C) Hypertensive nephrosclerosis (D) Pyelonephritis Answer is B. In this 22-year-old pregnant woman, it is critical to determine whether previously existing underlying renal disease is present because of the overlap with signs of preeclampsia and the differing prognoses of the two conditions. Preeclampsia is a complication that presents after 20 weeks of gestation, which suggests the abnormal proteinuria in this patient is associated with another disease. The finding of 2+ protein on dipstick urinalysis is not specific, but the hematuria and erythrocyte casts suggest underlying glomerulonephritis rather than preeclampsia.

4- A 50-year-old man who has had diabetes mellitus for 12 years is concerned about becoming dependent on dialysis. His disease has been inadequately controlled (hemoglobin A1C range 8.7% to 11.8%). His urinalysis is strongly positive for protein, and his serum creatinine concentration has increased from 1.2 to 1.9 mg/dL in the past 8 months. His blood pressure has been 150/90 mm Hg and on occasion as high as 210/120 mm Hg. Which treatment is most important to reduce the process of this patient's kidney disease? A. Insulin B. Antihypertensive agent C. HMG-CoA reductase inhibitor ("statin") D. Protein-restricted diet E. Aspirin Answer is B. This patient has already started on the path to progressive diabetic nephropathy. Although all of the preventive measures still warrant attention, blood

pressure control at this stage is most important. From the onset, good glycemic control (hemoglobin A1C 7% to 7.5%), a protein-regulated diet to reduce intraglomerular pressure as well as antihypertensive therapy with an ACE-inhibitor should be considered.Antihypertensive therapy attenuates the decline in renal function in patients with all forms of diabetes.

5- A 26-year-old patient is presented with amenorrhea of 2.5-month duration. Lab analysis indicates increased levels of thyroid and cortisol binding proteins. There is an elevated total cortisol and ACTH. Free cortisol level is within normal range. These findings are most likely suggestive of which of the following conditions? A. Hypothyroidism B. Addison's disease C. Conn's disease D. Cushing's disease E. Pregnancy answer is E.

Two weeks following a viral illness, a teenage boy breaks out in an evolving rash that is remarkable for target lesions. What is the primary treatment? a. Epinephrine b. Glucagon c. Corticosteroids d. Antihistamines e. Symptomatic or supportive therapy depending on severity. Answer is e. its erythema multiform

The most prevalent of allergic disease in school-age children is: a. Atopic dermatitis b. Food allergy c. Asthma d. Allergic rhinitis e. Drug allergy Answer is d.

Cheilosis and glossitis are features of ( more than one answer ): a. vit A def. b. riboflavin def. c. vit. C def. d. pyridoxine def. e. vit. E def. Answers are b&d. Which one is the most common adverse effect of intranasal steroids? a. Nasal irritation b. Septal perforation c. Nasal bleeding d. Short stature e. Adrenal suppression Answer is a.

Increased risk for intussusception was observed as a rare complication following immunization with which vaccine? a. IPV b. OPV c. Rotavirus vaccine d. HAV e. HBV Answer is c.

A 60 y/o Japanese man visiting US with excellent health until 6 months ago, when he first noted mild upper abdominal fullness after meals. On P/E hyperpigmented, heaped-up velvety lesions in the neck, axillae, and groin is noted. Which of the following conditions is associated with the skin findings? a- Non-Hodgkin's lymphoma b- Anorexia nervosa c- Acute leukemia d- Adenocarcinoma of the stomach

e- Addison's disease

Answer is d, skin lesion is acanthosis nigricans.

Which of the following should be done annually after age 40 in the asymptomatic, average- risk man in order to promote the early detection of cancer? a- Colonoscopy b- Sigmoidoscopy c- Digital rectal examination with palpation of the prostate d- Digital rectal examination with palpation of the prostate and stool guaiac e- Digital rectal examination with palpation of the prostate, stool blood test, and chest x-ray

Answer is c.

During a routine checkup, a 65 y/o man is found to have a level of serum Alk Ph. three times the upper limit of normal. Serum Ca and ph. concentrations and LFT results are normal. He is asymptomatic. The most likely diagnosis is a- metastatic bone disease b- primary hyperparathyroidism c- occult plasmacytoma d- Paget's disease of bone e- osteomalacia answer is d.

Which of the following is NOT a predisposing factor for the development of a hernia? a- Ascites b- Obesity

c- Cystic fibrosis d- Chronic obstructive pulmonary disease e- Peritoneal dialysis answer is b. Ascites, peritoneal dialysis, ventriculoperitoneal shunt, cystic fibrosis, and chronic obstructive pulmonary disease all predispose patients to hernia formation because they increase intraabdominal pressure. Other risk factors include a positive family history, undescended testis, and genitourinary abnormalities.

A 5-month-old infant has had several episodes of wheezing, not clearly related to colds. The pregnancy and delivery were normal; the infant received phototherapy for 1 day for hyperbilirubinemia. He had an episode of otitis media 1 month ago. There is no chronic runny nose or strong family history of asthma. He spits up small amounts of formula several times a day, but otherwise appears well. His growth curve is normal. An examination is unremarkable except for mild wheezing. Which one of the following is the most likely diagnosis? A) Benign reactive airway disease of infancy B) Cystic fibrosis C) Unresolved respiratory syncytial virus infection D) Early asthma E) Gastroesophageal reflux Answer is E, gastroesophageal reflux is a common cause of wheezing in infants. At 5 months of age, most infants no longer spit up several times a day, and this is a major clue that the wheezing may be from the reflux. Also, there is no family history of asthma and the wheezing is not related to infections. Cystic fibrosis is more likely to present with recurrent infections and failure to thrive than with intermittent wheezing.

*** ECG findings of pulmonary edema include all of the following except: a- deep S1 b- depressed ST in lead I & II c- prominent Q1 & inversion of T3 d- left axis deviation e- clockwise rotation in the precordial leads answer is d. it cause Rt. axis deviation.

*** chest pain & friction rub 3 days after admission to ICU, indicate which of the ffg? a- misdiagnosis of infarction

bcde-

chest trauma viral infection transmural infarction dissecting aneurysm

answer is d.

*** Neuropathy secondary to gout manifests as: a- NS b- ARF c- ATN d- Isosthenuria & moderate albuminuria e- Malignant HTN Answer is d.

*** Tx of choice for cutaneous manifestation of protoporphyria is: a- phenobarb. b- Corticosteroid c- High carb. Diet d- Beta caroten e- Chlorpromazine Answer is d.

*** 25 y/o female with intermittent double vision, on CXR an Ant. Mediastinal mass is noted. Whats the next step of Mx? a- serum Ca messurement b- brain MRI c- evaluation of T cell function d- serum gamma globulins messurement e- order GTT answer is d. association of thymoma and myasthenia gravis. 5-10% of Px with thymoma have low serum gamma globulins.

*** 76 y/o male with 80 pack/y smoking Hx, was diagnosed with lung cancer 4 m ago, hes brought to hospital in state of coma, his serum ca= 16 mg/dl, which of the ffg, would be the most useful to reduce ca rapidly: a- acetazolamide b- furesmide c- hydrochlorothiazide

d- manitol e- spironolactone answer is b- furesmide which increase urinary excretion of ca.

*** 50 y/o man with Hx of smoking & cough for the past 2 months, is found to have a solitary pulmonary nodule with no mediastinal dis on CXR. Hes expected to have best prognosis if he has which type of ffg tumors? a- small cell carcinoma b- poorly diff. Adenocarcinoma c- metastatic carcinoma d- well diff. SCC e- well diff. Adenocarcinoma answer is e- well diff adenocarcinoma

*** 27 y/o woman presents with sudden pain & decrease in hearing in Rt. ear while picking it with matchstick. On P/E, a traumatic rypture of Rt. tympanic membrane & some blood clot is seen. Whats the best advice or procedure? a- advice her to use earplug while shampooing & showering b- advice her to clean the ear canal with Q-tips c- remove the blood clot d- antibiotic eardrops e- oral + eardrop antibiotics answer is a- advice her to use earplug while shampooing & showering

*** 3 days after hospital admission wih Dx od acute pancreatitis, a 45 y/o man has 5 of the ranson criteria, On P/E, T=102, BP=120/70, PR= 130, RR= 18, Px is in obvious distress, abdomen is distended with no audible bowel sounds, tenderness on upper abd. without any rebound tenderness is noted. Whats the best next step to confirm the Dx? a- serum amylase b- paracentesis c- ERCP d- Dynamic CT scan e- Laparascopy f- US g- Abdominal x-ray Answer is d- dynamic CT scan is the best investigative way for pancreatic necrosis.

*** A 65 y.o. man with ischemic heart disease comes to the ER in acute CHF. The ophthalmic medicine most likely to be responsible is a. Acetazolamide b. Pilocarpine c. Tobridex d. Trifluoridine e. Timolol maleate Answer is e- timolol maleate.

*** A 40 y.o. w/spastic bladder is treated w/ anticholinergics. The pt. will complain of: a- Diplopia b- Decreased distance vision c- Decreased near vision d- Decrease in color vision e- Snowy, sparkly vision Answer is c- decreased near vision

*** Which of the following lab values would be most consistent with a woman who has a Hx of lung cancer and has been on chemotherapy for several weeks.The patient now presents with acute anemia. a. reticulocyte count of 60 x 109 b. plasma Hgb of 7.2 g/dl c. Hct of 34% d. MCV of 70 fL e. RBC of 5.9 million/mm3 answer is a.

*** A healthy 12-month-old boy has a brother with adenosine deaminase deficiency. Which of the following vaccines should you NOT administer to the infant? (A) Diphtheria-tetanus-pertussis (DTP) (B) H. influenzae (HiB) (C) Hepatitis B (Hep B) (D) Oral polio vaccine (E) Measles-mumps-rubella (MMR)

answer is D- The main problem in this scenario is administering a live, attenuated vaccine that may be followed by transmission to the immunocompromised sibling and cause severe, possibly fatal, disease. Of the attenuated vaccines, the oral polio vaccine is the most dangerous because the virus is shed in the feces, while attenuated vaccines administered by inoculation (such as measles-mumpsrubella), which are not shed by the recipient, are considerably less likely to be transmitted from person to person. The other three vaccines are not live vaccines and have no special risk in an immunocompromised individual

*** A 30-year-old man sustains brain damage as the result of an automobile accident. Neurologic examination reveals incomplete retrograde amnesia and severe anterograde amnesia as well as inappropriate social behavior, including hyperphagia, hypersexuality and general disinhibition. The brain injury would most likely involve the: (A) frontal lobes, lateral convexity. (B) frontal lobes, medial surface. (C) temporal lobes, lateral convexity. (D) temporal lobes, medial surface. (E) thalami. Answer is D- Bilateral damage of the medial temporal gyri, including the amygdalae, may cause severe memory loss (hippocampal formations). Such damage to the amygdalae may lead to inappropriate social behavior (e.g., hyperphagia, hypersexuality, general disinhibition). Bilateral destruction of the amygdalae results in the Klver-Busy syndrome.

*** A 24 y/o college student, G1P1 presents for her annual pelvic examination and Pap smear, and renewal of her ocp. Hx & P/E are unremarkable. Her Pap smear is subsequently reported as high-grade squamous intraepithelial lesions (HGSIL). The most appropriate next step in this patient's management is (A) Repeat Pap smear in one year. (B) Repeat Pap smear now. (C) Colposcopy with ECC and directed biopsy. (D) Excisional biopsy. (E) Cervical conization. Answer is C. HGSIL on screening examination requires histologic diagnosis so that appropriate treatment can be selected. Repeat Pap smear is another screening test and is thus inappropriate. Conization for diagnostic purposes is needed only if colposcopy and biopsy prove inadequate.

*** A 63 y/o man with no significant past medical history is admitted with an acute abdomen secondary to gastric perforation. If a definitive ulcer procedure is performed on this patient, which of the following procedures is associated with the lowest ulcer recurrence rate? (A) Truncal vagotomy (B) Truncal vagotomy and pyloroplasty (C) Vagotomy and antrectomy (D) Parietal cell vagotomy (E) Gastric bypass answer is c. Among the choices provided, vagotomy with a simultaneous antrectomy has the lowest ulcer recurrence rate. However, this procedure is associated with a higher morbidity secondary to anastomotic complications (e.g., leak or stricture) or problems associated with denervation of the proximal stomach and distal bowel. Complications include postvagotomy diarrhea, dumping syndrome, delayed gastric emptying and alkaline reflux gastritis.

*** A 16 y/o boy presents to his physician with a history suggestive of systemic lupus erythematosus. The patient has a rash, arthralgia and mild proteinuria. Serologic evaluation detects no antibodies of diagnostic value. Which one of the following diseases is most likely to be diagnosed in this patient? (A) Rheumatoid arthritis (B) Common variable hypogam-maglobulinemia (C) C2 deficiency (D) Sjgren's syndrome (E) Wegener's granulomatosis answer is c. A deficiency in complement component C2 may manifest itself as a disorder similar to systemic lupus erythematosus, possibly because of a failure of complementdependent mechanisms to eliminate immune complexes. Most patients with rheumatoid arthritis have several antibodies (most notably the anti-IgG antibody known as rheumatoid factor). Similarly, Sjgren's patients are diagnosed by detection of antibodies to exocrine gland duct epithelium. Patients with Wegener's granulomatosis have antineutrophil cytoplasmic autoantibodies. Common variable hypogammaglobulinemia will evidence itself by the occurrence of repeated bacterial infections.

1- a 4 y/o child presents with upper respiratory illness. P/E reveals mental retardation, eczema, hypopigmentation and blue eyes. Whats the most likely Dx? a- Down syn. b- Tuberous sclerosis c- PKU d- Osteogenesis imperfecta e- Cretinism f- Galactosemia Answer is PKU.

2- 28 y/o male IV drug abuser presents with painful, erythematous nodules on fingerpad and linear hemoorhage beneath the fingernail of indexfinger. On P/E a grade II, high pitched diastolic murmur is heard on 2nd & 3rd Rt. ICS. Whats these lesion on the hand are related to? a- immunocmplex vasculitis b- thrombocytopenia c- coagulation factor def. d- DIC Answer is a. Px has infective endocarditis due to staph, aureus involving aortic valve.the nodules are oslers nodes and nailbed finding is splinter hemorrhage and both are example of immunocomplex vasculitis.

3- 53 y/o woman presents with SOB and weight loss despite eating well, she also have intermittent episodes of heart racing. On P/E, multinodular thyroid and warm and moist palm are noted. Hr=106/min, whats the most likely Dx? a- graves dis. b- Papillary adenocarcinoma c- Plummers dis. d- Follicular adenoma Answer is c ( or toxic nodular goiter ), its hyperthyroidism associated with multinodular goiter.

4- on a routine examination of a 5 y/o female a heart murmur is heard. Its medium pitched, systolic ejection murmur, with musical quality. Which of the ffg is the characteristic of this murmur: a- its best heard along the lower Lt. & midsternal border b- its common in infancy c- its best heard while Px is lying down on the Lt. d- its less intense with fever or excitement

answer is A. an innocent murmur, which is best heard in supine position. It occurs only in systole ( never in diastole ), uaually grade I or II, and heard on LSB in children between ages 3-7 ( rarely in infancy ).

5- Which of the ffg is consistent with an adult Px with RA & salysilate intoxication: PH PCO2 Bicarb a- 7.29 53 25 b- 7.38 22 12 c- 7.53 49 39 d- 7.43 70 46 e- 7.28 28 12 answer is b. respiratory alkalosis+metabolic acidosis

** 32 y/o athelete presented with jaundice.no previous liver disease.investigation showed elevated Alk Ph.,no viral infection and no stones. what is ur presumptive diagnsis and possible cause? cholestatic jaundice due to methyl testosterone administration.

** 35 y/o woman had an attack of billiary colic. which analgesic is contraindicated and why? morphine increase intrabilliary pressure which worsens the pain.

1- A 70-year-old African-American male who has been hospitalized for 2 weeks for congestive heart failure develops severe, persistent diarrhea. For the past 3 days he has had abdominal cramps and profuse semi-formed stools without mucus or blood. The patients current medications include captopril (Capoten), digoxin, furosemide (Lasix), subcutaneous heparin, spironolactone (Aldactone), and loperamide (Imodium). He has coronary artery disease, but has been relatively pain free since undergoing coronary artery bypass surgery 4 years ago. An appendectomy and cholecystectomy were performed in the past, and the patient has since been free of gastrointestinal disease. On physical examination his blood pressure is 100/80 mm Hg, pulse 100 beats/min and regular, and temperature 37.0 C (98.6 F). He has mild jugular venous distention and crackles at both lung bases. Examination of his heart is unremarkable, although there is 1+ dependent edema. His abdomen is diffusely tender without masses or organomegaly. Rectal examination is normal. The results of routine laboratory tests, including a CBC, chemistry profile, EKG, and urinalysis, are all normal. The stool examination shows numerous white blood cells. Of the following, the most likely diagnosis is

A) viral gastroenteritis B) Clostridium difficile colitis C) ulcerative colitis D) gluten-sensitive enteropathy (celiac sprue) E) digoxin toxicity This patient most likely has Clostridium difficile colitis, suggested by semiformed rather than watery stool, fecal leukocytes (not seen in viral gastroenteritis or sprue), and a hospital stay greater than 2 weeks. While this disease has traditionally been associated with antibiotic use, it is posing an increasing threat to patients in hospitals and chroniccare facilities who have not been given antibiotics. The primary sources for infection in such cases have been toilets, bedpans, floors, and the hands of hospital personnel. Prompt recognition and treatment is essential to prevent patient relapse and to minimize intramural epidemics. The diarrhea of ulcerative colitis usually contains blood and occurs intermittently over a protracted course. Digoxin toxicity is likely to be accompanied by electrocardiographic and laboratory abnormalities, particularly hyper- or hypokalemia.

2- A 79-year-old man is admitted to the medical ward 3 days status post subdural hematoma drainage, C3 cervical spine fracture, and fixation of multiple extremity fractures sustained in a motor vehicle accident. The patient is now awake and oriented to person, place, and time, but is a lower cervical spine incomplete quadriplegic. Physical examination reveals some minimal sensation in the legs, but no ability to move the extremities. There is a Foley catheter in place that is draining yellow colored urine. Doppler ultrasonography demonstrates a thrombus in the left popliteal vein. The most important next step in the management of this patient is A. daily Doppler ultrasonography of the lower extremities B. inferior vena cava filter placement C. subcutaneous heparin D. tissue plasminogen activator thrombolysis E. warfarin F. weekly ventilation/perfusion scans for a pulmonary embolus The correct answer is B. This patient has documented deep venous thrombosis (DVT) on ultrasonography and has had recent intracranial surgery. Intracranial surgery is an absolute contraindication to anticoagulation. Because the patient has a documented DVT, an inferior vena cava filter is necessary to prevent potentially fatal pulmonary emboli.

Daily ultrasonography of the lower extremities (choice A) is a way to follow the extent of the documented thrombus in the left popliteal vein. It is not sufficient to simply follow the extent of the clot, however. The known DVT puts him at a risk for a pulmonary embolus, and therefore he needs an inferior vena cava filter. Anticoagulation with subcutaneous heparin (choice C) is absolutely contraindicated as the patient has had recent neurosurgery. Thrombolysis (choice D) will treat the current thrombus in the left popliteal vein, but it will not prevent further thromboses from occurring. Thrombolysis with tissue plasminogen activator is also contraindicated so soon after intracranial surgery. Anticoagulation with warfarin (choice E) is absolutely contraindicated as the patient has had recent neurosurgery. A screening study for pulmonary emboli (choice F) is inadequate for this high-risk patient. Although both ventilation/perfusion scanning and CT pulmonary angiography are effective studies to diagnosis a pulmonary embolus, this patient needs definitive treatment to prevent a pulmonary embolus.

3- A 23-year-old college student comes to the clinic because of odynophagia with solids and liquids and dysphagia that is most severe when eating solid foods. The patient had a past medical history of Shigella colitis last year while she was a Peace Corps volunteer in Peru. She takes oral contraceptives and smokes 1 pack of cigarettes daily. She does not drink alcohol. Vital signs are: temperature 37.8 C (100 F), blood pressure 100/70 mm Hg, pulse 79/min, and respirations 8/min. Physical examination is normal. Electrocardiogram reveals normal sinus rhythms with a rate of 85/min and a markedly enlarged QRS complex in leads V3-V5. Chest x-ray reveals an enlarged cardiac silhouette. A barium esophagram demonstrates a tapering of the distal esophagus that eventually releases as the esophagus is distended. There is no evidence for extrinsic or intrinsic compression of the distal esophagus or an esophageal mass. There is no reflux. The test most likely to lead to a unifying diagnosis in this case is A. an agglutination test for trypanosomes B. a chest CT C. an esophageal manometry D. a liver biopsy E. a myocardial biopsy

The correct answer is A. The findings of achalasia and cardiomyopathy in a patient with history of travel to Central or South America support the diagnosis of Chagas disease. This is caused by infection of Trypanosoma cruzi in the muscles of the heart, esophagus, and colon. A serum agglutination test is a noninvasive means of testing for this infection, and is highly sensitive. Left untreated, heart failure and megacolon could develop. A chest CT (choice B) would not reveal any specific findings of Chagas disease. Using a CT to evaluate for an occult cancer causing esophageal narrowing is superfluous given the findings on the barium esophagram, which is specific for achalasia. An esophageal manometry (choice C) is a confirmatory test for achalasia. Given the highly suggestive findings of achalasia on the barium esophagram, manometry would not be necessary. A liver biopsy (choice D) has no role in the evaluation of Chagas disease. A myocardial biopsy (choice E) is not necessary to diagnose Chagas disease with the availability of the serum agglutination test for trypanosomes. Invasive procedures should be deferred until they are absolutely necessary.

4- A 17-year-old girl is brought to the office by her mother because she has missed many periods. The girl admits to binge eating and exercising in order to prevent weight gain. She tells you that she is definitely not pregnant, because she has not had any sexual relations in the past 11 months and thinks she is not getting her menstrual period because of the excessive physical exercise she has been doing in the past several weeks. Physical examination is significant for bradycardia and significant weight loss compared to the last year. A pregnancy test is negative.At this time you should order A. amylase B. BUN and creatinine C. liver function tests D. serum potassium E. thyroid function tests The correct answer is D. Bulimic patients frequently engage in compensatory behaviors to prevent weight gain. Those include self-induced vomiting, abuse of diuretics, laxatives, enemas, or diet pills. The metabolic disorders frequently seen in these patients are, mostly hypokalemia and hypomagnesemia.

Amylase (choice A) is not a routine test ordered in bulimic patients. If there has been evidence of long starvation and suspicion of other medical conditions, its level might alter. BUN and creatinine (choice B) can be increased if there are signs of dehydration, secondary to the abuse of diuretics. Otherwise, these values should not be changed significantly. Liver function tests (choice C) are usually not changed in bulimic patients. They can be ordered to rule out other medical conditions if necessary. Thyroid function tests (choice E) should be done as a part of regular workup of patients presenting with this clinical picture. It is not, however, the first to be ordered.

5- A 76-year-old woman is brought to the hospital by her son because of "rapid breathing." She has advanced Alzheimer disease and is unable to give a coherent history. She was recently diagnosed with breast cancer. She lives alone, but normally has a health care aide during the day. The aide was not available when the son tried to reach her to ask if anything happened. The son has not seen his mother in 2 months. An accentuated fall in systolic blood pressure during inspiration would most likely suggest A. anxiety B. cardiac tamponade C. myocardial infarction D. senile aortic stenosis E. sepsis The correct answer is B. This question describes pulsus paradoxus, which is when there is an accentuated fall in systolic blood pressure (>10 mm Hg) during inspiration. There is usually a small decrease in blood pressure during inspiration, but it is exaggerated in cardiac tamponade because the external compression caused by fluid accumulation around the heart leads to impaired ventricular filling, reduced left ventricular stroke volume, and a reduction in systolic blood pressure. Cardiac tamponade can occur acutely after trauma or develop chronically from the accumulation of pericardial fluid malignancies (she has breast cancer), uremia, infections, collagen vascular diseases, or radiation.

All of the other choices, anxiety (choice A), myocardial infarction (choice C), senile aortic stenosis (choice D), and sepsis (choice E), may be associated with tachypnea, but are not usually direct causes of pulsus paradoxus.

*** A 5 y/o boy is brought to the clinic because of a fever for 5 days and a sore throat and malaise. The mother tells you that he is usually a very healthy child and he is up to date on all of his immunizations. Besides the mother, he lives at home with an older brother and sister, neither of them are sick. On P/E, T=39.5 C (103.1 F), he has a peeling rash on his extremities, one 2 cm lymph node on the right anterior cervical chain, a confluent truncal rash, and mild conjunctivitis. Appropriate management is taken. The most important long-term Mx of this child is: A. antibiotic prophylaxis to prevent rheumatic fever B. echocardiograms to look for coronary artery aneurysms C. excisional biopsy and surveillance of lymph nodes for malignancy D. nothing, as this is a case of scarlet fever and he will completely recover E. serial lumbar punctures The correct answer is B. The vignette demonstrates a case of Kawasaki disease which is characterized by high fever for >5 days, unilateral cervical lymph node, macular papular rash to truck, peeling of hands and feet, and conjunctivitis. The most important sequela of Kawasaki is the development of coronary aneurysms, and so children need to be monitored with EKGs and echocardiograms for at least 2-3 months after the acute illness. Aneurysms form from 7-45 days after illness onset. Streptococcal pharyngitis is characterized by erythema of the oropharynx, palatal petechiae, and cervical lymphadenopathy. There is no peeling, rash or conjunctivitis. Antibiotics are used for treatment and rheumatic fever prophylaxis (choice A). The enlarged lymph node is a reactive lymphadenopathy and need not be biopsied or excised (choice C). Upon proper treatment for KD, lymphadenopathy regresses. This is not a case of scarlet fever (choice D) which is characterized by exanthem and a fine, sandpapery rash, and is usually caused by Streptococcus. Although children with KD often present with a toxic appearance, lumbar punctures (choice E) is not the standard of care and need not be performed.

*** A 57-year-old woman with a long history of Crohn's disease comes to the office because of slowly progressive ataxia and paresthesias. She is currently off all medications. Physical examination shows a decreased vibratory and positional sense and mild ataxia. A CT of the head is unremarkable. Laboratory studies show:

You suspect B12 deficiency secondary to Crohn's disease. A Schilling test is performed. You expect the study to show: In order: Radiolabeled Vitamin B12 urinary excretion, Radiolabeled B12 excretion after intrinsic factor added , Radiolabeled B12 excretion after intrinsic factor + antibiotics A. Normal / Normal / Normal B. Low/ Normal/ Normal C. Low /Low/ Low D. Low /Low/ Normal E. Normal /Low/ Normal The correct answer is C. This patient likely has a lesion in her ileum from Crohn's disease. This is the area where B12 is preferentially absorbed. If there is a lesion in the ileum, the Schilling test will reveal no ability to absorb B12. An understanding of the Schilling test should facilitate answering this question. If B12 deficiency develops, a Schilling test can help to define why this happened. The first step is to saturate the body with B12 by giving a large IV dose of normal B12. Simply, this ensures that any additional B12 absorbed will be excreted in the urine. This patient does not lack intrinsic factor so adding IF will not improve B12 absorption. Additionally, this patient's problem is not the lack of dietary B12, so giving B12 alone could not correct the underlying problem.

*** You are asked to see a 3 wk/o infant in the ED with a 1-day history of fever. The parents measured his temperature because he "felt warm" to them and found a temperature of 38.3 C (101.0 F). He has been feeding normally, taking 2 ounces of formula every 3-4 hours. He had 6 wet diapers the previous day. Examination shows an

active infant with a temperature of 38.8 C (101.8 F). His skin perfusion is good and his physical examination, including examination of his tympanic membranes, is normal. There are no ill household contacts. The most appropriate next step is to A. discharge the patient with close outpatient follow up B. inquire about the mother's group B streptococcal status at delivery C. obtain the infant's vaccination history D. order a urinalysis and, if negative, do blood and CSF cultures E. send blood, urine, and CSF cultures and begin empiric intravenous antibiotic therapy The correct answer is E. There is absolutely no way to reliably distinguish a self-limited viral illness from sepsis or meningitis in an infant less than 4-weeks of age. Accordingly, all infants with a fever greater than 38 C (100.4 F) in this age group require full evaluation, including admission and parenteral antibiotics. The incidence of sepsis is somewhere between 5-10% with this degree of fever and can be catastrophic if missed, resulting in death or permanent neurologic disability. Discharging the patient (choice A) is inappropriate because well-appearing infants in this age group may still have a potentially lethal bacterial disease. At 3 weeks, this infant is at the peak incidence of late onset group B streptococcal disease, but the mother's group B streptococcal status (choice B) does not correlate well with occurrence of late onset group B streptococcal disease and so it is not relevant here. Maternal group B streptococcal screening is useful in the management of early-onset disease (disease within the first few days of life). At 3 weeks this infant is unlikely to have received any vaccinations (choice C), with the possible exception of hepatitis B vaccine. He would not in any case have received Haemophilus influenzae or conjugated Streptococcus pneumoniae vaccine, which are the only two that could potentially influence his susceptibility to sepsis or meningitis. Urinalysis (choice D) is not reliable in excluding urinary tract infection in very young infants, and therefore is not a good screening test to decide if further evaluation is necessary.

*** A 30 y/o woman is brought to the emergency department after a high-speed MVA in which she was an unrestrained driver. She was conscious at the scene with a Glasgow Score of 13. There were beer bottles evident in the car. On arrival at the emergency department, primary survey shows an abdominal bruise on the RUQ. Her vital signs are

stable and blood alcohol level is 2449 mg/dL. Her PMH is remarkable for alcoholic pancreatitis twice in the previous 3 years. The patient complains of marked right upper quadrant pain and has significant tenderness on palpation of her right upper and right lower quadrants. An abdominal CT scan shows pericholecystic fluid and possible gallbladder wall thickening. The most appropriate next step to diagnose possible gallbladder rupture is: A. diagnosis can be made by CT scan alone B. ERCP C. HIDA scan D. exploratory laparoscopy E. exploratory laparotomy The correct answer is D. This patient has suffered a traumatic injury to her right upper quadrant and there is a clinical suspicion of gallbladder rupture. The gold standard for such a diagnosis is direct visualization. Since laparoscopy is widely available, it has become the modality of choice to undertake direct visualization of the suspected injury. The diagnosis cannot be made by CT alone (choice A). The CT shows only pericholecystic fluid. This is consistent with chronic or acute cholecystitis, both of which are a possibility given her history of alcohol use and pancreatitis. The issue, given her trauma, is whether the gallbladder has ruptured. An ERCP (choice B) is an endoscopic diagnostic and therapeutic tool for the management of biliary disease. It has no role in the diagnosis of gallbladder rupture. It can however be used to demonstrate gallbladder filling with contrast after the removal of biliary stones. A HIDA scan (choice C) is an imaging modality utilizing radioactive tracers to visualize the gallbladder. However, given the widespread use of laparoscopic equipment, this test has been largely supplanted by the new, more sensitive diagnostic laparoscopy.

Exploratory laparotomy (choice E) has attendant intra- and postoperative complications that make it largely reserved for centers where laparoscopy is not available. It is also reserved for patients with extensive abdominal trauma or clinical signs and symptoms consistent with massive abdominal pathology where laparoscopy would not be useful or for patients that require a laparotomy for another indication.

*** A 64 y/o man comes to the clinic because of a "spot" on the side of his face that has been there for about 8 months. He says that he is an executive at a local company and is retiring at the end of the year. The company has hired a portrait artist to paint his picture that will hang in the boardroom for many years to come, and so he realized that this is a good time to "have this thing taken off." He thinks that the lesion has not grown since he noticed it, but he has not paid it much attention. He plays tennis every weekend at his country club and then lies in the sun with his wife. This is the first time you have seen this patient, but he tells you that he has been very healthy and has only suffered through "a couple of bouts of kidney stones" over the years. Physical examination shows a 2.3-cm waxy, verrucous, dark brown papule with a stuck-on appearance.The most likely diagnosis is A. actinic keratosis B. basal cell carcinoma C. dermatofibroma D. melanoma E. psoriasis F. seborrheic keratosis G. squamous cell carcinoma The correct answer is F. This patient most likely has seborrheic keratosis. This lesion is characterized by light brown to black papules or plaques with an adherent waxy, greasy scale. The "stuck-on" appearance is very characteristic. It is most often found on the face and trunk.

A 35 y/o white female is brought to your office after a brief loss of consciousness. No tonic-clonic activity was observed, nor did the patient experience urinary incontinence. She recalls that about 10 seconds prior to blacking out she felt nauseated, began sweating, and became pale. When she recovered, she had no memory loss or confusion. Her physical examination, including a thorough neurologic examination, is unremarkable. A CBC, blood profile, and EKG are all within normal limits. Appropriate management at this time would include a.an EEG b.24-hour Holter monitoring c.echocardiography d.a CT scan of the head e.reassurance

e- Patients with vasovagal syncope often present with prodromal symptoms such as nausea, diaphoresis, pallor, and lightheadedness. This often occurs in stressful or frightening situations. These premonitory symptoms often occur before the loss of consciousness. A quick recovery of mental function after an episode of loss of consciousness makes it much more likely that this was a syncopal episode rather than a seizure, as seizures are often followed by a postictal period of residual confusion. A patient does not always have to experience tonic-clonic movements for the diagnosis to be a seizure disorder, since brief tonic-clonic movements can be seen in some syncopal episodes. However, patients who experience an aura or incontinence of urine or feces often have had a seizure.

For a pregnant woman in the 26th week of gestation who tested positive for Streptococcus B you should start the treatment: a.right away b.one week before delivery c.24 hours before delivery d.during delivery d- Physicians who culture for GBS carriage during prenatal visits should do so late in pregnancy (35-37 weeks gestation); cultures collected earlier do not accurately predict whether a mother will have GBS at delivery.

You diagnose acute pancreatitis in a 45-year-old white male. As you initiate therapy and closely monitor his progress, which one of the following complications is most likely to develop? a.Hypomagnesemia b.Hypoglycemia c.Hypercalcemia d.Acute hypertension a- Treatment strategies for pancreatitis vary somewhat and are related to the degree of inflammation. In patients with mild pancreatitis, a therapeutic regimen of avoidance of oral intake, intravenous hydration, and analgesia usually suffices. However, patients with more severe pancreatitis are likely to develop significant complications, and require closer monitoring. Patients with hypotension and vascular instability frequently require massive fluid resuscitation. Patients with metabolic complications such as hyperglycemia, hypocalcemia, or hypomagnesemia may require insulin, calcium, or magnesium supplementation.

Immediate gastric lavage is contraindicated in treating acute ingestion of which one of the following? A.Salicylates B.Strychnine C.Ethanol D.Acetaminophen E.Phenothiazines b- Ten to 30 minutes after ingestion of strychnine, untoward symptoms begin. Often without any warning the patient falls into violent convulsions. Gastric lavage is postponed until treatment designed to prevent the convulsions is started. Initiation of gastric lavage as soon as possible is indicated in treating poisoning by salicylates, ethanol, acetaminophen, and phenothiazines.

Which one of the following depressed patients is most likely to commit suicide? a.A 26-year-old male who repeatedly denies any thoughts of suicide b.A 30-year-old female who has been hospitalized overnight on several occasions for attempted suicide c.A 50-year-old recently divorced alcoholic male who feels life is hopeless d.A 50-year-old female who thinks of suicide and fears she might act on her thoughts c- Assessment of suicidal risk is critical in determining the need for and duration of hospitalization of depressed patients. Most suicides are planned, not impulsive, and carried out successfully most often by the elderly, males, those in poor health, alcoholics, schizophrenics, those who have recently lost a loved one (especially a mate), and those suffering from depressive disorders. Many depressed patients think about suicide, and a physician should take these patients seriously; however, among this group of patients, the 50-year-old recently divorced alcoholic male has the highest risk of successful suicide.

Which disease could be with higher BP in lower extremity than in upper extremity? TAKAYASO: High in the lower

A 70-year-old former shipyard worker, who smokes one pack of cigarettes daily, notes progressive weight loss and debility over a period of 6 months. Over a period of 1 month, he develops right chest pain and a nonproductive cough. A chest x-ray reveals extensive pleural thickening, pleural effusion, and hilar adenopathy. A pleural biopsy confirms mesothelioma. Which of the following is a favorable prognostic factor in this patient? a. Male sex b. Extent of disease at diagnosis c. Good performance status d. Pain as presenting symptom

e. Age over 65 f. None of the above f- All of the clinical components noted exhibit very poor prognostic factors in the patient with mesothelioma, a particularly aggressive disease that remains unresponsive to many forms of aggressive therapy.

Which one of the following has proven most useful for breast cancer screening in women who have had silicone breast implants? a.Thermography b.Ultrasonography c.Mammography d.Magnetic resonance imaging c- Screening by conventional film-screen mammography, supplemented by the displaced or Eklund view, is the recommended method for breast cancer screening in women with breast implants. The displaced view draws the breast forward while placing the implant posteriorly, increasing the amount of breast tissue visualized. None of the other modalities has proven useful for screening asymptomatic women for breast cancer.

A 55-year-old woman has had profuse watery diarrhea for 3 months. Laboratory studies of fecal water show the following: Sodium: 39 mmol/L Potassium: 96 mmol/L Chloride: 15 mmol/L Bicarbonate: 40 mmol/L Osmolality: 270 mosmol/kg H2O (serum osmolality: 280 mosmol/kg H2O) The most likely diagnosis is A villous adenoma B lactose intolerance C laxative abuse D pancreatic insufficiency E nontropical sprue The answer is A In the case described, the osmolality of fecal water is approximately equal to serum osmolality. Furthermore, there is no osmotic "gap" in the fecal water; the osmolality of the fecal water can be accounted for by the stool electrolyte composition: {2 [(Na+) + (K+)]} = [2 (39 + 96)] = 270. A villous adenoma of the colon typically produces a secretory diarrhea. Lactose intolerance, nontropical sprue, and excessive use of milk of magnesia produce osmotic diarrheas with osmotic "gaps" caused by lactose,

carbohydrates, and magnesium, respectively. Pancreatic insufficiency causes steatorrhea, not watery diarrhea.

A 60-year-old white female is scheduled to have a total abdominal hysterectomy. She is currently in good health, but the general surgeon is concerned because the patient had a pulmonary embolus 10 years ago.Which one of the following is most effective for prevention of another embolus? a.No prophylaxis necessary b.Impedance plethysmography, 36 and 72 hours after surgery c.Aspirin prophylaxis d.Full heparinization after surgery e.Subcutaneous heparin prophylaxis e- This patient is considered at high risk for a venous thromboembolism because of general surgery, age greater than 40, and previous history of a pulmonary embolus. In numerous clinical trials, heparin, 5000 U subcutaneously 2 hours prior to surgery, followed by 5000 U subcutaneously every 8 to 12 hours until the patient is ambulatory, has statistically reduced the incidence of deep vein thrombosis. Full heparin therapy is not necessary. Aspirin is not effective, and impedance plethysmography would not prevent thrombosis.

An obviously intoxicated 50-year-old white male is brought to the emergency department after the car he was driving hit a telephone pole. He has a fracture of the femur, and is confused and uncooperative. He is hemodynamically unstable. Initial physical examination of his abdomen does not indicate significant intra-abdominal injury. Which one of the following would be best for determining whether laparotomy is needed? a.Peritoneal lavage b.Contrast duodenography c.Ultrasonography of the abdomen d.An MRI scan of the abdomen e.A CT scan of the abdomen a- Physical examination of the abdomen is often unreliable for detecting significant intraabdominal injury, especially in the head-injured or intoxicated patient. In a hemodynamically unstable patient with a high-risk mechanism of injury and altered mental status, peritoneal lavage is the quickest, most reliable modality to determine whether there is a concomitant intra-abdominal injury requiring laparotomy. Computed tomography of the abdomen and contrast duodenography may complement lavage in stable patients with negative or equivocal lavage results, but in an unstable or uncooperative patient these studies are too time-consuming or require ill-advised sedation. Ultrasonography may also complement lavage in selected patients, but its

usefulness is limited in the acute situation. Magnetic resonance imaging is extremely accurate for the anatomic definition of structural injury, but logistics limit its practical application in acute abdominal trauma.

A 67-year-old white male retired factory worker was found to have an abdominal aortic aneurysm on routine physical examination. A vascular surgeon has recommended operative repair. The patient is a former smoker, with a 10-pack-year smoking history. Three years ago, while under the stress of a forced retirement, he had angina which responded well to sublingual nitroglycerin. He has had no angina for 1 year. Which one of the following conditions would be the most likely cause of perioperative death in this patient? a.Pulmonary complications b.Peripheral embolization c.Operative site infection d.Myocardial infarction e.Cerebral infarction Answer is d. Myocardial infarction is the major cause of perioperative death in patients undergoing surgery for abdominal aortic aneurysm. This is especially true of patients with a history of known coronary artery disease or with EKG abnormalities, and it is true even if the coronary artery disease is stable. Cerebral infarction occurs infrequently, even though cerebral vascular disease is commonly present in this patient group. Operative site infection, peripheral embolization, and pulmonary complications are all significant postoperative complications which may cause death, but they are less frequent causes of death than myocardial infarction.

Which one of the following is true of a pregnant patient who has diabetes mellitus? a. Adjustments in hypoglycemic medications are best made by following urine glucose readings b. Oral hypoglycemic agents are useful during pregnancy in patients with mild diabetes mellitus c. A precise knowledge of fetal age is important to a successful outcome for the fetus d. Because the fetal pancreas helps control the diabetes, ketoacidosis is less likely during pregnancy c- Ideally, the child of an overtly diabetic woman should be delivered close to term. Precise knowledge of fetal age (by menstrual history, accurate measurements of uterine height during the second trimester, and confirmation by sonography) is very important to a successful outcome for the fetus.

Severe growth retardation is diagnosed in the fetus of a 19 y/o unmarried white female at 36 wks gestation. The Dx is based on biparietal diameter and there is scant amniotic fluid.Which one of the following is the most appropriate management? a. Perform serial L/S ratios until greater than 3.0, followed by prompt delivery b. Induce labor, with careful fetal monitoring c. Perform an immediate C/S d. Follow the mother weekly with serial ultrasounds e. Follow the mother weekly with NST b- Appropriate management of the preterm infant who is severely growth retarded depends on several factors. Generally those near term should be delivered promptly. By the time that growth retardation has become severe, the fetus is usually mature enough to survive if delivered promptly. However, the fetus must be monitored carefully during labor, with facilities for immediate cesarean section if there is deterioration, and the neonate must receive excellent neonatal care beginning immediately after delivery.

A 53 y/o woman presents with a long Hx of dyspepsia, fullness, and belching. An endoscopy shows severe antral erythema and a small hiatal hernia, but no ulcer. Because of her long history of dyspepsia, a serum gastrin test and serum IgG test for Helicobacter pylori are done. The serum gastrin test result is 1800 pg/mL and the test for H. pylori is negative. The next test that should be done in this patient is: a- push enteroscopy b- secretin stimulation test c- gastric acid level messurement d- pancreas angiography e- endoscopic US answer is c. the finding of a high serum gastrin concentration is not infrequently a reason for referral for tests to diagnose Zollinger-Ellison syndrome, she does not have any of the features of the ZE, (recurrent multiple peptic ulcers and/or diarrhea). The first test that should be done is to assess whether the patient has hyperchlorhydria or hypochlorhydria by inserting a NGT and measuring basal acid secretion. However, some of the highest serum gastrin concentrations are found in patients with achlorhydria or hypochlorhydria due to gastric atrophy (for example, in pernicious anemia or after the prolonged use of antisecretory drugs, particularly proton-pump inhibitors).

A 42 y/o female with Hx of 3 spontaneous abortions presented with pain in the left leg, which later on Dx of DVT was confirmed. Which of the following findings is most likely in the laboratory results of this patient? A - Polycytemia B - Thrombocytopenia C - Low white blood cell count D - Hyponatremia E Hyperkalemia b- In this patient with confirmed DVT and remarkable Hx of the spontaneous abortions, is highly suggestive of antiphospholipid antibody syndrome (antibodies directed against either phospholipids or plasma proteins bound to anionic phospholipids). Most common symptoms are venous and arterial thrombosis, recurrent fetal losses, and thrombocytopenia. Other possible findings include livedo reticularis, migraine headaches, Raynauds disease, hemolytic anemia, neurologic dysfunction, renal disease, pulmonary hypertension, avascular necrosis, and adrenal insufficiency.

A 53 y/o male smoker presented with a 2 wk Hx of expectorating sputum streaked with a blood. He denies any fever or chills or increases in the intensity of his cough. P/E and CXR are unremarkable. Which of the following is an appropriate next step in the diagnostic work-up of this patient? A - Fiberoptic bronchoscopy B - MRI of the chest C - High resolution CT of the chest ( HRCT ) D - A and B E - A and C Answer is E. Fiberoptic bronchoscopy and HRCT are complementary to each other. HRCT can demonstrate all tumors seen on bronchoscopy as well as several which are beyond bronchoscopic range. On the other hand, HRCT could not detect bronchitis or subtle mucosal abnormalities that could be seen on bronchoscopy. Also, HRCT is particularly useful in diagnosing bronchiectasis and aspergillomas, while bronchoscopy is diagnostic of bronchitis and mucosal lesions such as Kaposis sarcoma. The patient in question is at high risk for pulmonary carcinoma; as of today, the procedures are considered complementary in this setting.

A patient with cytomegalovirus retinitis has been treated with IVganciclovir for the last 4 months. On the most recent lab findings a sudden drop in the plt count was noted (20,000 mm3). Which of the following is the most appropriate action in this situation?

A - Discontinuation of the therapy B - Exchange of ganciclovir with intravenous foscarnet C - Exchange of ganciclovir with acyclovir D - Exchange of ganciclovir with valcyclovir E - Continuation of ganciclovir therapy Answer is B. The major drugs that are used for treatment of CMV retinitis are IVganciclovir and foscarnet, oral ganciclovir, intraocular ganciclovir, and IV cidofovir. Ganciclovir and foscarnet have equivalent efficacy against the retinitis. Major side effects of the ganciclovir are neutropenia and thrombocytopenia (limiting use in up to 16% of patients). Ganciclovir should not be given with absolute neutropenia of less than 500 mm3 and thrombocytopenia 25, 000 mm3.

A 76 y/o man is hospitalized with stroke in the area of the right middle cerebral artery. He had a paroxysm of cough immediately after attempt to eat.On P/E he is alert and oriented to time, place, and person. He has severe dysarthria but no signs of aphasia. He has facial asymmetry due to Lt sided facial droop, but his gag reflex is intact. Which of the following is the most appropriate way to provide nutrition to this patient? A - Placement of a percutaneous gastrostomy tube B IV alimentation C - Feeding through a NGT D - Oral feeding supervised by a nurse and suctioning as needed E - Clear liquid diet with advanced diet as soon as possible depending on patients clinical status Answer is B. After a stroke about 25-45% of all patients develop dysphagia. The main problem that stems from dysphagia is aspiration pneumonia, which, if it develops, greatly complicates the clinical course and contributes to mortality. The patient in question had an attack of cough after an attempt to eat. This is a common sign of dysphagia. Physical examination of this patient revealed several findings that suggest dysphagia (facial nerve paresis and dysarthria). It is a common misconception that presence or absence of a gag reflex correlates with the risk of aspiration. This is not true. More important in the assessment of the aspiration risk are speech articulation, ability to swallow, and tongue movement.

A 16 y/o male in good health presents to you for a school physical. His family hx reveals that his father died of colon cancer at age 37. Your physical exam reveals a healthy young man with several lipomas on his back and legs and a nodule on his jaw. You should: a. Suggest colonoscopy at age 30. b. Suggest colonoscopy now. c. Suggest flexible sigmoidoscopy and barium enema at age 30.

d. Suggest flexible sigmoidoscopy and barium enema now. e. Biopsy his jaw lesion. Answer is D. case of Gardners Syndrome.

An elderly male complains of severe muscle weakness in his thigh muscles and proximal arm muscles, although this weakness is mild. He states that his weakness is worse in the morning immediately after getting out of bed and improves during the day. On P/E, it is apparent that muscle strength increases with repetition of the grip strength test and later diminishes. Which of the following tests should be included in the workup of this patient? A - CXR B - Tensilon test C - Abdominal CT D - Colonoscopy E Cystoscopy Answer is A. Symptoms in this man are consistent with Eaton-Lambert syndrome. This syndrome shares the same pathologic site with myasthenia gravis (the neuromuscular junction) and has a similar path physiology (an autoimmune disease). Eaton-Lambert syndrome is usually associated with malignancy. Up to 70% of these patients have associated small cell lung cancer, and this disease must be ruled out in every patient with presenting symptoms of Eaton-Lambert syndrome. Clinical presentation of the Eaton-Lambert syndrome includes weakness that is typically seen early in the hip girdle, making it difficult for the patient to rise from a chair or to climb the stairs. Less dramatic is shoulder girdle weakness. Involvement of the bulbar muscles or diplopia is rare, but ptosis is frequently seen. Symptoms are also likely to be more prominent in the morning; autonomic dysfunction may cause erectile dysfunction and dry mouth. Differentiation from myasthenia gravis may be hard since patients with Eaton-Lambert syndrome have a positive AChR-Ab test in 13% of cases.

A 50 y/o man presents to the ER complaining of 6 hours of severe diarrhea. He has had eight to ten liquid stools, without blood. He has diffuse cramping abdominal pain alleviated by each bowel movement, but he denies tenesmus. He has mild untreated hypertension but no other medical problems. He denies recent travel. Other family members have not been ill. Vital signs: PR= 94/min, BP= 148/96 mm Hg, no changes with standing, T= 37 C (98.6 F). The P/E is normal. The best initial approach to this patient is: (A) Obtain stool specimens for culture, examination for fecal leukocytes, and examination for ova and parasites

(B) Empiric antidiarrheal therapy (C) Empiric antibiotic therapy with trimethoprimsulfamethoxazole or ciprofloxacin (D) Reassurance and oral fluids (E) Flexible sigmoidoscopy with mucosal biopsy Answer is D. This patient has an acute diarrheal illness without evidence of severe inflammation, dehydration, or other risk factors. The great majority of acute diarrheal illnesses are infectious (viral or bacterial) and self-limited. Oral hydration and reassurance are the most appropriate care for such patients. If the diarrhea persists or if the patient develops a more toxic clinical picture, additional diagnostic tests would be indicated. At the time of presentation, stool studies are not helpful. Empiric therapy with antibiotics is not indicated. Flexible sigmoidoscopy is indicated only in persistent or inflammatory diarrhea.

A 9 y/o girl is brought to the clinic because she has felt "sick" and has been unable to go to school for the past 2 days. She complains of a headache, congestion, rhinorrhea, and double vision. Her PMH is remarkable for recurrent otitis media 2 years ago for which she eventually received bilateral myringotomy tubes. She lives at home with her mother and grandmother who are both cigarette smokers. T=38.8 C (101.8 F), PR=120/min, RR=20/min, and visual acuity is 20/20 in both eyes. P/E shows tympanic membranes with evidence of previous surgery, but are otherwise normal, an erythematous oropharnyx with exudation, and slight exophthalmos of the left eye. On the left, the ocular examination also demonstrates periorbital edema, injection of the conjunctiva, trace restriction of extraocular movements, and an afferent pupillary defect. The right eye is normal. The rest of her physical exam is unremarkable. Whats the best next step in Mx? A. A CT of the head B. Gentamycin eye drops treat most cases, but patients need close observation C. Hospitalization and IV antibiotics are indicated D. Oral antibiotics are the first line of treatment The correct answer is C. This patient has orbital cellulitis. Patients present with lethargy, fevers, signs and symptoms consistent with sinusitis, proptosis of the globe, restriction of extraocular movements, periorbital edema, injected conjunctiva, and afferent pupillary defects. Hospitalization and IV therapy should be started at once as this condition is potentially fatal, especially if the cavernous sinus becomes involved.

A 37 y/o woman with a history of bipolar disorder is brought to the ER by ambulance obtunded and ataxic. The patient has not been hospitalized in over 5 years and has been stably maintained on outpatient therapy with a regimen of lithium carbonate 600 mg by mouth twice a day. While you are examining her, she has a tonic-clonic seizure lasting approximately 30 seconds. As part of the laboratory work-up, the patient's lithium level is found to be 4.2 mEq/L. After basic airway and cardiovascular support, the most appropriate next step in this patient's management is to: A. administer a loading dose of phenytoin B. get a renal consultation for emergent dialysis C. order an MRI of the patient's head D. provide conservative management including fluid and electrolyte replacement E. send liver function tests The correct answer is B. The patient presents to the ED with potentially life-threatening lithium toxicity. The treatment of choice for lithium levels greater than 4.0 mEq/L is hemodialysis.

A 16 y/o girl is brought to see you by her mother, who is a nurse in your office. The patient is hoping to play for the school volleyball team and needs a clearance form signed by a doctor. Her past medical history includes exercise-induced asthma and allergic rhinitis for which she takes albuterol and loratadine. During the interview with the patient, while you are speaking with her alone, she reveals that she has been sexually active with her boyfriend for the last 2 years. She has not yet discussed this fact with her parents. Her physical examination, including pelvic exam, is normal. At this time the patient should be screened for : A. chlamydia B. gonorrhea C. hepatitis B D. HIV E. herpes simplex F. syphilis

The correct answer is A. Routine screening for chlamydia in all sexually active women aged 25 and younger, as well as in asymptomatic women older than 25 who are at high risk is recommended.

An 18 y/o boy is brought to the ED by ambulance. He and his friends were ice-skating on a frozen pond when the ice broke and the young man became submersed in cold water. His friends admit to drinking alcohol prior to the incident. The patient's rectal T=33 C (91.4 F), PR= 57/min & weak, BP= 80/30 mm Hg, and RR= 7/min. P/E reveals a comatose young man. An EKG is most likely to show A. delta wave B. diffuse broadening of T waves C. prominent U waves D. shortened QT intervals E. upward deflection following the S wave The correct answer is E. Hypothermia prolongs repolarization causing an upward deflection following the S wave (distinctive convex elevation of the J point), which is usually called an Osborn wave (or J wave of Osborn).

A 71 y/o woman is brought to the physician by her distressed daughter. The daughter relates that, 3 days ago, her mother began to complain of RUQ abdominal pain and then she complained of chills, nausea, and some vomiting. On P/E reveals an obtunded, hypotensive, and obviously very sick elderly woman. She has impressive pain to deep palpation in theRUQ, along with muscle guarding and rebound. T= 40 C (104 F). Lab results : WBC= 22,000/mm3 with multiple immature forms, bilirubin= 5 mg/dL and Alk Ph= 840 U/L. serum amylase= normal. An emergency sonogram shows multiple stones in the gallbladder, normal thickness of the gallbladder wall without pericholecystic fluid, dilated intrahepatic ducts, and common duct with a diameter of 2.1 cm. The sonographer cannot identify stones in the common duct. In addition to IV fluids and antibiotics, which of the following is the most appropriate next step in management? a- Elective cholecystectomy b- Emergency decompression of the common duct c- Emergency cholecystectomy

d- Emergency surgical exploration of the common duct e- Emergency transhepatic cholecystostomy The correct answer is B. The Dx is acute ascending cholangitis. This deadly disease is seen in patients with longstanding gallstones who get one or more stones in the common duct, where they produce partial obstruction that allows ascending infection. The chills, very high fever, and extremely elevated alkaline phosphatase are diagnostic. The key component of therapy is immediate decompression of the common duct, which is full of pus. How it is achieved is less important. ERCP is usually the first choice, but it can be done by PTC or by open surgery.

33 y/o woman G3P2, with Hx of two preterm deliveries, comes at 32 wks gestation with contraction every 4 min & 3 cm cervical dilation. A tocolytic drug administered which raised her serum Glc from 95 to 170. What was the pharmacologic agent ? a- Terbutaline b- Magnesium sulfate c- Indomethacine d- Nifedipine e- Meperidine hydrochloride The answer is a- Terbutaline. Its a beta 2 agonist which relaxes the smooth muscle of uterus and its SE is raising blood Glc through glycogenolysis.

An febrile 53 y/o woman developes hypotension, tacchycardia & oliguria 24 hrs after abdominal cholecystectomy. Her skin is cold & clammy. Which of the ffg is most responsible for her symptoms? a- Gram (-) sepsis b- Hemoperitoneum c- Acute MI d- Pulmonary embolus e- Pneumothorax Answer is b- hemoperitoneum. She shows classic symptoms of hypovulemic shock.

A 6 month old child falls and hits his head. Which of the following signs would be the MOST indicative of serious neurological injury? a- The parent states the child was pale and sweaty for a few minutes after the fall b- A single post-fall episode of emesis c- Lethargy immediately after the head injury

d- Hypotension e-A single grand mal seizure immediately after the fall Infrequently, infants become hypotensive from blood loss into either the subgaleal or epidural space after head trauma. Hypovolemia can occur because of open cranial sutures and fontanelles. Transient paleness, lethargy, diaphoresis, and emesis are common after minor head trauma and do not necessarily signify significant neurological injury. Persistence of any of these signs or symptoms, or change in mental status is concerning. Seizures may occur shortly after head injury and are usually self-limited. However, about 50 percent of patients with posttraumatic seizures have positive findings on head computed tomography (CT). Children with two or more seizures or a GCS < 8 should be strongly considered for anticonvulsant therapy.

Which of the following drugs is NOT associated with acute pancreatitis? (A) Heparin (B) Furosemide (C) Rifampin (D) Salicylates (E) Warfarin The answer is A. Drugs and toxins are major causes of acute pancreatitis. Some of the medications associated with the occurrence of acute pancreatitis are oral contraceptives, estrogens, phenformin, glucocorticoids, rifampin, tetracycline, isoniazid, thiazide diuretics, furosemide, salicylates, indomethacin, calcium, warfarin, and acetaminophen. Other etiologic factors contributing to the development of pancreatitis include infection, collagen vascular disease, metabolic disturbances, and trauma.

An 11 y/o boy is rushed into the ED following a MVA. The patient was a restrained, front seat passenger, when an oncoming car hit the passenger side of the car. The boy denies loss of consciousness, but in the hospital he complains of pain over his right side. His BP=118/59, PR=139/min, and RR=24/min. On P/E, he has decreased breath sounds over the right base, and there is ecchymosis over the right flank. His abdomen is soft with tenderness in the RUQ. Appropriate Mx of his airway with neck stabilization is provided and he is resuscitated appropriately. Imaging studies of his neck are negative and a CXR does not show pneumothorax or rib fractures. Dipstick of spontaneously voided urine is positive for blood. U/A confirms the presence of 50 RBC/hpf. In regards to his hematuria the next most appropriate course of action is to: A. begin empiric antibiotic therapy B. obtain a CT scan of abdomen and pelvis C. order a renal/bladder ultrasound

D. perform a retrograde urethrogram E. place a Foley catheter F. repeat the urinalysis G. schedule an outpatient intravenous pyelogram (IVP) The correct answer is B. obtain a CT scan of abdomen and pelvis. The management of hematuria associated with trauma differs in adults and children. In the adult population, imaging is performed only in those patients with gross hematuria or microscopic hematuria plus hypotension. This differs from the pediatric patient. In children, any degree of hematuria (gross or microscopic) should be investigated with imaging studies. One reason for this discrepancy is that large amounts of catecholamines released in injured children may sustain blood pressure in the face of hypovolemia. A CT scan is the most useful imaging modality in this setting. A CT scan is noninvasive, accurate and fast, and it can help in assessing the size and extent of retroperitoneal hematomas and renal parenchymal trauma. Not only does this child have microscopic hematuria (an indication by itself to perform imaging studies), but he also has signs, (flank ecchymosis and tenderness), that raise the suspicion of renal injury. High suspicion for renal injury (i.e., rib fracture, flank contusion, deceleration injury) is another indication for perform imaging studies.

*** A 45 y/o woman presents with acute pancreatitis with a serum amylase concentration

of 2010 U/L, T=37.2 C (99.0 F), the leukocyte count is 12,000/mL; plasma Glc=250 mg/dL; and serum ca=8.4 mg/dL. She recovers after 1 wk of supportive therapy. No obvious cause for the pancreatitis is found; ultrasonography is normal, and the fasting serum triglycerides are 350 mg/dL. The best course of action to diagnose the cause of the pancreatitis is: (A) ERCP (B) No further diagnostic testing until another attack occurs (C) CT scan with contrast (D) Measurement of serum CA 19-9 (E) MRI of the pancreas B- No further diagnostic testing until another attack occurs The best course of action in a patient with a first attack of relatively mild pancreatitis even with a high serum amylase concentration is to see whether further attacks occur before continuing invasive investigations. The presumption is either that the patient has passed a small single gallstone or that the cause has not yet been identified, that is, it is

idiopathic pancreatitis. The latter is often a once-only disease, and therefore no further investigations are warranted at this time. An ERCP and CT scan or MRI would be indicated after recurrent mild attacks or two severe attacks to try to establish a diagnosis, particularly common duct stones or pancreas divisum. Measurement of CA 19-9 is not indicated. Although a full lipid workup in this patient would be good medicine and should be done in any case (and may also be an indirect evidence of gallstones), it is unlikely to be the cause of pancreatitis. Hyperlipidemic pancreatitis is unusual if lipid levels are less than 600 mg/dL. After an initial severe attack, some authorities would do a repeat ultrasound within 3 to 6 months just to exclude gallstones.

***1- An African- American couple with no known family hx of sickle cell dis., want to know what the chance is that their child have sickle cell dis. To obtain a more accurate assessment, RBC electrophoresis of the couple was done. The male has no HbS, but the female has a clear HbS band. Knowing that the carrier rate among African-Americans is approximately 8%, or 1 in 12, the chance that their child will have sickle cell dis. Is most likely which of the following? a- 0 b- 1/144 c- 1/288 d- 1/576 e- 1/1936

2- If there was no information about the parent electrophoresis, what would be the chance of the child having the dis.? a- 0 b- 1/144 c- 1/288 d- 1/576 e- 1/1936 1- a- 0, 2- d- 1/576 Px with sickle cell dis., are homozygous for the HbS. Because only one potential parent has the HbS allel, the only way their child could inherit two HbS genes is if the fathers sperm underwent a new mutation, which is an extremely unlikely event. Therefore, their child does not have a chance of inheriting the dis. Howerever, theres a chance of that their child would be a carrier, having only the sickle cell trait. With no information, the odds that two carriers will meet is 1/12 x 1/12= 1/144, and the odds that two carriers will have an infected child is , so, the odds that two Africaamericans will have a child with dis. is 1/144 x = 1/ 576

***A 28 y/o woman, G4P4, at 28 wks gestation, presents for prenatal visit. She says that she has not felt the baby move for 2 days. She has a Hx of HTN, which shes being treated with alfa methyl dopa, BID. On P/E, BP= 145/85, fundal height at 30 cm, and babys in transverse position. No FHR was detected with Doppler feotoscope. Whats the next step of Mx ? a- Perform a NST b- Perform an amniocentesis c- Obtain real-time US for cardiac motion d- Obtain a maternal abd. x-ray for fetal assessment e- Obtain a quantitative beta HCG f- Observation and check up in 2 wks Answer is c- Obtain real-time US for cardiac motion Its the method of choice for fetal death.

*** A 9 y/o boy presents with confusion & decreased school performance, he soon developes a spastic gait with dysarthria, dysphagia & visual loss. MRI shows massive white matter demyelination in the post. hemisphere. Whats the most likely Dx? a- MS b- Metachromic leukodystrophy c- Adrenoleukodystrophy d- Subacute sclerosing panencephalitis e- CJD Answer is c- adrenoleukodystrophy which is an x-linked dis. that affects young males, usually begin at 7-8 yrs of age. Its characterized by demyelination of the CNS, adrenal insufficiency, mental deterioration, aphasia, apraxia & dysarthria. 1/3 of patients have loss of vision also.

*** An elderly Px developes painful swelling of Rt. parotid gland 10 days after cholecystectomy. This is most likely secondary to which of the ffg ? a- Staph. Aureus b- Duct obstruction by a stone c- A viral infection d- Hemorrhage e- An immunologic reaction Answer is a- staph aureus. The px most likely has acute surgical parotitis ( sialadenitis ), which usually occur 1 wk post op in elderly patients who have poor dental hygiene and have intubated. Staph aureus is the most common organism. Tx is surgical drainage & antibiotics.

Which of the ffg can confirm the Dx of gastroparesis? (A) Endoscopy (B) Upper gastrointestinal series (C) Ultrasonography (D) Nuclear medicine scintigraphy (E) Electrogastrography Answer is D- nuclear medicine scintigeraphy There are no specific signs and symptoms of delayed gastric emptying, and when this disorder is suspected, the diagnosis of gastroparesis can usually be made by various diagnostic methods. The presence of retained food in a dilated stomach at the time of upper gastrointestinal contrast studies or detected endoscopically is indicative of underlying motor dysfunction when mechanical obstruction has been excluded. However, some patients with decreased motility of the stomach may not exhibit retained food. Ultrasonography has been used to determine the emptying rate for liquids and appears to be relatively accurate, but gastric scintigraphy of either solid or liquid test meals labelled with a radiopharmaceutical has proven to be the most accurate method of detecting underlying gastric motility abnormalities. Gastric scintigraphy supplies a quantitative result and can be used to evaluate the response to therapy. Electrogastrography is a research tool and its clinical application is unclear.

A 53 y/o alcoholic man with known cirrhosis presents with recent onset of abdominal swelling and fever. On P/E PR=92/min, BP=98/62 mm Hg, T=38.5 C (101.3 F), palmar erythema, spider angiomata, moderate abdominal swelling with bulging flanks, and a liver span of 8 cm are noted. There is no evidence of asterixis. Lab results: ALT=55 U/L, AST=62 U/L, serum BR=1.3 mg/dl, serum total pr.=7.3 g/dl, serum Alb=3.8 g/dl Paracentesis reveals 25 mL of straw-colored fluid. Ascitic fluid albumin is 2.9 g/dL; the fluid contains 1000 RBC and 960 leukocytes/microliter, with 90% lymphocytes and 10% PMN. The fluid is negative for amylase, and Gram stain shows no organisms.The most appropriate next intervention is:

(A) Cefotaxime, 1 g every 6 hours intravenously (B) Laparoscopy with biopsy and culture of peritoneal lesions (C) CT scan (D) Norfloxacin for prophylaxis of spontaneous bacterial peritonitis (E) Exploratory laparotomy Answer is B- laparascopy ewith Bx & culture of peritoneal lesions This patient has evidence of chronic liver disease on physical examination. The recent onset of ascites is complicated by fever and an elevated number of ascitic fluid leukocytes with a lymphocyte predominance. The ascitic fluid lymphocytosis is typical of tuberculous peritonitis, peritoneal carcinomatosis, or chylous ascites. In spontaneous bacterial peritonitis, the absolute polymorphonuclear cell count is >250/L and represents over 70% of the total leukocytes. This patient is at increased risk for tuberculous peritonitis because of the debilitating conditions of alcoholism and cirrhosis. The most sensitive procedure for diagnosing suspected tuberculous peritonitis is laparoscopy with biopsy for culture and histologic examination of peritoneal lesions. This does not require surgical laparotomy. Diagnosis to the underlying cause of fever and ascitic fluid lymphocytosis should take precedence over initiation of diuretic treatment. Cefotaxime therapy is appropriate treatment for spontaneous bacterial peritonitis, which is usually caused by enteric gram-negative organisms or streptococci. Aminoglycosides are to be avoided in these patients because of the risk for precipitating or exacerbating renal insufficiency. After an initial episode of spontaneous bacterial peritonitis, prophylaxis with oral antibiotics that provide selective intestinal decontamination, such as norfloxacin, ciprofloxin, or trimethoprim sulfamethoxazole, will considerably reduce the incidence of recurrent peritonitis. CT scan is insensitive for diagnosis of infection and is most useful for detecting malignancy, which is not likely in this patient. CMDT: Fever may suggest infected peritoneal fluid, including bacterial peritonitis (spontaneous or secondary). In immigrants, immunocompromised hosts, or severely malnourished alcoholics, tuberculous peritonitis should be considered. The accuracy of the SAAG exceeds 95% in classifying ascites. It should be recognized, however, that approximately 4% of patients have "mixed ascites," ie, underlying cirrhosis with portal hypertension complicated by a second cause for ascites formation (such as malignancy or tuberculosis). Thus, a high SAAG is indicative of portal hypertension but does not exclude concomitant malignancy. In Western societies, half of patients have underlying cirrhosis and ascites from portal hypertension. In such patients, a diagnosis of tuberculous peritonitis may go unsuspected

because symptoms are attributed to the underlying liver disease. A high index of suspicion is required for prompt diagnosis and treatment. http://www.sma.org/smj1999/aprsmj99/ahmad.pdf

A LBW infant at 2 months of age is brought because he has not benn eating well. On P/E, hes pale & tacchycardic, The lungs are clear and theres no hepatosplenomegaly. CBC shows Hgb= 6 mg/dl, which of the ffg is the most likely cause of anemia in this infant? abcdeMegaloblastic Sickle cell dis. Anemia of prematurity Alfa thalassemia Homozygous beta thalassemia

Answer is c- anemia of prematurity, which occurs in LBW infants 1-3 months after birth. Hgb is below 7-10 gr/dl.

You are called to see a 29 y/o woman, who was admitted to the hospital 24 hrs earlier because of a stiff neck and a temperature of 39.7 C (103.4 F). She is presumed to have meningitis, but the results of the CSF analysis are still pending. An LP was performed when she arrived at the hospital and IV antibiotics were started. She is now complaining of a dull, frontal headache that is worse when she is sitting upright and relieved by lying down. V/S: T=39.3 C (102.8 F), BP=110/70, PR= 75/min, RR=18/min. P/E shows nuchal rigidity and there is a palpable purpuric rash on her lower extremities and trunk. Funduscopic examination is unremarkable. The most appropriate next step is to A. administer sumatriptan, intramuscularly B. advise her to remain in a horizontal position C. ask her if she has been drinking red wine D. order a CT scan of the brain to look for intracranial hemorrhage E. order an MRI of the brain to look for an intracranial mass The correct answer is B. advise her to remain in a horizontal position

This patient most likely has a LP headache, which often begins within 24 hours of the procedure. The headache is positional in nature, meaning that it is worse when she is sitting upright and relieved by lying down. It is believed to be due to a loss of cerebrospinal fluid during the lumbar puncture, causing decreases in the cushioning of the brain. Patients should be told to remain horizontal after the procedure to decrease the incidence of this complication. The treatment includes remaining in a horizontal position and possibly intravenous caffeine sodium benzoate. If this is not effective, an epidural blood patch should be performed.

A 28 y/o man comes to the ED because of moderate Lt. shoulder pain that is worse with abduction of the shoulder. He plays baseball occasionally with friends and has noticed that the pain worsens when throwing the ball. He denies any Hx of trauma. On P/E weakness of the shoulder, most pronounced with abduction is noted. A shoulder x-ray reveals no fractures or dislocations. The most appropriate next step in management is to A. advise patient that he should rest and ice the shoulder B. order an emergent CT scan of the shoulder C. order an emergent MRI of the shoulder D. order an outpatient MRI of the shoulder E. prescribe nonsteroidal antiinflammatory drugs The correct answer is D. order an outpatient MRI of the shoulder Rotator cuff tears are chronic use injuries most common in throwing athletes. The clinical symptoms are weakness and instability of the shoulder. An x-ray may show a subacromial spurring, a high-riding shoulder, or calcific tendonitis. Diagnosis is made by MRI of the shoulder. This is a non-emergent diagnosis and the imaging can be performed on an outpatient basis. Repair is made arthroscopically.

A 17 y/o girl comes to the clinic because of a "reddish bump" on her abdomen. She states that she has had it for as long as she can remember, but her current boyfriend is concerned that it may be "cancer". She has no other complaints and does not take any medications other than ocp for birth control. Her grandfather has malignant melanoma of the eye and her mother has breast cancer. P/E shows a 0.4cm pinkish-brown, raised lesion on the left periumbilical region, 3cm lateral to the umbilicus. A small amount of soft tissue is palpated beneath the lesion. It is directly on an imaginary line that you can trace along the body from the left axilla to the left inguinal region. The remainder of the

physical examination is normal. In discussing this lesion with the patient, you should inform her that : A. coal tar ointment and ultraviolet light therapy is an old but useful treatment B. she has a 70% chance of having a kidney and/or urinary tract defect C. the lesion needs to be removed immediately because it is precancerous D. this is most likely a congenital defect that can be removed for cosmetic purposes E. this needs to be removed immediately and she will need radiation therapy The correct answer is D. This patient most likely has an accessory nipple (polythelia) and extraglandular breast tissue (polymastia), which are present in approximately 2% to 5% of the population. They usually occur along the "milk line," which is an embryologic line that extends from the axilla to the inguinal region. Normal breasts form along this line and other areas of breast tissue and accessory nipples can too. Patients often think that they are "moles" and generally are only concerned about the cosmetic appearance. They can be simply removed as an outpatient, and additional treatment is not usually necessary.

A 74 y/o woman comes to the ED because of a 5 hr history of Rt sided weakness. The sudden weakness came on while she was giving her husband a bath. She says that she was sure that she was having a stroke because her husband has had 2 strokes in the past, but her symptoms resolved spontaneously before she arrived at the hospital. She is very concerned about her risk for a stroke because her husband is completely dependent upon her for care. She is able to ambulate normally and has no residual functional loss. BP=110/80 mm Hg, PR=70/min. Funduscopic examination is normal. A thorough neurologic examination is unremarkable. The most appropriate next step is to : A. advise her to take a daily aspirin B. auscultate the carotid arteries C. obtain an MRI of the head D. order a carotid sonogram with Doppler E. schedule an elective carotid endarterectomy The correct answer is B. This patient most likely had a transient ischemic attack, which is a neurologic event that lasts for less than 24 hours. It is often due to atherosclerotic

disease and therefore auscultation of the carotid arteries may demonstrate the presence of a bruit, which suggests atherosclerotic disease at the carotid bifurcation. TIAs from carotid disease may occur due to ischemia from stenosis or from an embolization of small plaques from the ulceration of the plaque. The next step in this case is to auscultate the carotid arteries because this is the least invasive, cheapest, and quickest method to detect atherosclerotic disease.

A 92 y/o man is brought to the clinic by his son and daughter who tell you that their father "has gone blind". Further investigation uncovers that his visual loss has been a slowly progressive process with no ocular pain. His son and daughter are concerned because the patient lives alone. The patient does not complain of any difficulty seeing and is agitated that he has been brought to your office. Examination reveals normal vital signs and a visual acuity of 20/400 in both eyes. Pupil examination, extraocular movements, and confrontational visual fields are all normal. A penlight exam of the eye shows a yellow-brown color to the lens in both eyes, but is otherwise unremarkable. Direct ophthalmoscopy is very difficult and a sharp view of the retina is not possible. The family should be advised that: A. Age alone is a common cause of decreased vision and his level of vision is what you would expect for a patient his age B. Eye surgery is necessary to prevent total blindness (no light perception) C. Glaucoma is the most likely diagnosis and prompt ophthalmology consult is indicated D. He has macular degeneration and low vision aides should be considered to assist with activities of daily living E. Tell the patient and family that as long as the patient is comfortable with his vision no treatment is necessary The correct answer is E. This patient has cataracts in both eyes. The typical clinical presentation of a patient with cataracts is that of slowly progressive visual loss. Patients may also complain of difficulty with glare and bright lights. They often note decreased color brightness and contrast sensitivity. On exam, a lens that has a cataract will appear yellow-green to yellow-brown. A mature cataract will present as a white lens and visual acuity of hand motions or light perception. With advanced cataracts, the fundoscopic exam may be difficult or not possible. The treatment for cataracts depends on the level of visual deficit experienced by the patient. Appropriate treatments range from observation/education, spectacles, and cataract surgery. If a patient with cataracts is comfortable with his current vision, then treatment with glasses or surgery is not necessary. Referral to an

ophthalmologist may be appropriate for more in depth counseling based on the level of concern of the patient.

A 3 y/o boy is brought to the ED by his parents because of a 24 hr Hx of intermittent, generalized abdominal pain. The parents tell you that he complains of the pain for 10minute episodes and during these times he refuses to walk, but then he spontaneously returns to his normal activities. This occurred 8-9 times yesterday. Today the symptoms occurred more frequently and were associated with 3 episodes of non-bloody, non-billous emesis so the parents brought him into the hospital. There is no history of fever, constipation, or soiling. On examination the patient appears tired and has mild diffuse abdominal pain. He has guaiac-positive stool. PR=125/min. The study most likely to provide a diagnosis is : A. an abdominal x-ray B. a barium enema C. a CBC with differential D. a CT scan of the abdomen E. a LP The correct answer is B. This patient presents with a very common complaint in the pediatric population, abdominal pain. The key to this case is the quality and frequency of this abdominal pain. The pain was described as being diffuse and intermittent with periods of resolution of the symptoms. This type of pain pattern, along with emesis, the lethargy seen in the emergency department, and the guaiacpositive stools should raise red flags for the diagnosis of intussusception. In intussusception a segment of bowel (most commonly the distal ileum into the cecum) telescopes into an adjacent segment causing obstruction. This obstruction tends to resolve and recur causing the intermittent abdominal pain. The barium enema is diagnostic and in many cases a curative procedure as well and is therefore the study of choice in this case.

A 64 y/o farmer comes to the clinic with an injury to the foot, which happened at his farm 24 hrs ago. He reports that he was working on the farm, when he accidentally stepped on a rusty nail, which penetrated deep into his foot. He took some analgesics and he was feeling well. His wife urged him to come to the clinic. He denies any fever, chills, or

rigor. His past medical history is significant for non-insulin dependent diabetes mellitus, which is well controlled with oral therapy. He has no past surgical history. Examination of the foot reveals a deep penetrating wound in the sole of the left foot. There is no associated erythema or induration around the foot. No foreign particles are noted along the edges of the foot, which is tender on palpation. There is no motor or sensory loss. No active bleeding is noted in the penetrating wound. The patient tells you that he had 3 doses of tetanus toxoid injections when he was young. The last tetanus injection was 8 years ago for a similar episode. The most appropriate next step in management is to A. administer a tetanus toxoid booster injection B. administer a tetanus toxoid booster injection and human immunoglobulin C. provide human immunoglobulin D. provide no additional therapy at this time E. surgically debride the wound The correct answer is A. Tetanus prone wounds are any wounds that are over 6 hours old, deep, crushed or penetrated, contaminated with soil, associated with compound fractures, partial or full thickness burns, or human or animal bites. Patients who received 3 doses of tetanus toxoid in the last 5 years do not need further therapy for a tetanus prone wound or for a clean wound. Patients who received a tetanus toxoid between 5-10 years ago and have a tetanus prone wound need booster tetanus toxoid injections, whereas those with a clean wound would need no further therapy. When the last dose of tetanus toxoid was more than 10 years ago, then both clean wounds and tetanus prone wounds need tetanus toxoid booster injections. In addition, those with a tetanus prone wound, whose last dose was more than 10 years ago, need to have human immunoglobulin (choice C) administered.

** A 60 y/o diabetic male is brought to ER by his wife for evaluation. His wife reports that he had developed high fevers 2 days prior, which were attributed to a viral infection and no medical attention was sought. His wife became concerned when the fever persisted despite the use of Tylenol and he became increasingly lethargic. Vitals: HR=130, RR=24, BP=70/55, T=102F On examination, he was flushed, warm to the touch, confused, and diaphoretic. Mucus membranes appeared dry. His neck was supple with negative Kernig and Brudzinski signs. His eyes were equal and reactive to light with no papilledema, and his lungs were clear to asculation bilaterally with no evidence of rales, rhonchi, or wheezes. On cardiac exam he was tachycardic with a normal S1 and S2. A flow murmur was heard. The abdominal and neurologic exams were unremarkable. Examination of the dorsal aspect of the right foot revealed foot cellulites. What would be the first appropriate step of treatment in this man?

A. Immediate dose of IV cefazolin (Ancef) B. Fluid resuscitation with normal saline C. Drainage and culture of wound site D. Obtain CBC, Chem 10, ABG, blood cultures, urine cultures, wound culture E. Stat CT of head Answer is B. This patient is in septic shock given his BP of 70/55 and temperature of 102F. In addition, patient demonstrates flushing, which indicates vasodilation, another characteristic of septic shock. This is most likely the result of bacteremia secondary to his foot cellulitis. In hemodynamically unstable patients, airway, breathing, circulation should be assessed and fluid resuscitation should be started immediately. This should be quickly followed by drainage and culture of wound site, urine culture, and bloodwork including: CBC, Chem10, ABG, blood cultures. Furthermore, empiric antibiotics should be started intravenously. In this case, the most common organisms would be Strepococcus and Staphylococcus, making cefazolin (Ancef) would be an appropriate choice. Mental status changes are often evaluated by CT scan. However, this patient's changes are most likely secondary to his hemodynamic instability and should resolve with resolution of his septic shock.

** A 25 y/o female is brought to ER after being involved in a MVA. She is hypotensive and tachycardic. She has good breath sounds bilaterally. Fluid resuscitation is started immediately with Lactated Ringers solution, and blood is sent for type and cross. She has minor scrapes and lacerations on her torso. She remains hypotensive even after 2L of Lactated Ringers solution and blood transfusion with packed red blood cells is begun. After infusion of 2 units, her blood pressure stabilizes at 110/80 mm Hg. She is placed under observation. 3 hours later she complains of chills, generalized pruritus and difficulty breathing. On exam she is found to have a temperature of 101 F, BP of 110/80, HR 100, RR 20. HEENT is normal. Neck is supple with no JVD. Auscultation of the lungs reveals crackles at the bases. Heart exam is normal. Abdomen is soft, nontender, nondistended, with good bowel sounds, there is no hepatosplenomegaly. Extremities reveal no edema, no cyanosis, and good range of motion. Neurologic exam is unremarkable. Examination of skin reveals a generalized urticarial rash. She is treated with diphenhydramine (Benadryl) and acetaminophen with resolution of symptoms. In the future, if this patient were to require another blood transfusion, what precautions need to be taken? A. None B. Only leukocyte poor packed red blood cells should be given. C. Only irradiated packed red blood cells should be given.

D. Only CMV negative packed red blood cells should be given. E. This patient is not a candidate for any further blood transfusions.

Answer is B. Patient is experiencing a leukoagglutinin reaction. This is an immunologic reaction to antigens present on the white blood cells which are invariably present in the transfused blood products. It occurs most commonly in patients who have been sensitized through previous transfusions or pregnancies. Typical presentation includes fever and chills within 12 hours after transfusion. In addition, cough and dyspnea often occur. Patients respond to acetaminophen and benadryl. Depletion of white blood cells from the blood products can reduce the incidence of future reactions. Pretreatment with acetaminophen and benadryl may also help reduce the recurrence of the reaction.

** A 35 y/o African American female presents to her primary care physician complaining of pain in both legs. The pain started one day ago and got progressively worse. It is localized mostly on the anterior aspect of her legs below the knees. She also complains of malaise and fatigue and thinks she may have a low grade fever. She has no significant past medical or family histories. She takes no medications and is not allergic to any medications. She denies alcohol, tobacco or recreational drug use. Physical exam is unremarkable except for tender erythematous nodules located on the extensor surfaces of both legs below the knees. A chest Xray is done,which demonstrates hilar lymphadenopathy with no parenchymal involvement. Whats the most likely Dx? Answer is sarcoidosis. Painful red nodules located on the anterior aspect of the legs is typical of erythema nodosum. Erythema nodosum may be associated with various infections such asTB, coccidiomycosis, streptococcosis, hepatitis B, syphilis, and yersinia. It may also be due to drugs such as sulfonamides and ocp. Other diseases that are associated with it include sarcoidosis, rheumatic fever, leukemia, and IBD. The erythema nodosum in this patient is most likely secondary to sarcoidosis as evidenced by the significant hilar lymphadenopathy. In addition, the incidence of sarcoidosis is highest in African Americans, and women are affected much more frequently than men. Disease onset is commonly in the third or fourth decade of life.

*** A 43 y/o homeless man is brought in by the paramedics after he was found confused and wandering in the streets. On evaluation he is agitated, confused and combative. He is diaphoretic, emanates a foul odor, and his clothes are filthy with evidence of emesis. He has slurred speech, is noted to be talking to people who are not there, and swatting at invisible objects. Upon questioning he makes an incoherent comment about seeing bugs on the walls. He has a generalized tremor and appears fearful of the staff. His vital signs show: BP 135/80, HR 102, RR 20, Temp 99.8 F. His mucus membranes are dry and skin turgor is normal. Chest and cardiac exams are normal. Abdominal exam is significant for slight hepatomegaly. Neurologic exam reveals intact cranial nerves, DTR's are symmetric bilaterally, but he refuses to cooperate with the remainder of the neurologic exam. He is disoriented to person, time and place. Which of the following changes are consistent with this patient's presentation? ALT MCV [Mg+2] Glucose No No A. inc dec dec change change B. inc inc inc dec dec No C. inc inc inc dec change No No D. inc inc inc change change No No No E. inc inc change change change AST

Answer is B. This patient is exhibiting signs and symptoms of delirium tremens (DT). DT is organic psychosis, which is usually the result of acute alcoholic withdrawal. It is characterized by mental confusion, tremors, visual hallucinations, diaphoresis, and autonomic hyperactivity. It usually manifests within 25-72 hours after the last drink, but, can occur as late as 7-10 days later. DT tends to occur in individuals with previous withdrawal episodes. This patient is a chronic alcoholic and alcohol can cause liver damage. As a result he can be expected to have elevated AST and ALT levels. Usually AST is two times more elevated than ALT. Gluconeogenesis in the liver may be impaired resulting in hypoglycemia. In addition, hypoglycemia, hypomagnesemia and an elevated MCV are commonly seen in alcoholics, and are most likely the result of malnutrition.

** A 19 y/o college student with height 5'2" and weight 120lbs is brought to the emergency room after she experienced two episodes of coffee ground emesis. Her vital signs on arrival are normal. On physical exam she has poor eye contact and a blunted affect. She is oriented to person, time and place. Her mucus membranes are moist, front teeth show loss of enamel. She has mildly enlarged parotid glands bilaterally. Her breath has a rancid odor. Her chest and cardiac exam are normal. She has mild epigastric tenderness, good bowel sounds, no distention, no hepatosplenomegaly. Neurologic exam is unremarkable. A nasogastric tube is placed which drains clear fluid after an initial 10ccs of coffee ground material. An upper endoscopy is performed which demonstrates a linear mucosal tear at the gastroesophageal junction and a diagnosis of a Mallory-Weiss tear is made. From your understanding of the patient's underlying condition, which of the following acid-base abnormalities would you expect? A. Hypochloremic metabolic alkalosis B. Hypochloremic metabolic acidosis C. Hyperchloremic metabolic alkalosis D. Hyperchloremic metabolic acidosis E. No abnormalities are expected.

Answer is A On Hx & P/E , this patient has many concerning signs and symptoms. Her wornappearing enamel, bilaterally enlarged parotid glands, rancid breath odor, and poor eye contact point to an eating disorder, most likely bulimia nervosa, given her normal height to weight ratio. In addition to her chronic disease, she is acutely experiencing pain secondary to a Mallory-Weiss tear. A Mallory-Weiss tear ia a mucosal tear at the gastroesophageal junction, which arises from events that suddenly raise transabdominal pressure. Examples include sudden lifting, or as in this patient, retching or vomiting. Given this patient's history of retching and vomiting, one would expect her to have hypochloremic metabolic alkalosis, as H+ and Cl- are lost during vomiting.

45 y/o male with atrial rate of 300/min & ventricular response in 2:1 ratio, has been treated with an antiarrhythimc agent, which slowered the atrial rate to 200/min & increased the ventricular rate to 200/min. What was the pharmacologic agent used? a- adenosine b- digoxin c- propranolol d- quinidine e- verapamil Answer is d- quinidine.

Quinidine slowers the atrial rate due to its vagolytic action & increases AV conduction. So, an atrial flutter at 200/min was conducted to the ventricles in a 1:1 ratio, resulting in an increase in ventricular rate to 200/min. This effect could be avoided by use of a drug that slows AV nodal conduction like choices a, b, c & e.

Which of the ffg correctly identifies the sequence of cardinal movement of labor that fetus undergoes prior to delivery? a- descent, ext. rotation, extension, int. rotation, flexion b- descent, flexion, int rotation, extension, ext rotation c- int rotation, flexion, descent, ext rotation, extension d- flexion, extension, int rotation, descent, ext rotation e- extension, descent, int rotation, ext rotation, flexion answer is b- descent, flexion, int rotation, extension, ext rotation

35 y/o woman, G3P2, at 39 wks gestation undergoes a vaginal delivery of a 3295 gr male neonate. The placenta was not completely removed & after bleeding she underwent a total abdominal hysterectomy due to placenta accreta & receives 5 unites of packed RBC. Which of the ffg hormones are most likely to be affected by this scenario? a- ACTH b- PRL c- TSH d- FSH e- ADH f- None of the above g- All the above Answer is b- PRL due to developing sheehans syn.

A 26 y/o woman with recently diagnosed Hodgkin's disease is admitted to the ICU after an overdose of lorazepam and alcohol. She is intubated and stable. Which of the ffg would be expected to help prevent the development of ventilator-associated pneumonia in this patient? A. Keeping the patient in the semi-recumbent position B. Routine changes of the ventilator circuit C. Nasotracheal (as opposed to oral) intubation D. Prophylactic antibiotics E. Chest physiotherapy

Answer is A. Keeping the patient in the semi-recumbent position Ventilator-associated pneumonia accounts for more than 50% of all infections in the ICU. Handwashing, semi-recumbent positioning, kinetic (rotational) bed therapy, and drainage of condensate from ventilator circuits are effective strategies for preventing such infections. However, routine changes of the ventilator circuit and chest physiotherapy are ineffective. There is some evidence to support oral, as opposed to nasal, intubation. Influenza and pneumococcal vaccination, which should be addressed in all patients at the time of hospital discharge, would not affect the development of ventilator-associated pneumonia.

In a patient with steatorrhea, which of the following test results is the most specific for the diagnosis of chronic pancreatitis? A. Abnormal Schilling test that does not normalize with intrinsic factor B. A serum carotene value of 25 g/mL C. A 72-hour fecal fat content of 30 g D. A serum trypsinogen value of 5 ng/mL E. An abnormal 13C triolein breath test Answer is D. a serum trypsinogen of 5 ng/ml The serum trypsinogen concentration, when low, is the only test in this group that is specific for pancreatogenous steatorrhea. The other test results are associated with, or diagnostic of, steatorrhea but do not identify specific cause of steatorrhea. The Schilling test, if abnormal after intrinsic factor, can occur with ileal disease, bacterial overgrowth, or chronic pancreatitis. A low serum carotene concentration, abnormal 72hour fecal fat test, or abnormal 13C triolein breath test are present with steatorrhea of any cause, for example, small bowel mucosal disease, bacterial overgrowth, chronic pancreatitis, and ileal resection.

A 38 y/o farmer has acute onset of severe, progressive dyspnea associated with cough and weakness. These symptoms occurred 1 hour after the patient had worked in a silo that had been filled with fresh corn silage the previous day. Which of the following is most likely to have caused his acute pulmonary decompensation? A. Exposure to thermophilic actinomyces B. Viral pneumonia C. Pneumothorax D. Nitrogen dioxide pneumonitis E. Allergic bronchopulmonary aspergillosis

Answer is D. Nitrogen dioxide pneumonitis Silo filler's disease results from inhalation of oxides of nitrogen, including nitrogen dioxide, which tends to accumulate at the top of tall storage silos. The oxides of nitrogen are generated soon after the silo is filled with fresh silage that is subsequently used for animal feed. Nitrogen dioxide, when dissolved in the aqueous film that lines the respiratory tract, becomes nitric acid and produces a chemical pneumonitis. When the farmer enters the top of the silo to level the silage or prepare it for mechanical unloading, exposures can be quite high. The chemical pneumonitis of silo filler's disease must be differentiated from farmer's lung, which is an immunologic reaction to molds or thermophilic actinomycetes that grow in hay during the storage season. Exposure to this organic material occurs when farmers spread the hay for animal feed during the winter. Hypersensitivity pneumonitis can then occur 6 to 8 hours after exposure.

Which of the ffg inhalational anesthetics will provide the most rapid rate of recovery? Blood:Gas partition coefficient a- nitrous oxide 0.5 >100 b- desflurane c- sevoflurane d- isoflurane e- halothane 0.4 0.7 1.4 2.3 7 3 1.4 0.8 min. alveolar concentration (%)

Answer is b- desflurane With an inhalational anesthetic, both rate of onset & rate of recovery depends on blood: gas partition coefficient, so the lower the coefficient, the recovery time is shorter.

A 7 y/o female is reffered to the physician because of cessation of speech in midconversation, a blank stare & flickering of the eyelids which lasts for 10 seconds and then she resumes regular activity. The physician suspects a seizure disorder and asks the child to hyperventilate, which cuase the symptoms to occur. Which of the ffg is characteristics of this disorder in EEG? a- hypsarrhythmia b- an interictal slow-spike wave c- centrotemporal spikes

d- a 3/sec spike & wave pattern e- a 6/sec spike & wave pattern answer is d- a 3/sec spike & wave pattern. ( absence seizure ) hypsarrhythmia -- > infantile spasm interictal slow-spike wave -- > Lenox- Gastaut syn. centrotemporal spikes -- > benign partial epilepsy 4-6/sec spike & wave pattern -- > juvenile myoclonic epilepsy

a 2 y/o symptomatic HIV-infected child presents because of exposure to measles. His immunization is up to date. He received IG 6 wks ago, which of the ffg is the most appropriate course of action? abcdeAdminister monovalent measles vaccine Administer another MMR vaccine Administer IG Administer Vit C Do nothing, because the child is already immunized for measles

Answer is c- Administer IG. Symptomatic HIV infected children exposed to measles should receive IG prophylaxis regardless of vaccination status, because they have poor immunologic response to vaccine. If a child received IG within 3wks of exposure , no additional IG is required.

A 27 y/o woman with schizophrenia and hx of seizure is stabilized on valproate & then started on clozapine. One wk later, she complains of sedation & sialorrhea. Which of the ffg should be performed immediately? abcdeclozapine level prolactine level thyroid function valproate level WBC count

Answer is e- WBC count.

Weekly monitoring of WBC count during Tx with clozapine is required because 1% risk of agranulocytosis. Sedation & sialorrhea are untoward effects of clozapine Tx. Clozapine & valproate level may have clinical significance, but are not immediately required.

A 7l y/o woman presents with pain in the Rt. lateral thigh and buttock. She has had this pain for the last 6 months, and symptoms are aggravated by prolonged standing and walking, and they improve with rest. In recent years, she has also had intermittent LBP, but there has been none in the last 6 months.On P/E, the patient has mild limitation of rotation in both hips but no pain on motion. There is diffuse mild tenderness in the Rt. lateral thigh and buttock. Abduction of the hips against resistance does not increase her pain. The lumbar region of the spine is not tender, but there is moderate increase in lumbar lordosis and in paravertebral muscle tone. Her pain is not increased by bending forward, but there is some increase in pain with full spinal extension. There are no neurologic deficits.What is the most likely cause of this patient's pain? A. Osteoarthritis of hip B. Arthritis of sacroiliac joint C. Trochanteric bursitis D. Osteoarthritis of lumbar spine E. Spinal stenosis Answer is D. Osteoarthritis of lumbar spine. Referred pain from the lower lumbar facet joints or intervertebral discs is often reported by the patient as hip pain, with localization to the buttock and lateral thigh. The pain is characteristically aggravated by maneuvers that increase the force brought to bear on these joints, particularly spine extension as well as lateral bending or rotation toward the painful side. Pain over the spine may or may not be present, but even in its absence, there may be paravertebral muscle spasm contributing to reduced spine motion. Frequently referred pain may have a different localization from radicular pain at the same level. Lower lumbar nerve root impingement usually causes pain below the knee. Pain resulting from hip joint pathology is usually felt in the groin and anterior thigh. Hip motion, particularly rotation, is usually painful. Sacroiliac arthritis usually results in constant pain and is particularly severe in the morning and improves with motion. Trochanteric bursitis causes lateral hip and thigh pain, with tenderness over the trochanter. A provocative test designed to increase the pain involves abduction of the hips against resistance; this results in contraction of muscles that attach to the trochanter. The maneuver did not increase pain in this patient. Spinal stenosis causes compression of lumbosacral nerve roots, usually producing bilateral lower leg pain.

A 60 y/o woman has a long hx of subumbilical abdominal pain occurring during periods of stress and sometimes aggravated by ingesting coffee, nuts, or chocolate and by constipation. Recently, the pain has become worse on the left side of the abdomen. The results of barium enema and upper GI series are normal, and a CT of the abdomen and pelvis shows a 3 x 3-cm loculated cyst in the tail of the pancreas. There are no cysts in the liver or kidneys. The results of all blood tests, including the test for amylase, are normal. The best management of this patient is: (A) (B) (C) (D) (E) Surgical removal of the cyst Three monthly CT scans to determine whether the cyst enlarges or subsides Reassurance and no further evaluation ERCP to determine whether the cyst communicates with the pancreatic duct Needle aspiration of the cyst and examination of the cyst fluid

Answer is A. The incidental finding of a pancreatic cyst can be a difficult decision-making problem. In the healthy patient, the best course is to advise surgery. It is almost impossible to distinguish a cystadenoma from a cystic adenocarcinoma or a benign solitary cyst due to chronic pancreatitis from a malignant cyst or mucinous duct ectasia. Because about 20% of these cysts turn out to be malignant, the patient may prefer to opt for one of the other alternatives, for example, three monthly CT scans to determine whether the cyst enlarges, ERCP to determine whether the cyst communicates with the pancreatic duct and this is more likely to be due to pancreatitis, or a needle aspiration of the cyst and examination of the cyst fluid for malignant cells. Clearly, in older patients with multiple medical problems, one might agree to this course before undertaking surgery. However, whatever course is decided upon one should be very wary of telling the patient that they have a small insignificant finding and nothing further needs to be done.

A 57 y/o man with cirrhosis attributable to alcohol and hepatitis C has been treated with diuretics for ascites for 3 months. He has abstained from alcohol for 4 months and has been compliant with a 2-g sodium diet. Medications include aldactone, 300 mg/d, and furosemide, 80 mg twice a day. P/E shows a moderately wasted, icteric-appearing man with a BP of 105/62 mm Hg, PR of 94/min, and no fever. Chest examination is clear to auscultation and percussion, and the abdominal examination reveals tense, massive distention with shifting dullness and active bowel sounds; the liver and spleen size cannot be determined by palpation. Pitting edema is present to the mid-shins bilaterally. Laboratory studies:

Serum sodium Serum potassium Serum bicarbonate Serum chloride Serum total bilirubin Serum albumin Blood urea nitrogen Serum creatinine Prothrombin time

128 meq/L 5.0 meq/L 32 meq/L 100 meq/L 3.2 mg/dL 2.4 g/dL 18 mg/dL 1.9 mg/dL 15.5 seconds

Abdominal ultrasound demonstrates massive ascites, a small diffusely echogenic liver without focal masses or biliary dilation, and splenomegaly. The most appropriate treatment at this time is: (A) Addition of hydrochlorothiazide therapy (B) Total paracentesis with albumin infusion (6 to 8 g/L of fluid removed). (C) Continue current diuretic therapy and re-educate the patient about dietary salt restriction (D) Discontinue diuretic therapy and observe Answer is D. This patient has decompensated liver disease, has developed tense ascites despite aggressive diuretic therapy with high doses of combined diuretic therapy, and presents with hyponatremia and renal insufficiency. His ascites can be considered diureticintolerant because he demonstrates hyponatremia and renal insufficiency on the current regimen. Consequently, continuing the present diuretic regimen is not appropriate; his condition would likely deteriorate further if more aggressive diuretic management with addition of thiazide diuretics were attempted. Large-volume or total paracentesis is effective in managing refractory ascites. It is recommended that this be performed with albumin infusion (6-8 g/L) to decrease the risk for exacerbation of renal insufficiency. It has been shown that large-volume paracentesis (>5 L) can occasionally result in prolonged circulatory dysfunction typified by decreased intravascular volume in cirrhotic patients when performed without colloid infusions. In this patient with marked renal insufficiency, as suggested by a serum creatinine of 1.9 mg/dL in the setting of advanced liver disease and muscle wasting, who has no symptoms attributable to large-volume ascites that necessitate immediate treatment, large-volume paracentesis may result in deterioration despite colloid infusion and should be avoided. Consequently, this patient can be considered diuretic intolerant; discontinuing diuretics and observation is the preferred therapy at this time.

Which of the following can cross the placental membrane ? a. IgM b. Insuline c. Bacteria d. Methyl Dopa e. Steroid Hormones answer is e.

68 y/o man is brought to you in a comatose state, his eyes are in midposition and pupils are pinpoint, what is the most likely location of the hemorrhage? a. pons b. putamen c. thalamus d. cerebellum The correct answer is a. pons. Option b is incorrect, putament hemorrhage cause deviation of both eyes to the side of the lesion (away from hemiparesis), but the sizes of pupils are normal. Option c is also incorrect, in thalamus hemorrhage, although the sizes of the pupils are small; both eyes are deviated inward and look at nose. Option d is also incorrect, patients with cerebellum hemorrhage have a normal size of pupils and they are unable to look toward the side of the lesion.

23 y/o woman presents with excessive hair growth on face, chest, and on the abdomen, her physical and blood examinations are normal, what is the most likely cause? a. familial b. ovarian tumor c. 21-hydroxylase deficiency d. polycystic ovary syndrome The answer a. Familial (or idiopathic) is the most common cause of hirsutism. Ovarian tumors are very rare causes of hirsutism (only 0.8%). 21-hydroxylase deficiency causes ambiguous genitalia in baby girls. Polycystic ovary syndrome is suspected in obese women with amenorrhea and a serum LH:FSH ratio greater than 2.0.

A 35 y/o African American man presents to your office for a first time visit. He has no past medical history, and takes a multivitamin daily. He is active and runs approximately 3 miles/wk. He has a 10-pack/yr Hxof smoking and has 1-2 beers/wk. v/s: T=37 C (98.4 F), BP=110/60 mm/Hg, PR=65/min and regular, and RR=15/min. On P/E, a II/VI systolic ejection murmur at the Rt. upper sternal border is noted that has no respiratory variation, increases in intensity upon going from supine to standing and Valsalva, and decreases in intensity with sustained handgrip. There is a prominent, nondisplaced PMI. The remainder of his examination is unremarkable. The most likely Dx is: A. aortic stenosis B. IHSS C. MR D. PDA E. tricuspid regurgitation The correct answer is B. IHSS This condition involves an aortic outflow obstruction that is dependent upon both preload and afterload. As preload is decreased (i.e., as with a Valsalva maneuver or moving from supine to erect) the ventricular chamber size is decreased, the degree of outflow obstruction is increased, and the intensity of the murmur is increased. As the afterload is increased (i.e., with handgrip) the end systolic chamber size is increased, the degree of outflow obstruction decreased, and the intensity of the murmur is decreased. The prominent PMI suggests a left ventricle that has hypertrophied in response to the outflow obstruction. The identification of IHSS is important since those with this form of cardiomyopathy are at increased risk for sudden death.

A 57 y/o man with schizoaffective disorder, whose symptoms were in remission until 2 wks ago, is brought to the ED by his girlfriend. He will not talk with you but the girlfriend tells you that he has a hx of "overdoses", and she is afraid he has taken a lot of "his pills". He has been complaining of voices telling him he "should be dead". He has not left the house in a month and has spent several hours a day looking out the window for the "king and savior" to "come take him". His medications include haloperidol, valproic acid, and a small dose of amitriptyline for chronic pain related to nerve damage in his leg, which occurred in a motor vehicle accident 10 years ago. He has no other medical problems. A chart review reveals that he has no allergies and was diagnosed with schizoaffective disorder 30 years ago. Vitals: T=37.0 C (98.6 F), BP=110/70 mm Hg, PR=70/min, and RR=26/min. He is a depressed appearing man with very poor eye contact. He smells of alcohol. He does not acknowledge you, but will answer some questions for his girlfriend. He does admit to feeling that "life is not worth living" and feeling "more religious than usual". He seems slightly drowsy and knows the date. His physical examination is normal. The most appropriate next step in management is to order A. acetaminophen and salicylate levels B. blood alcohol level

C. ECG D. urine toxicology for street drugs E. valproic acid level The correct answer is C.ECG This patient may have taken several cardiotoxic medications. Haloperidol and other antipsychotics increase the QT interval, as do the tricyclics. In an overdose for either medication, he is at risk for sudden death from torsades.

A 14 y/o boy is brought to the office by his mother because of "bedwetting" episodes that have been occurring about twice a week for the past few months. The mother says that she noticed this "problem" when she washed his pajamas, and he "refused" to talk about it when she tried to bring it up. She is hoping that he will talk to you. You ask her to leave the room so you can have some privacy with her son. He starts the conversation by stating that "this is getting embarrassing" and he "doesn't understand what's going on." He says that he gets up and finds his pajamas "wet and sticky." He denies any dysuria or frequency during the day, and denies any problems at school or at home. He is on the basketball team, socializes with friends, and gets good grades. Physical examination is unremarkable and shows a pubic hair stage of Tanner IV and genital development Tanner stage III. The next best step is to A. advise him to stop drinking water at 8 pm and urinate before bed B. begin a 3-day treatment regimen with trimethoprim-sulfamethoxazole C. obtain a urine sample for U/A & culture and sensitivity D. order a renal ultrasound E. reassure him that this is a completely normal part of puberty F. order CBC & electrolytes The correct answer is E. reassure him that this is a completely normal part of puberty This adolescent boy is most likely having nocturnal emissions ("wet dreams"), which are a normal part of puberty. A nocturnal emission is when the penis becomes erect during sleep and ejaculates. If the boy or his mother do not know about nocturnal emissions before they occur, they may think that he urinated. He should be reassured that this is normal. It is also important to discuss it with his mother so she does not make him feel uncomfortable about it in the future.

A 36 y/o woman comes to the ED because of a severe headache. She states that the headache woke her up from sleep 6 hours ago, and was not relieved by aspirin or acetaminophen. She also noticed that she has neck stiffness and that "it hurts" during neck extension and flexion. She was recently diagnosed with Hodgkin's disease, but before that, she was usually "pretty healthy," except for a few UTI and HTN. V/S: T=39.0 C (102.2 F), BP=130/80 mm Hg, PR=75/min, and RR=17/min. She appears

lethargic. P/E shows nuchal rigidity, flank tenderness, and a mid-systolic click. Funduscopic examination shows bilateral optic disc swelling. Whats the most appropriate first step to do: A. B/C & administer IV ceftriaxone B. order a CT scan of the head C. order a MR angiogram of the head D. order an MRI of the head E. perform a LP The correct answer is A. B/C& IV ceftriaxone This patient has the signs and symptoms that are suggestive of meningitis, but she also has papilledema (optic disc swelling), which means that an intracranial mass must be ruled out before performing a lumbar puncture (to reduce the risk for brain herniation). However, empiric antimicrobial therapy should be started before the neuroimaging study so that clinical deterioration does not occur.

A 17 y/o girl is brought to the office by her mother because she has missed many periods. The girl admits to binge eating and exercising in order to prevent weight gain. She tells you that she is definitely not pregnant, because she has not had any sexual relations in the past 11 months and thinks she is not getting her menstrual period because of the excessive physical exercise she has been doing in the past several weeks. P/E is significant for bradycardia and significant weight loss compared to the last year. A pregnancy test is negative.At this time whats the first step? A. order amylase B. check BUN and creatinine C. order LFT D. check serum K E. thyroid function tests The correct answer is D. check serum K Bulimic patients frequently engage in compensatory behaviors to prevent weight gain. Those include self-induced vomiting, abuse of diuretics, laxatives, enemas, or diet pills. The metabolic disorders frequently seen in these patients are, mostly hypokalemia and hypomagnesemia. Thyroid function tests should be done as a part of regular workup of patients presenting with this clinical picture. It is not, however, the first to be ordered.

45 y/o truck driver involved in MVA, resulted in closed hesd injury, presents to ER, he was intubated at the field and on arrival hes oxygenated well with assisted ventilation, has normal BP & moderate tacchycardia. GCS = 7, pupils are equal & slowly reactive. After stabilization, a head CT is done which shows small SAH & Rt. frontal lobe contusion. Abd. CT is normal. The optimal Mx of this px ICP is : a- fluid restriction, hyperventilation, IV steroids

bcde-

fluid restriction, hyperventilation & ventriculostomy fluid restriction & osmotic diuresis normovolemia, normocarbia & ventriculostomy craniectomy

answer is d- normovulemia, normocarbia & ventriculostomy The principle of Mx of closed head injury is to maintain cerebral perfusion & oxygenetaion to prevent secondary brain damage. Remember this formula: CPP ( cerebral perfusion pressure )= mean BP ICP, So normal CPP requires adequate circulating blood volume with maintanace of normovolemia. Early ventriculostomy is beneficial to permit controlled drainage of CSF to maintain normal CPP. Note: Fluid restriction, hyperventilation, hypercarbia ( -- > vasodialation & inc. ICP ) should be avoided.

A 70 y/o man with hx of HTN & mild CHF ( which was controlled with digoxin & diuretics ) is admitted for an AAA repair. To faciliate perioperative Mx, a swan- ganz cath. is inserted in the OR. During a first few hours post op hes noted to have BP= 140/70, HR=110, flat neck veins, pulmonary arterial wedge pressure of 9 mm/hg & poor urine output. 1- The next best step of Mx is : a- IV furesmide b- Bolous IV crystalloid c- Dopamine infusion d- Nitroprusside infusion e- IV digoxin Several hours after this intervention, BP=150/85, HR=90, neck veins are distended, pulmonary arterial wedge pressure is 17 and urine output is still low. 2- At this point, Mx should be: a- IV furesmide b- Bolous IV crystalloid c- Dopamine infusion d- Nitroprusside infusion e- IV digoxin Answers are: 1- b, 2-d In the initial post op period, px has a low pwp & poor urine output, renal perfusion is compromised by hypovolemia with subsequent inadequate preload & decreased cardiac output. So at this time IV fluid resuscitation is appropriate. After fluid bolous px developes extended neck veins & elevated pwp, indicating biventricular dysfunction with increased left end diastolic pr. & increased left ventricular end systolic volume. CO is low & urine outpout is not improved, so with px hx all scenario is due to increased afterload which can be reduced by nitroprusside infusion.

70 y/o man presents with back pain & difficulty urination. On DRT, he has a hard, irregularenlarged prostate. PSA is elevated & osteoblastic lesions of vertebral column & pelic bones are noticed. The Tx of choice is: a- radical prostatectomy b- transurethral prostatectomy c- cytotoxic chemotherapy d- hormonal manipulation e- radiotherapy answer is d- hormonal manipulation this elderly px has metastatic prostate cancer, so Mx would be tumor control for palliation of symptoms, which is hormonal manipulation either with orchiectomy or exogenous estrogen therapy.

A 75 y/o woman is brought to ER from nursing home for jaundice & mental confusion. The nusrsing home notes states that she became less responsive & developed jaundice over the last 2 wks. PMH is positive for HTN, DM & prior colon resection for colon cancer at age 55. V/S : BP= 100/60, HR= 110, T= 101.5, P/E shows no response to verbal command but withdraws to pain, a mild jaundice with tenderness in epigastrium & RUQ. Whats the most likely Dx? a- Hepatitis A b- Biliary stricture c- Choledochal cyst d- Liver metastasis e- Choledocholithiasis f- Cirrhosis g- Pancreatitis Answer is e- choledocholithiasis. Common bile duct stones may be the cause of acute bile duct obstruction without warning resulting in jaundice, pain & sepsis. The sepis may manifest as fever, hypotestion & altered mental status. A 66 y/o woman who has previously been healthy undergoes emergency surgery for a ruptured AAA. Intraoperatively she requires 8 units of packed red blood cells to maintain her blood pressure and hematocrit. After surgery she is hemodynamically stable. On the third post-op day she appears jaundiced, but abdominal examination is unremarkable and she is afebrile. Lab results: Total serum BR=8.3 mg/dL (direct= 6.3 mg/dL) Serum Alk. Ph.= 360 U/L Serum AST = 51 U/mL

The most likely explanation for the woman's jaundice is A: a stone in the common bile duct B: halothane hepatitis C: posttransfusion hepatitis D: acute hepatic infarct E: benign intrahepatic cholestasis The answer is E- Benign intrahepatic cholestasis Benign post-op intrahepatic cholestasis can develop as a consequence of major surgery for a catastrophic event in which hypotension, extensive blood loss into tissues, and massive blood replacement are notable. Factors contributing to jaundice include the pigment load from transfusions, decreased liver function resulting from hypotension, and decreased renal bilirubin excretion caused by tubular necrosis. Jaundice becomes evident on the second or third postoperative day, with bilirubin levels (mainly levels of conjugated bilirubin) peaking by the tenth day. Serum Alk. Ph. concentration may be elevated up to tenfold, but AST level is only mildly elevated. Hepatitis, choledocholithiasis, and hepatic infarct are unlikely diagnoses in the absence of abdominal tenderness, fever, or a significant rise in AST levels. The incubation period of posttransfusion hepatitis is 7 weeks, making this diagnosis unlikely.

A 6 y/o boy is brought by his mother because of a "red rash" that she noticed today. She says that 3 days ago he had a cough, runny nose, and fever that responded to ibuprofen. T=37 C (98.6 F) and he has a normal P/E with the exception of an erythematous, blanching macular rash on his legs. You diagnose him with a viral exanthem and advise the mother to encourage the child to drink liquids and to use ibuprofen as needed for fever. One week later, the mother brings the child back to the office and reports that the rash has "changed", he has developed colicky abdominal pain several times per day, and he is complaining of left knee pain. V/S: T=37.2 C (99 F), BP=100/65 mm Hg, PR=100/min, and RR=15/min. P/E reveals a well-appearing child with palpable purpura of both lower extremities, normal neck examination, clear lungs, and a soft, non-tender abdomen. His left knee is painful on flexion, but it is not erythematous or warm, and there does not seem to be an effusion. His gait is normal. The most appropriate study at this time is : A. arthrocentesis B. colonoscopy C. cultures of blood, urine, and cerebrospinal fluid D. echocardiography E. urinalysis

The correct answer is E. This patient most likely has Henoch-Schonlein purpura (HSP), a smallvessel vasculitis seen most commonly in children between the ages of 2 and 8. The child's preceding upper respiratory tract infection, low-grade fever, and arthralgias are all common elements of this disease. The typical rash of HSP is an evanescent, erythematous, macular rash on the lower extremities that progresses over the course of days to petechiae and palpable purpura. These change in color from red to purple to brown before eventually fading, normally over the course of weeks. HSP is an IgA-mediated autoimmune vasculitis, which can cause tissue damage as a result of immune complex formation. Deposition of these immune complexes in the kidneys can lead to nephritis, which is the leading cause of permanent sequelae from HSP. End-stage renal disease is an uncommon but possible outcome. It is important to perform frequent urinalyses for early detection of kidney involvement.

A 5 y/o boy is admitted to the hospital because of his increasing irritability and fever. The mother reports that the child has been having upper respiratory symptoms for the past week. In the last few days, the child has been constantly rubbing his left ear and has been increasingly irritable. The patient has had multiple ear infections in the past, that were treated with oral antibiotics. Following recurrent episodes of these ear infections, the child was advised to take a prophylactic, single dose of amoxicillin at bedtime. The child has been taking these antibiotics regularly for the past 3 months. On examination, the child is found to have a fever of 38.8 C (101.8 F). Examination of the right ear does not reveal any abnormalities. The examination of the left ear is uncomfortable, but the external auditory meatus appears normal. The tympanic membrane is examined after the removal of the cerumen and is noted to be hyperemic, bulging with indistinct anatomical landmarks. Light reflex is diminished and there is limited mobility on pneumatic insufflations. Some amount of middle ear effusion is also noticed. The most appropriate next step in the management is to A. advise a tympanostomy tube placement B. continue antibiotic prophylaxis with amoxicillin C. prescribe a 10-day course of oral antibiotics D. recommend decongestants, along with oral antibiotics E. start intravenous antibiotic therapy The correct answer is A. Referrals for discussion of a tympanostomy tube placement should be considered if there is chronic bilateral effusion for more than 3 months in duration, unilateral effusion for more than 3 months in duration, language development delay, hearing loss of more than

20 decibels, or failure of antibiotic prophylaxis. This particular child has a history of recurrent acute otitis media treated with antibiotic prophylaxis. Failure of antibiotic prophylaxis requires tympanostomy tube placement.

An 8 mo/o infant is braught because of constipation. The father, a "stay-at-home dad," tells you that she has been having 1 bowel movement every 3-4 days, and that the stool is always very hard. He says that she is doing very well otherwise; she is a very happy and easy little girl. She is fed primarily infant formula and he is starting to introduce solid foods. He says that he is concerned because he remembers always having to change "very dirty" diapers for both of his other children, at least twice a day. Physical examination is unremarkable. A rectal examination shows guaiac negative brown stool. The most appropriate next step is to: A. advise him to give her mineral oil 3 times a day until she is "regular" B. advise him to give her prune juice or pear juice C. determine thyroid-stimulating hormone levels D. order a barium enema E. order rectal manometry and a rectal biopsy F. reassure him that all infants have different bowel habits The correct answer is B. Constipation is a common problem in formula-fed infants, and it is best treated by increasing the amount of fluids in the diet, especially with fruit juices that contain sorbitol, such as prune and pear, which help to relieve constipation. It is often caused by a diet that is too low in fluids or deficient of bulk.

A 7 day/o boy who is the product of an uncomplicated gestation is brought to the physician because of hypospadias. The baby is otherwise healthy, and is urinating without any difficulty. On physical examination, vital signs are stable, lungs are clear and the heart is beating at a regular rate. The only abnormal physical finding is the hypospadias. U/A is negative for infection. Which of the following is the most appropriate next step? A. Measuring serum creatinine level B. Schedule a renal ultrasound C. Obtain an IVP D. Cystography E. Performing a circumcision

The correct answer is B. Children with hypospadias are prone to urinary tract infections and other urinary tract anomalies. They require careful evaluation. A renal ultrasound is a safe way of diagnosing neonatal urinary tract pathology.

A 4 y/o old boy falls from the jungle gym at preschool. He sustains minor abrasions and contusions, and is taken care of by the school nurse. His parents take him that same afternoon to his regular pediatrician, P/E is unremarkable, his Hgb=14 g/dL, and U.A shows microhematuria. Which of the following is the most appropriate next step in management? A. CT scan of the abdomen and pelvis B. Reassure the parents that microhematuria from minor trauma will resolve spontaneously C. Serial hemoglobin and hematocrit determinations D. Urologic workup, starting with a sonogram E. Retrograde ureterogram and cystogram Answer is D. Microhematuria after trivial trauma in children may be a sign of a congenital anomaly that makes the urinary tract unusually vulnerable. So work up is necessary. The first, noninvasive test should be the sonogram.

A 5 wk/o infant is brought for a 4 wk hx of noisy breathing that has not improved. She has otherwise been healthy except for a current URI for the past 4 days, which according to the parents, has worsened the noisy breathing. On P/E, she has inspiratory stridor. The noisy breathing improves when the infant is asleep. Which of the following is the most likely diagnosis? A. Bronchoalveolar carcinoma B. Foreign object obstruction C. Laryngomalacia D. Bacterial pneumonia E. Tuberculosis The correct answer is C. The patient has stridor on examination which is an inspiratory obstruction that is sensitive to airflow changes. In children, the most common cause of stridor is laryngomalacia.

A 5 y/o boy develops a headache, cough, myalgia and a fever. He has been a healthy child with all immunizations up to date. He is given a decongestant and an aspirin for his symptoms with some relief. However, 4 days later, he is brought back by his parents

because of persistent vomiting and irritability. On P/E, he is found to be semicomatose, becoming combative on stimulation. Which of the following would confirm the Dx? A. Serum ammonia B. Serum BUN C. Serum calcium D. Serum opiate level E. Serum Na The correct answer is A. The child is presenting with symptoms of Reye syndrome, which is an acute encephalopathy associated with high ammonia levels. It most commonly occurs in young children after a viral illness. Administration of aspirin increases the risk of developing this disorder. The vomiting is characteristic.

A 3 wk/o African American boy is presented because of a generalized seizure 2 hrs ago. The infant is highly irritable with high pitched cry, his weight is 2.5 kg, BP=70 /40 mm Hg, PR=145/min and RR=50/min. Laboratory results: Blood Glc=120 mg/dL BUN= 50 mg/dL Serum Na=170 mEq/L Serum Ca=8.5 mg/dL Serum magnesium=1.5 mg/dL Which of the following is the most likely cause of this infants seizure? A. Hypocalcemia B. Hypoglycemia C. Hypomagnesemia D. Intracranial hemorrhage E. Meningitis Answer is D- intracranial hemorrhage The level of serum sodium in this patient is 170 mEq/L. Infants who have hypernatremic dehydration are irritable and lethargic, and have a high-pitched cry. This type of dehydration results from a greater loss of hypotonic fluid than sodium and accounts for about 15% cases of dehydration. Because the patient has no history of diarrhea or vomiting, the hypernatremia may be due to inadequate supply of mother's milk that does not match the insensible water loss. Another cause can be the high concentration of sodium in mothers milk. Generally, after the child's birth, sodium in the colostrum decreases from its highest level to its lowest level by the fourth week. However, some mothers continue to excrete high sodium in their milk and can potentially cause recurrent hypernatremia and in some case intracranial hemorrhage in the infant.

3 y/o child presents with recurrent Rt. lower lobe pneumonia. Growth parameters is on 25th percentile & developmentally appropriate for his age. PMH is significant for ear

infection at 18 months & gastroenteritis at 2 yrs of age. Which of the following conditions is most likely responsible for this pt.'s disease? 1.Primary B- or T- cell immunodeficiency 2.Cystic fibrosis 3.Chediak higashi syn. 4.Congenital lung abnormality 5.Foreign body aspiration Answer is 5. foreign body aspiration Foreign body aspiration is the most common cause of recurrent pneumonia in an otherwise healthy 3 yo child. Becoz of the lung's anatomy,the rt.side is more common for aspiration.

A 4 mo/o white male is braught for his well-child visit. Both his father and his teenage brother have idiopathic epilepsy, which began during early childhood. Six hrs after the infant's 2-month DTaP he developed a fever of 39.5 C (103.1 F) rectally. His parents are concerned about the safety of further DTP immunization. Which one of the following is the best approach to DTaP immunization at this visit? a. Administer the routine DTaP immunization b. Administer DTaP (acellular pertussis) c. Administer one-half the usual dose of DTaP d. Administer dT (no pertussis component) e. Delay further DTaP until he is 6 months old Answer is a. False assumptions regarding contraindications often result in the needless deferment of indicated immunizations. The list of contraindications and precautions for DTaP immunization does not include a previous febrile reaction unless the fever was greater than 105 F, nor does it include a family history of seizures.

1- 32 y/o man presents with dizziness and syncope when elevates his right arm. Subclavian steal syn. 2- 70 y/o man with back pain, increased urinary hydroxyproline and serum alkaline phosphatase. Pagets dis. of the bone 3- 28 y/o woman complains of tremors, nervousness, and weight loss, ECG shows atrial fibrillation. Hyperthyroidism 4- 62 y/o man with gastric cancer presents with velvety brown thickened skin of the body folds. Acanthosis nigricans

5- 9 y/o boy presents with pain in the medial side of the foot, X-ray shows a narrow scaphoid. Kohlers dis. 6- 5 y/o child presents with a painful rib swelling, X-ray shows a rounded osteolytic lesion. Eosinophilic granuloma 7- 62 y/o man had few episodes of complete blindness in one eye which lasted for 30 seconds each. TIA 8- 50 y/o man with intermittent claudication, impotence, and pain in the buttocks. LeRiche syn. 9- 2 y/o child is brought to you with a thin-walled cyst in the supraclavicular area. Cystic hygroma 10- 6 months after cardiac surgery, a 58 year old man develops fever, pericarditis, and an elevated ESR. Dresslers syn. 11- 8 y/o girl is brought to you with fever, cervical adenopathy, conjunctivitis, and a scarlatiniform rash on the upper chest. Kawasaki dis. 12- 32 y/o woman presents with severe right lower quadrant pain and a palpable adnexal mass, she has no fever and her hCG is negative. Ovarian cyst torsion 13- 24 y/o woman develops erythematous skin eruption following a week's course of antibiotic therapy for UTI. Erythema multiform 14- 62 y/o woman complains of jaw pain when chewing and diminished vision of her right eye. Temporal arteritis 15- 52 y/o man presents with severe midchest pain followed by bouts of vomiting, chest X-ray shows pneumomediastinum. Esophageal rupture 16- 73 y/o woman presents with sticky, yellowish vaginal discharge, the vaginal mucosa is thin and pale. Atrophic vaginitis 17- 9 y/o boy presents with recurrent episodes of abdominal pain and painful swelling in the hands. Hereditary angioedema 18- 62 y/o woman presents with severe joint disease, hepatosplenomegaly, and leg ulcers. Feltys syn. 19- 38 y/o man complains of severe headaches and blurred vision, his jaw is protruding and his tongue is enlarged. Acromegaly

20- 62 y/o man presents with a severe, slowly progressive abdominal pain, followed by a bloody diarrhea. Intestinal infarction 21- 45 y/o diabetic woman presents with numbness over the palmar side of the thumb, the index and middle fingers. Carpal tunnel syn. 22- 46 y/o man develops paralysis of the proximal and distal muscles of the leg after recovering from pneumonia. Guillain barre syn

Вам также может понравиться